Vous êtes sur la page 1sur 196

https://www.facebook.com/groups/Dharmo.Rakshati.

Rakshitah/ Page - 1 of 196

PREFACE
LL.B. Study Notes
305E Intellectual Property Law

Stay connected using facebook.


Source : Public domain print/ internet contents. URLs of some of such resources are
listed herein. Credits/ copyrights duly acknowledged.
22-Nov-2017. Version-1.3 compiled by ketan.bhatt@iitbombay.org in academic pursuit.
Follow URLs for details. Dedicated to students of the subject. No claim is made/ implied
about originality of this document, or its truthfulness.
Gujarat University Syllabus is in BOLD text. References to questions listed herein below,
are to such questions which were asked in Gujarat University examinations.
This PDF doc has navigation built-in. ie viewer shall be able to quickly move around the
PDF with mouse clicks. However, sometimes due to browser limitations, click-able links
may not work. In that case just download the PDF and open it in any standard PDF
viewer, eg Adobe Reader.
Lastly, | : | This doc also promotes the cause of law students. Do
suggest better answers and do point-out mistakes. Or alternately, feel free to update it
(for non-commercial purpose) as you deem fit. But, do keep sharing with the world.
Join https://www.facebook.com/groups/LLB.GujUni/ for study notes on any LLB subject.
Join https://www.facebook.com/groups/Dharmo.Rakshati.Rakshitah/
A happening place to discuss professional aspects of 'law' as it is actually practiced.
, ( ) Dharma, when protected, protects.
Scribd https://www.scribd.com/bhatt.net.in (repository of all study notes).

This is PREFACE. Menu ---> CONTENTS | Module-1 | Module-2 | Module-3 | Module-4

Refer : Bare acts are a good source, in any subject of law :


http://www.ipindia.nic.in/ <--- Controller General of Patents, Designs & Trade Marks.
https://www.icsi.edu/docs/webmodules/Publications/9.4%20Intellectual%20Property
%20Rights.pdf
http://www.wipo.int/edocs/pubdocs/en/intproperty/450/wipo_pub_450.pdf
http://epgp.inflibnet.ac.in/epgpdata/uploads/epgp_content/law/
http://shodhganga.inflibnet.ac.in/bitstream/10603/62276/7/chapter%202.pdf

CONTENTS | Module-1 | Module-2 | Module-3 | Module-4 http://duralex.bhatt.net.in/


https://www.facebook.com/groups/Dharmo.Rakshati.Rakshitah/ Page - 2 of 196

CONTENTS
305E Intellectual Property Law

Topic Page

Module-1 Introductory. 3

Module-2 Select Aspects of the Copyright Law in India. 63

Module-3 Intellectual Property Rights in Trademarks and Design. 95

Module-4 The Law of Intellectual Property: Patents. 148

Objectives of the course :


The importance of this Branch of their law is to be sufficiently realized in the Indian
Legal Education. Compendious courses on the law of copyright, trademarks and patents
are offered in few law schools as optional courses, but these do not either integrate the
significance of these subject matters under any comprehensive aspect of
"modernization" or "development " nor do they spread even emphasis between and
among the subject areas represented by these three interconnected bodies of the law.
The three areas are now internationally conceptualized as representing intellectual
property. It is often the case that while the law of patents and trademarks is referred to
as industrial property, the law relating to copyright is named intellectual property. While
both these terms could be suitably invoked, we here speak of intellectual property as
signifying all the three bodies of the law as well as the law on industrial designs.
Unlike other forms of property, intellectual property refers to regimes of legal
recognition of primarily the products of the mind or imagination. The subject matter of
property related rights is here pre-eminently based on mental labour the law relating to
intellectual property protects the right to mental labour.
The law confers right of proprietary nature on relative intellectual labour primarily on the
basis that it is in the interest of society and state to promote creativeness and
inventiveness. Limited monopoly provides incentive for greater inventive and innovative
efforts in society. An important aspect of the exploration in this course would be the
ways in which the laws strike a fair balance between the interests and rights of the
intellectual labourers on the one hand and organized industrial enterprises on the other.
Another dimension in a study of the ways in which this regime of laws militates against
or favours, community property in national cultures.
As concerns "modernization' crucial questions arise in the field of copyright protection in
computer software and hardware, interest electronic music and scientific research. Both
copyright , trademarks, design and patent law here relate basically to the law of unfair
competition and constitute an aspect of consumer protection and welfare not only in the
context of national perspectives but also in view of the waves of globalization already set
in. Both from the standpoint of human resources development, modernization and
justice it is important that the curricular change takes serious notice of these areas.

CONTENTS | Module-1 | Module-2 | Module-3 | Module-4 http://duralex.bhatt.net.in/


https://www.facebook.com/groups/Dharmo.Rakshati.Rakshitah/ Page - 3 of 196

Module-1 :
1) Introductory :
1.1) The meaning of intellectual property
1.2) Competing rationales of the legal regimes for the protection of intellectual
property
1.3) The main forms of intellectual property : copyright trademarks, patents,
designs
1.4) Other new forms such as plant varieties and geographical indications.
Introductions to the leading international instruments concerning
intellectual property rights : The Berne Convention, Universal Copyright
Convention, Union TRIPS the World Intellectual Property Rights
Organization (WIPO) and the UNESCO.
1.5) The status and position of IPRs in India in context with the International
Regime
1.6) Amendments in various legislations relating to IPRs India including
legislations of Patent, Copyright and Trademark in India and effects
thereof

This is Module-1. Menu ---> CONTENTS | Module-1 | Module-2 | Module-3 | Module-4

MODULE-1 QUESTIONS :

Explain the meaning of intellectual property and discuss its various forms, (Nov-
2014)
Discuss : New forms of intellectual property such as (i) plant varieties, and (ii)
geographical indications.
Explain in detail the meaning of Intellectual Property with illustrations and case-laws.
(Apr-2016)
Explain in detail the main forms of Intellectual Property. (Apr-2016)
Discuss : Distinction between Copyright, Patent, Trademark.
Discuss : Enforcement of IPRs.
Discuss : Need for dedicated team for Trade Marks Management/ Protection .
Discuss : Advantages and disadvantages of protecting IP.
Discuss : Competing rationales of legal regimes for protection of IP.
Discuss : The Berne Convention.
Discuss : Universal Copyright Convention.
Discuss : World Intellectual Property Rights Organization (WIPO) .

CONTENTS | Module-1 | Module-2 | Module-3 | Module-4 http://duralex.bhatt.net.in/


https://www.facebook.com/groups/Dharmo.Rakshati.Rakshitah/ Page - 4 of 196

Discuss : TRIPS and the UNESCO.


Discuss : (i) International legal regime protecting IPRs, (ii) Indian legal regime
protecting IPRs.
Discuss the development of Intellectual Property Law in India in the
international regime. (Dec-2015)
Explain the status and position of IPRs in India in context with the International
Regime. (Apr-2016)
Discuss : Assignment and Licensing of IP.
Discuss : Effects of legislations of Patent, Copyright and Trademark in India .
Discuss the amendments done in various legislations relating to intellectual
properly law in India and their effects, (Nov-2014)
Explain in detail the Amendments in various legislations relating to IPRs in India.
(Apr-2016)

This is Module-1. Menu ---> CONTENTS | Module-1 | Module-2 | Module-3 | Module-4

MODULE-1 ANSWERS :

Explain the meaning of intellectual property and discuss its various forms, (Nov-
2014)
Discuss : New forms of intellectual property such as (i) plant varieties, and (ii)
geographical indications.
Explain in detail the meaning of Intellectual Property with illustrations and case-laws.
(Apr-2016)
Explain in detail the main forms of Intellectual Property. (Apr-2016)
ANSWER :
Refer :
http://www.wipo.int/edocs/pubdocs/en/intproperty/450/wipo_pub_450.pdf
https://en.wikipedia.org/wiki/Intellectual_property
http://shodhganga.inflibnet.ac.in/bitstream/10603/62276/7/chapter%202.pdf
https://www.icsi.edu/docs/webmodules/Publications/9.4%20Intellectual
%20Property%20Rights.pdf
http://ccs.in/sites/default/files/ipr_india.pdf
Outline :
Intro
What is Intellectual Property ?

CONTENTS | Module-1 | Module-2 | Module-3 | Module-4 http://duralex.bhatt.net.in/


https://www.facebook.com/groups/Dharmo.Rakshati.Rakshitah/ Page - 5 of 196

Intangible nature of Intellectual Property


Intellectual Property as movable property and chose in actions
History of IPRs
Reasons for grant of IPRs
Forms of Intellectual Property
Copyright
Trademark
Industrial Design Right
Patent
Plant Varieties Rights (PVR) or Plant Breeder's Right (PBR)
Geographical Indications
Intro :
The twenty-first century will be the century of knowledge, the century of intellect.
A nations ability to translate knowledge into wealth and social good through
innovations will determine its future.
Innovations hold the key to the creation as well as processing of knowledge.
Consequently issues of generation, evaluation, protection and exploitation of
intellectual property would become critically important all over the world.
What is Intellectual Property ?
Intellectual property refers to creations of the mind :
eg,
inventions;
literary and artistic works; and
symbols, names and images used in commerce.
Intellectual property rights are like any other property right.
They allow creators, or owners, of patents, trademarks or copyrighted works to
benefit from their own work or investment in a creation.
The importance of intellectual property was recognized in
1883 Paris Convention for the Protection of Industrial Property ,
and 1886 the Berne Convention for the Protection of Literary and Artistic Works.
Both treaties are administered by the World Intellectual Property Organization
(WIPO).
These rights are outlined in Article 27 of the Universal Declaration of Human
Rights, which provides for the right to benefit from the protection of moral and
material interests resulting from authorship of scientific, literary or artistic
productions.

CONTENTS | Module-1 | Module-2 | Module-3 | Module-4 http://duralex.bhatt.net.in/


https://www.facebook.com/groups/Dharmo.Rakshati.Rakshitah/ Page - 6 of 196

Advantages of NOT taking any legal action to protect IP :


No legal concerns/fees.
Anyone can change and develop the idea freely.
Anyone can use an idea/ concept developed by others.
Potential to maintain competitiveness simply by being the first into the market.
Disadvantages of NOT taking any legal action to protect IP :
Someone else could potentially protect your original idea.
No guaranteed rewards for investors.
Intangible nature of Intellectual Property :
Intellectual Property has been conferred with rights in line with property rights in
almost all legal systems.
Property means the highest right a person can have to anything which does not
depend on anothers courtesy :
It includes ownership, estates, and interests in corporeal things
and also rights such as trademarks, copyrights, patents
and even rights in personem capable of transfer or transmission, such as debts;
and signifies a beneficial right to or a thing considered as having a money value,
especially with reference to transfer or succession, and to their capacity of being
injured.
Intellectual Property of whatever species is in the nature of intangible corporate
property. In each case it consists of bundle of rights in relation to certain material
objects created by the owner.
The term intellectual property has two parts, viz., intellectual and property .
Intellectual refers to the mind, and more particularly, to the efforts of the mind .
This part of the term tells us that this area of law is concerned with the realm of
human creation, novelty, and inventiveness. It relates to that sort of property
which a person creates through an application of his mind, rather than pre-existing
property that he somehow acquires.
Property is a bundle of rights in the hands of the owner .
Thus, intellectual property, like any other form of property,
is a bundle of rights, on efforts of mind, in the hands of the owner .
The owner of intellectual property can, therefore, deal with it in much the same
manner that the owner of any property can deal with his property: ie
i) It can be transferred, in whole, or in part, to another person;
ii) It can be exploited for commercial profit; and
iii) It can be subject to exclusive use by the owner.
Intellectual property rights are in essence negative rights ;

CONTENTS | Module-1 | Module-2 | Module-3 | Module-4 http://duralex.bhatt.net.in/


https://www.facebook.com/groups/Dharmo.Rakshati.Rakshitah/ Page - 7 of 196

this means that they allow one to prohibit certain acts with respect to their
intellectual assets.
IPRs are largely territorial rights,
exception being copyright, which is global in nature in the sense that it is
immediately available in all the members of the Berne Convention.
IPRs are awarded by the State and are monopoly rights ,
implying that no one can use these rights without consent of the right holder.
Intellectual Property as movable property and chose in actions :
Intellectual Property is a property in legal sense. IP is something that can be owned
and dealt with.
Most forms of IP are chose in action, rights that are enforced only by legal action
as opposed to possessory rights.
Intellectual property is treated on par with movable property,
and can be owned, transferred, licensed, and assigned, and is also subject to
taxation and stamp duty.
The distinguishing features of this property are its intangibility and non-
exhaustability by consumption.
Rights in intellectual property are granted by the State to an inventor, author, or
originator of products that reflect his intellectual effort and ingenuity.
Just as in the case of other rights, intellectual property rights carry with them a set
of duties that in some cases bind the owner of the rights himself, and in some
cases, bind other people.
For example,
if I own copyright in any training materials,
then everyone else has duty, not to make unauthorized copies, sale, or
distribution of these materials!
Likewise, if I have the exclusive patent right to manufacture and commercially
exploit my invention,
then I also have a duty, to ensure that the invention is available in sufficient
quantities to satisfy public needs, at affordable price.
If IPR owner does not fulfill his duties, the law provides for compulsory licensing,
or even revocation of my patent, in extreme cases.
History of IPRs :
While intellectual property law has evolved over centuries,
in the 19th century the term intellectual property began to be used,
in the late 20th century, it became commonplace in the majority of the world.
1808 : The first clear example of modern usage goes back as early as 1808, when
Intellectual Property was used as a heading title in a collection of essays.

CONTENTS | Module-1 | Module-2 | Module-3 | Module-4 http://duralex.bhatt.net.in/


https://www.facebook.com/groups/Dharmo.Rakshati.Rakshitah/ Page - 8 of 196

1845 : The term can be found used in an Oct-1845 Massachusetts Circuit Court
ruling in the patent case Davoll et al. v. Brown.
Justice Charles L. Woodbury wrote that,
"only in this way can we protect intellectual property, the labors of the mind,
productions and interests are as much a man's own...as the wheat he
cultivates, or the flocks he rears."
1960 : It was only after 1960 establishment of the World Intellectual Property
Organization (WIPO) as an agency of the United Nations,
that the term Intellectual Property really began to be used widely.
Reasons for grant of IPRs :
The philosophy behind grant of IPRs finds origin in the Lockes Theory of Property
according to which
labor should be rewarded.
Following are the main reasons for granting IPRs :
1. Incentive to invent;
2. To encourage disclosure;
3. Commercialization of technology through licensing; and
4. To increase dynamic efficiency.
5. To prevent misappropriation and unjust enrichment,
eg Confidential Information, Trade Secret, and Know-How.
6. Consequentiality justification :
When inventors, authors or artists have an exclusive right to reproduce and
sell their works, society benefits in consequence.
This proposition is based on two assumptions.
First, it assumes that such a right encourages inventors to invent, authors
to write and artists to paint.
Second, it presupposes that,
greater the quantity of inventions and creative works eventually released
into the public domain,
the more the public benefits through economic or cultural enrichment, or
enhanced quality of life.
ie, grant of IPRs results in desirable consequences.
7. Rights-based justification :
According to rights-based justifications for IPRs, property in intellectual works
is primarily a matter of justice rather than of public policy.
IPR laws exist to define and enforce the property rights but are not the source
of these rights;

CONTENTS | Module-1 | Module-2 | Module-3 | Module-4 http://duralex.bhatt.net.in/


https://www.facebook.com/groups/Dharmo.Rakshati.Rakshitah/ Page - 9 of 196

since to enjoy a property right over ones creative work is a human right.
According to such a view unauthorized use of somebodys invention or creative
work is an unfair and therefore illegal intrusion on the creator-proprietors
freedom to benefit from its use without interference.
Consequentiality justifications have inspired national IPR laws and policy making
far more than rights-based ones.
Forms of Intellectual Property :
Main forms of the intellectual property rights are:
a) Copyright which is available for artistic creative works in art, literary, music
etc.;
The term of Copyright is one of the most lengthiest, 50 years.
b) Trademark rights which are available for name, shape, logo etc.;
Term of a trademark is continuous subject to periodic renewal and no
cancellation action.
c) Designs rights which gives protection for the 3D objects like cutlery, trade
dress, motifs and patterns on fabric/ceramic etc.
Designs have the shortest term of 15 years10 years further extendable to
another 5.
d) Patent that grants monopoly to use and exploit a particular product or
process for a certain period;
Patent lasts for 20 yearsthough in practice it only starts to make money after
few years of grant.
Other forms of intellectual property rights are :
Plant Variety Rights (PVR) or Plant Breeder's Rights (PBR)
Geographical Indications Right
Copyright :
Copyright is a form of intellectual property, applicable to certain forms of creative
work. Copyrights frequently include reproduction, control over derivative works,
distribution, public performance, and "moral rights" such as attribution.
Copyright is a legal right created by the law of a country that grants the creator of
an original work exclusive rights for its use and distribution.
These exclusive rights are not absolute but limited by limitations and exceptions to
copyright law, including fair use.
Note : A major limitation on copyright is that copyright protects only the original
expression of ideas, and not the underlying ideas themselves.
While many aspects of national copyright laws have been standardized through
international copyright agreements, copyright laws vary by country.
Copyrights are usually granted for a limited time only. Typically, the duration of a

CONTENTS | Module-1 | Module-2 | Module-3 | Module-4 http://duralex.bhatt.net.in/


https://www.facebook.com/groups/Dharmo.Rakshati.Rakshitah/ Page - 10 of 196

copyright spans the author's life plus 50 to 100 years.


Some, but not all jurisdictions require "fixing" copyrighted works in a tangible form.
A copyright is often shared among multiple authors, each of whom holds a set of
rights to use or license the work, and who are commonly referred to as rights
holders.
Copyrights are considered territorial rights, which means that they do not extend
beyond the territory of a specific jurisdiction.
Some countries require certain copyright formalities to establishing copyright, but
most recognize copyright in any completed work, without formal registration.
Most jurisdictions recognize copyright limitations, allowing "fair" exceptions to the
creator's exclusivity of copyright and giving users certain rights.
The development of digital media and computer network technologies have
prompted reinterpretation of these exceptions, and inspired additional challenges
to copyright law's philosophic basis.
Generally, copyrights are enforced as a civil matter, though some jurisdictions do
apply criminal sanctions.
Advantages :
No official registration required. Comes into effect immediately.
No cost to marking something as copyright.
Disadvantages :
Does not protect ideas itself. Copyright protects only the way in which idea is
expressed.
To be effective it must be enforced. Involves legal costs.
Trademark :
A trademark, trade mark, or trade-mark is
a recognizable sign, design, or expression
which identifies products or services of a particular source from those of others ,
Usually, trademarks used to identify services are called service marks.
The trademark owner can be an individual, business organization, or any legal
entity.
A trademark may be located on a package, a label, a voucher, or on the product
itself.
For the sake of corporate identity, trademarks are often displayed on company
buildings, letterheads, etc.
The essential function of a trademark is,
to exclusively identify the commercial source or origin of products or services,
In other words,

CONTENTS | Module-1 | Module-2 | Module-3 | Module-4 http://duralex.bhatt.net.in/


https://www.facebook.com/groups/Dharmo.Rakshati.Rakshitah/ Page - 11 of 196

trademarks serve to identify a particular business as the source of goods or


services.
The use of a trademark in this way is known as trademark use.
Assuming there are no other trademark objections,
trademark rights generally arise out of the use of that sign in relation to certain
products or services.
Advantages :
Legally prevents others taking advantage of customer goodwill generated by TM
holder.
Disadvantages :
Having a registered a trade mark does not entitle you to the related internet
domain names.
The degree to which similar (not identical) trade marks infringe upon your
business is a matter for debate in civil action which has related costs.
Industrial Design Right :
Design rights started in the United Kingdom in 1787 with the Designing and
Printing of Linen Act.
An industrial design right is an intellectual property right that protects the visual
design of objects that are not purely utilitarian.
An industrial design consists of the
creation of a shape, configuration or composition of pattern or color, or
combination of pattern and color
in two or three-dimensional form containing aesthetic value .
An industrial design can be used to produce a product, industrial commodity or
handicraft.
Registering for an industrial design right is related to granting a patent .
Under the Hague Agreement Concerning the International Deposit of Industrial
Designs, a WIPO-administered treaty,
a procedure for an international registration exists.
To qualify for registration, the national laws of most member states of WIPO
require the design to be novel.
An applicant can file for a single international deposit with WIPO or with the
national office in a country party to the treaty.
The design will then be protected in as many member countries of the treaty as
desired.
Advantages :
Can be combined with legal protection from copyright .
Protect design innovation for up to 25 years.

CONTENTS | Module-1 | Module-2 | Module-3 | Module-4 http://duralex.bhatt.net.in/


https://www.facebook.com/groups/Dharmo.Rakshati.Rakshitah/ Page - 12 of 196

Disadvantages :
Cannot be applied to designs that concern how a product works or which are not
visible in normal use ascetics only
ID Right can be easily subverted by slight amendments to the basic design.
Patent :
Patent is a form of intellectual property.
A patent is a set of exclusive rights granted by a sovereign state,
to an inventor or assignee,
for a limited period of time,
in exchange for detailed public disclosure of an invention .
An invention is a solution to a specific technological problem,
and is a product or a process.
According to national laws & international agreements, there may be variations in,
the procedure for granting patents,
requirements placed on the patentee, and
extent of the exclusive rights.
Essentials for grant of a patent :
Patent application must include one or more claims that define the invention.
Each of the claim defines a specific property right.
These claims must meet relevant patentability requirements, such as,
novelty, usefulness, and non-obviousness.
What is included in Patent Right? The exclusive right granted to a patentee,
the right to prevent others, or at least to try to prevent others,
from commercially making, using, selling, importing, or distributing a patented
invention.
Under the World Trade Organization's (WTO) TRIPS Agreement,
patents should be available in WTO member states for any invention,
in all fields of technology,
provided they are new, involve an inventive step, and are capable of industrial
application.
There are variations on what is patentable subject matter from country to country,
even among WTO member states.
TRIPS provides that the term of protection available should be minimum 20 years .
Advantages :
A patent gives the legal right to stop others from using your invention.
Its existence may be enough to deter competitors

CONTENTS | Module-1 | Module-2 | Module-3 | Module-4 http://duralex.bhatt.net.in/


https://www.facebook.com/groups/Dharmo.Rakshati.Rakshitah/ Page - 13 of 196

Buys time (20 years) in which an inventor can develop a market to the product
or process.
Attractive to investors as it limits competition.
Disadvantages :
Takes time and money to establish a patent.
All patents have to be researched to ensure there are no existing patents of a
similar nature involves legal fees.
Not possible to guarantee that a patent (once granted) is valid, it can be legally
challenged and revoked with no refunds.
It is still up to the inventor to protect a patent.
In case of patent infringement, patentee has to follow remedy elsewhere.
Involves costs. The patent office does not take sides.
Grant of patent is no indication that the invention has merit or commercial value.
Some products and process can be varied slightly to get around the exact
wording of patents.
Plant Varieties Rights (PVR) or Plant Breeder's Right (PBR) :
https://en.wikipedia.org/wiki/Plant_breeders%27_rights
http://www.wipo.int/sme/en/documents/upov_plant_variety_fulltext.html
Intro :
New varieties of plants with improved yields, higher quality or better resistance
to pests and diseases
increase quality and productivity in agriculture, horticulture and forestry, while
minimizing the pressure on the environment.
The tremendous progress in agricultural productivity in various parts of the
world is largely based on improved plant varieties.
More so, plant breeding has benefits that extend beyond increasing food
production.
Development of breeding programs for certain endangered species can remove
the threat to their survival out in nature, as in the case of medicinal plants.
Need/ importance/ benefits of Plant Variety Rights :
While the process of plant breeding requires substantial investments in terms of
money and time,
once released, a new plant variety can be easily reproduced in a way that
would deprive its breeder of the opportunity to be rewarded for his
investment.
Clearly, few breeders are willing to spend years making substantial economic
investment in developing a new variety of plants if there were no means of
protecting and rewarding their commitment.

CONTENTS | Module-1 | Module-2 | Module-3 | Module-4 http://duralex.bhatt.net.in/


https://www.facebook.com/groups/Dharmo.Rakshati.Rakshitah/ Page - 14 of 196

Therefore, an effective system for the protection of plant variety is essential to


encourage breeders to invest in plant breeding and contribute to the
development of agriculture, horticulture and forestry and for the benefit of
society as a whole.
With these rights, the breeder can choose to become the exclusive marketer of
the variety, or to license the variety to others.
Breeders can bring suit to enforce their rights and can recover damages for
infringement.
What is PVR?
Plant Variety Rights (PVR) are rights granted to the breeder of a new variety of
plant that give the breeder exclusive control over
the propagating material (including seed, cuttings, divisions, tissue culture)
and harvested material (cut flowers, fruit, foliage)
of a new variety for a number of years.
PVR, also called a "plant breeder's right" (PBR),
is a form of intellectual property right granted to breeder of new plant variety
According to this right, certain acts concerning the exploitation of the protected
variety require the prior authorization of the breeder.
How Does the PBR System Work?
Any person who creates, or discovers and develops, a plant variety may apply
for PBR (i) in his country OR (ii) under the 1991 Act of the UPOV Convention.
"UPOV" is an intergovernmental organisation based in Geneva, Switzerland.
The acronym UPOV is derived from the French name of the organisation
namely, Union Internationale pour la Protection des Obtentions Vgtales".
Once the PBR has been granted to the breeder, it means in practice that the title
holder is the owner of the variety
and anyone else who wants to commercialise that protected variety requires
the authorisation of the holder of the PBR.
This authorisation is normally in the form of a license agreement between the
title holder and those who sell the variety.
Essentials for grant of a PBR :
For a variety to be protected, it must be novel, distinct, uniform and stable, and
must have a suitable denomination.
A variety is :
novel if it has not been commercialized for more than one year in the country
of protection;
distinct if it differs from all other known varieties by one or more important
botanical characteristics, such as height, maturity, color, etc.;

CONTENTS | Module-1 | Module-2 | Module-3 | Module-4 http://duralex.bhatt.net.in/


https://www.facebook.com/groups/Dharmo.Rakshati.Rakshitah/ Page - 15 of 196

uniform if the plant characteristics are consistent from plant to plant within
the variety;
stable if the plant characteristics are genetically fixed and therefore remain
the same from generation to generation, or after a cycle of reproduction in the
case of hybrid varieties.
The breeder must also give the variety an acceptable "denomination", which
becomes its generic name and must be used by anyone who markets the
variety.
Procedure to obtain PBR :
To be granted a PBR, it is necessary to file an application for examination by the
designated authority.
Procedure is simple and well defined, hence, the breeder is usually able to file an
application without the services of any agent acting on his behalf.
A PBR will be granted if the requirements are fulfilled.
Once granted, the PBR is valid for a minimum of
25 years in the case of trees and vines and
20 years in the case of other crops
from the date of granting the PBR.
IF PBR is granted by a country THEN, it is valid in the territory of the country
where it was granted
while in the case of intergovernmental organisations (eg CPVO Europe) which
grant PBRs, validity applies in all the member states of that organisation.
For example, the Community Plant Variety Office (CPVO) grants PBR, which
are valid in all member states of the European Union.
Examination of the variety to be protected :
The variety is submitted to the plant variety office,
who grow it for one or more seasons, to check that it is distinct, stable, and
uniform.
If these tests are passed, exclusive rights are granted for a specified period
(typically 20/25 years (or 25/30 years), for trees (and vines).
Misc Provisions :
Annual renewal fees are required to maintain the granted PVR rights.
Exemptions from PVRs :
Farmers may store the production in their own bins for their own use as seed,
but this does not necessarily extend to brown-bag sales of seed.
However, further sales for propagation purposes are not allowed without the
written approval of the breeder.
Compulsory licensing provisions are there to assure public access to protected

CONTENTS | Module-1 | Module-2 | Module-3 | Module-4 http://duralex.bhatt.net.in/


https://www.facebook.com/groups/Dharmo.Rakshati.Rakshitah/ Page - 16 of 196

varieties,
IF the national interest requires it and the breeder is unable to meet the
demand.
Geographical Indications :
What is a Geographical Indication?
A geographical indication,
is a sign used on goods that have a specific geographical origin
and possess qualities or a reputation due to that place of origin .
eg, name of the place of origin of the goods.
Agricultural products typically have qualities that derive from their place of
production and are influenced by specific local geographical factors, such as
climate and soil.
Whether a sign functions as a geographical indication is a matter of national law
and consumer perception.
The use of geographical indications is not limited to agricultural products .
They may also highlight place of origin, or specific manufacturing skills and
traditions.
eg Switzerland or Swiss, perceived as a geographical indication in many
countries for products made in Switzerland and, in particular, for watches.
What is an appellation of origin?
An appellation (action of giving a name to someone or something) of origin,
is a special kind of geographical indication used on products that have a
specific quality exclusively or essentially due to the geographical environment
in which the products are produced.
Geographical indication is genus. Appellations of origin is its specie .
Appellations of origin are protected in those states which are party to the Lisbon
Agreement for the Protection of Appellations of Origin and their International
Registration are,
Darjeeling for tea grown in the tea gardens of Darjeeling, India
Bordeaux for wine produced in the Bordeaux region of France,
Prosciutto di Parma or Parma ham for ham produced in the Parma
province of Italy
or Habana for tobacco grown in the Havana region of Cuba.
Why do geographical indications need protection?
Geographical indications are understood by consumers to denote the origin and
quality of products.
Many of them have acquired valuable reputations which, if not adequately
protected, may be misrepresented by commercial operators.

CONTENTS | Module-1 | Module-2 | Module-3 | Module-4 http://duralex.bhatt.net.in/


https://www.facebook.com/groups/Dharmo.Rakshati.Rakshitah/ Page - 17 of 196

False use of geographical indications by unauthorized parties, for example


Darjeeling for tea that was not grown in the tea gardens of Darjeeling, is
detrimental to consumers and legitimate producers.
the consumers are deceived into believing they are buying a genuine product
with specific qualities and characteristics,
and original producers are deprived of valuable business and suffer damage to
the established reputation of their products.
Difference between a geographical indication and a trademark :
A trademark is a sign used by a company to distinguish its goods and services
from those produced by others. It gives its owner the right to prevent others
from using the trademark.
A geographical indication guarantees to consumers that a product was
produced in a certain place and has certain characteristics that are due to that
place of production.
A trademark is given to a single juristic person,
A geographical indication may be used by all producers who make products
that share certain qualities in the place designated by a geographical
indication.
Generic geographical indication :
If the name of a place is used to designate a particular type of product, rather
than to indicate its place of origin,
THEN the term no longer functions as a geographical indication.
For example, Dijon mustard, a kind of mustard that originated many years ago
in the French town of Dijon, has, over time, come to denote mustard of that kind
made in many places.
Hence, Dijon mustard is now a generic indication and refers to a type of
product, rather than a place.
How are geographical indications protected?
Geographical indications are protected in accordance with
national laws,
and under a wide range of concepts, such as
laws against unfair competition,
consumer protection laws,
laws for the protection of certification marks
or special laws for the protection of geographical indications or appellations
of origin.
In essence, unauthorized parties may not use geographical indications if such
use is likely to mislead the public as to the true origin of the product.

CONTENTS | Module-1 | Module-2 | Module-3 | Module-4 http://duralex.bhatt.net.in/


https://www.facebook.com/groups/Dharmo.Rakshati.Rakshitah/ Page - 18 of 196

Applicable sanctions range from court injunctions preventing unauthorized use to


the payment of damages and fines or, in serious cases, imprisonment.
What is WIPOs role in the protection of geographical indications?
WIPO administers a number of international agreements that deal partly or
entirely with the protection of geographical indications
in particular, the Paris Convention and the Lisbon Agreement.
WIPO meetings offer Member States and other interested parties the opportunity
to explore new ways of enhancing the international protection of geographical
indications.

This is Module-1. Menu ---> CONTENTS | Module-1 | Module-2 | Module-3 | Module-4


GO TO MODULE-1 QUESTIONS.
GO TO CONTENTS.

Discuss : Distinction between Copyright, Patent, Trademark.


ANSWER :
Refer :
http://www.saprlaw.com/taxblog/copyright_final.pdf
Intro :
Copyrights, trademarks, and patents are commonly referred to as intellectual
property.
Each one gives the owner exclusive rights to the work, meaning the owner has the
right to prevent anyone else from using their work.
Each one deals in different spheres and the main difference is as follows :
Concepts :
a. A copyright protects the expression of a persons ideas.
Copyright protection is given to creative works like writing, computer programs,
music, lyrics, graphic designs, sculpture, photographs, movies, and sound
recordings.
The expression must be original, which, in this context, means a work that is
not an copy of another work.
b. To qualify for a patent, an invention must be novel, useful & non-obvious.
Non-obvious means a person who is in the field and understands the subject,
views the invention as a surprising and significant development in the field.
c. A trademark protects recognizable sign, design or expression which is
used to identify origin of a product or a service.
A trademark describes something and is not the thing being described.

CONTENTS | Module-1 | Module-2 | Module-3 | Module-4 http://duralex.bhatt.net.in/


https://www.facebook.com/groups/Dharmo.Rakshati.Rakshitah/ Page - 19 of 196

An example of a trademark would be a corporate identity, such as a logo, which


is placed on products to inform consumers that the product originated from that
particular company.
Illustration, Mr. X invents a plane that can travel in both the atmosphere as well
as space.
Mr. X would get patent for the invention of such a plane as the idea is novel and
non-obvious and fulfills other requirements like usefulness and technical
solution.
Then, Mr. X puts X symbol in his plane implying his creation or show causing
the source. This symbol would be eligible for Trademark.
Then, Mr. X decides to write a book on the invention and provides a CD with it.
The expression of the book and CD is protected under the copyright.
Registration :
Copyright is automatic. As soon as you create a work, you have copyright
protection. Registration is necessary in order to defend that protection, but the
registration is not what creates the copyright.
Trademarks and patents come into being only when you register them and your
registration is approved. The approval is not automatic.
Trademark may become automatic under the common law after the usage of the
particular mark for a certain number of years.
Patents : Registration is a MUST. Else, it is considered that patent has been
waived.
Terms of IPR :
As a general rule, copyrights last for lifetime, plus an additional sixty years.
A patent will last for twenty years after the date of application for the patent.
Trademarks are issued for a finite period, but they can be renewed. In theory, a
trademark could last forever.
Renewal :
Once copyrights or patents expire, they cannot be revived. Copyright and patent
after the expiry of certain period would come to public domain and can be copied
by anyone constituting no infringement.
Trademark lasts forever until renewed. They expires on non-renewal.
Territorial nature :
A copyright is generally, technically territorial, or only good within the country of
origin. Most nations, however, have agreements with other countries to honour
each others' copyrights. Not every country shares such relationships, however, and
a few countries provide little or no protection for works produced in other nations.
Patent and Trademark are generally has international character .

CONTENTS | Module-1 | Module-2 | Module-3 | Module-4 http://duralex.bhatt.net.in/


https://www.facebook.com/groups/Dharmo.Rakshati.Rakshitah/ Page - 20 of 196

Overlap between copyright and trademark :


There is some overlap between Copyright and trademark.
If you paint a picture, that picture is protected by copyright .
What if someone sees your picture and wants to use it as a logo for the company
they run?
Now, that same picture could be a trademark.
The pictures status as a trademark does not affect its copyright status .
Both protections will be there, and it is only a question of which protection IPR
owner uses to enforce his rights in the work. That, in turn, depends on how
those rights are violated.
If your picture is just copied, its a copyright infringement.
If its used to sell a different product, its probably a trademark infringement as
well.

This is Module-1. Menu ---> CONTENTS | Module-1 | Module-2 | Module-3 | Module-4

Discuss : Enforcement of IPRs.


ANSWER :
Refer :
http://shodhganga.inflibnet.ac.in/bitstream/10603/62276/7/chapter%202.pdf
Intro :
Intellectual property enforcement mechanism is dealt with under,
1. TRIPS (Trade-Related Aspects of Intellectual Property Rights Agreement) .
2. National legislations,
Importance of Enforcement Mechanism :
Having intellectual property laws is not enough. They have to be enforced. This is
covered in Part 3 of TRIPS.
Intellectual Property Rights are a central feature of knowledge economy and with
their inventiveness and creativity;
Various of any economy generate wealth of new ideas, designs and creative
materials across all sectors.
Considerable private and public resources are invested to encourage innovative and
creative assets in the form of IPRs.
But if these efforts are not safeguarded through effective enforcement mechanism
the very purpose of maintaining IP system will be defeated.
Hence an effective Intellectual Property Right (IPR) legal regime/ system is
important to any trade because it provides confidence to business that

CONTENTS | Module-1 | Module-2 | Module-3 | Module-4 http://duralex.bhatt.net.in/


https://www.facebook.com/groups/Dharmo.Rakshati.Rakshitah/ Page - 21 of 196

rights will be respected and that profits will be returned to IPR holders when
goods and services are traded in the region.
Accessible, sufficient and adequately funded arrangements for the protection of
rights are crucial in any worthwhile intellectual property system.
Strong IPR systems boost economic growth, promote investment and develop
industries that promote creativity and innovation.
TRIPS call for enforcement procedures which permit effective action against IP
infringement.
1. IPR enforcement under TRIPS :
<read from TRIPS Agreement in this doc>
In short, the TRIPS agreement says
Governments have to ensure that intellectual property rights can be enforced
under their laws,
and that the penalties for infringement are tough enough to deter further
violations.
Procedures for enforcement must be fair and equitable, and not unnecessarily
complicated or costly. They should not entail unreasonable time-limits or
unwarranted delays.
People involved in enforcement should be able to ask a court to review an
administrative decision or to appeal a lower courts ruling.
2. IPR enforcement under National legislations :
Remedies for Infringement under Indian Laws fall in two categories :
A. Civil Actions
B. Criminal Actions
Indian IP laws contain enough provisions for administrative, civil and criminal
remedies for infringement of the rights.
However, the problem lies in effective enforcement, especially due to scientific and
technological advancements, which have led to newer ways of circumventing laws.
The rapid technological evolution and its effect on enforcement calls for newer and
effective ways to handle the situation.
2A. Civil Actions :
When any IP is infringed the right owner gets civil cause of action.
After filing of suit, plaintiffs can seek ad interim and interim relief, including,
injunctions,
orders for discovery and inspection,
orders for interrogatories.
Ad interim order operates till interim relief application is decided,

CONTENTS | Module-1 | Module-2 | Module-3 | Module-4 http://duralex.bhatt.net.in/


https://www.facebook.com/groups/Dharmo.Rakshati.Rakshitah/ Page - 22 of 196

whereas interim order operates till suit is decided.


Ad-interim and interim injunctions are granted under Order 39, Rules 1 and 2,
read with Section 151 of the Code of Civil Procedure, 1908.
Infringement and passing-off actions can be instituted by filing a suit in the
appropriate court.
tort of passing off protects the goodwill of a trader from misrepresentation.
All IP laws state the appropriate court in which such suits can be instituted.
eg, under the TM Act, suits for trademark infringement or passing off can be
filed in the district court within the local limits of whose jurisdiction alleged
infringement occurred,
Court cases often reach a final hearing after twelve to sixteen years from the
date of their filing. Therefore, obtaining an interim injunction becomes crucial for
the plaintiffs, especially in intellectual property lawsuits.
The damages are awarded only after the final hearing .
Courts also grant injunctions in a quia timet (anticipatory) action if the plaintiff
proves that defendants activities or proposed activities would lead to violation of
plaintiffs rights.
Supreme Court in Wander Ltd. v. Antox India (P) Ltd laid down the principles for
the granting of an interim injunction.
For the grant of such ad interim and interim orders, the plaintiff has to show
that he has a prima facie case,
that the balance of convenience is in his favor,
and the hardship suffered by the plaintiff would be greater if the order is not
granted.
If the plaintiff is able to convince the Court of these points, then plaintiff can
obtain an ad interim and interim injunction within a couple of days of filing of the
suit.
Some courts also grant ex parte injunctions if a strong case is made.
However, Ex-parte injunction is issued when formalities like notice will defeat
the purpose of suit and if there is perpetual damage to the property.
In granting an ex-parte order the basic safeguards of equity must be strictly
enforced.
Generally, a plaintiff is required to give at least forty-eight hours notice to the
defendant for a hearing of the interim application.
If the defendant appears before the court, he may be granted further time to file
his reply and the plaintiff in turn may be allowed to file his response to the
defendants reply.
The hearing of the interim applications could go on for three to four days,

CONTENTS | Module-1 | Module-2 | Module-3 | Module-4 http://duralex.bhatt.net.in/


https://www.facebook.com/groups/Dharmo.Rakshati.Rakshitah/ Page - 23 of 196

depending upon the complexity of the matter.


Both the parties have the liberty to file an appeal from the interim order and
subsequently the parties may have to fight the matter even up to the Supreme
Court of India.
The appellate court also has the power to grant interim orders pending the final
hearing of appeal.
Indian courts have realized the importance of protecting IP and have started
granting innovative orders.
Recently the courts have started granting punitive and exemplary damages in
the intellectual property law matters.
In Time Incorporated v. Lokesh Srivastava and Anr, the Delhi Court observed :
This Court has no hesitation in saying that
the time has come when the Courts dealing actions for infringement of
trademarks, copy rights, patents etc
should not only grant compensatory damages but award punitive damages
also with a view to discourage and dishearten law breakers who indulge in
violations with impunity out of lust for money
so that they realize that in case they are caught, they would be liable not
only to reimburse the aggrieved party but would be liable to pay punitive
damages also, which may spell financial disaster for them.
2B. Criminal Actions :
Intellectual Property Rights can be enforced by initiating infringement actions in
civil courts or through criminal prosecution.
Procedure for both civil and criminal actions has been specifically set out under
the relevant IP legislations governing the specific form of IP.
The IP statutes provide for punishments and fines in respect of infringements of
different IPs.
It may be noted that the costs of filing, registration and litigation are relatively
low in India as compared to foreign countries but it is felt that the legal
procedures in India can take quite longer time.
Over the years various decisions passed in favour of foreign companies against
local infringers have demonstrated the Indian judiciarys impartial approach.
In copyright and trademark infringement cases (which come under criminal
litigation) courts routinely tend to award damages.
In India, criminal action is normally the result of a complaint made by rightful
owners to magistrates or police authorities.
As in other commonwealth countries, in India also state becomes a party to such
criminal proceedings against infringers.

CONTENTS | Module-1 | Module-2 | Module-3 | Module-4 http://duralex.bhatt.net.in/


https://www.facebook.com/groups/Dharmo.Rakshati.Rakshitah/ Page - 24 of 196

This is Module-1. Menu ---> CONTENTS | Module-1 | Module-2 | Module-3 | Module-4

Discuss : Need for dedicated team for Trade Marks Management/ Protection
ANSWER :
Refer :
http://www.legalserviceindia.com/trademarks-copyrights/trade
%20markmainpage.htm
http://epgp.inflibnet.ac.in/epgpdata/uploads/epgp_content/law/08._intellectual_pr
operty_law/27._intellectual_property_assignment_and_licensing_/et/5795_et_27_
et.pdf
<Note : This discussion is Trade Marks. However, same applies to other IPRs as well>
Need for dedicated team for Trade Marks Management/ Protection :
Trade Mark is a valuable asset for any business activity.
Trademarks need to be nurtured just like human beings.
Need for dedicated team for Trade Marks Management/ Protection, can not be over
emphacised.
However, if the creation of an exclusive trademark department is not economical
for any reason,
then it is better to entrust this task to a Trademark Attorney who should be
properly instructed to maintain all particulars and papers with regard to
trademarks either on retainer or work-to-work basis.
Trademarks management comprises two aspects :
1. Trademark Policy
2. Trademark Protection
1. Trademark Policy :
It is a marketing function. Normally the marketing personnel of an organization will
take care of this trademark policy letter known as ' Brand Management
2. Trademark Protection :
It is a legal function. Hence, it is better to integrate the trade marks department
with the legal department of an organization.
The principal duty of the Trademarks Department is to protect and administer the
trademark of the company
i.e. by getting registration under the relevant laws of a particular country, the
country of registration, the list and classes of goods and the services covered ,
renewals, action against the infringes and dishonest users and so on.
The trademarks department has an additional task in advising the marketing
personnel
in the choice of new trademarks,

CONTENTS | Module-1 | Module-2 | Module-3 | Module-4 http://duralex.bhatt.net.in/


https://www.facebook.com/groups/Dharmo.Rakshati.Rakshitah/ Page - 25 of 196

their protect ability


and their availability,
and also in the legal aspects of trademark policy.
The Trademarks Department may also be entrusted with
managing the licensing of trademarks,
drafting license agreements, user agreement for use of trademarks
and also to look after other areas of Intellectual Property such as Patents,
Designs, Copyrights and technical know how.

Functions of the trade marks department / management :


Advise the marketing department with regard to the choice of a new trademark.
Legal clearance of a new trade mark by conducting searches in the Trademarks
Registry and also in the market places with regard to the availability of identical or
similar marks in respect of similar goods and services.
Submit trademark applications and advise the company to go for registration in a
country where the goods are to be exported or sold.
Since there is globalization of industry and trade, it is better to seek International
protection of the trademarks and other Intellectual Property.
Advise the company for proper use of trademarks after obtaining registration in
order to avoid the attack on the registered trademarks on the ground of non-use
by business competitors.
Initiate legal action against the infringes by filing civil suits or criminal complaint
against the infringes and dishonest traders.
Get competent authorities to conduct search and seizure/ raid the premises when
the infringed or spurious goods are being manufactured or marketed.

CONTENTS | Module-1 | Module-2 | Module-3 | Module-4 http://duralex.bhatt.net.in/


https://www.facebook.com/groups/Dharmo.Rakshati.Rakshitah/ Page - 26 of 196

To maintain individual files for each and every trademark of the company for easy
reference.
It is better to computerize the Trademarks Department by creating a software for
this kind of ' Trademarks Management '.
Appoint/ retain competent lawyers to safeguard IPR assets through legal action.

This is Module-1. Menu ---> CONTENTS | Module-1 | Module-2 | Module-3 | Module-4

Discuss : Advantages and disadvantages of protecting IP.


ANSWER :
Refer :
https://www.icsi.edu/docs/webmodules/Publications/9.4%20Intellectual
%20Property%20Rights.pdf
http://shodhganga.inflibnet.ac.in/bitstream/10603/62276/7/chapter%202.pdf
https://www.researchgate.net/publication/228871485_The_Rationales_for_Intellec
tual_Property_Rights_The_Twenty-First_Century_Controversies
https://www.researchgate.net/publication/272241887_Aaron_Swartz_and_the_Bat
tles_for_Freedom_of_Knowledge
http://ccs.in/sites/default/files/ipr_india.pdf
Moot question here is :
Shall intellectual properties be protected by legal regimes?
and IF yes, then to what extent?
Outline :
Productive cycle of innovation and invention.
Need for legal regimes protecting IPRs.
Brief arguments for & against protecting IPR under a strong legal regime.
A. Advantages of legal regimes protecting IPRs.
B. Disadvantages of legal regimes protecting IPR.
C. Logical conclusion on the extent to which IPRs shall be protected.
Way To Go ! Copyleft & Creative Commons license (CC).
Productive cycle of innovation and invention :
Intellectual property is one of the valuable assets in commercial transactions,
eg IP licensing in cases of,
joint ventures, foreign collaborations, manufacturing, purchase or distribution
agreements, or mergers and acquisitions.
Licences to use IP, are often combined with transfers of know-how and are

CONTENTS | Module-1 | Module-2 | Module-3 | Module-4 http://duralex.bhatt.net.in/


https://www.facebook.com/groups/Dharmo.Rakshati.Rakshitah/ Page - 27 of 196

increasingly an important term in technology transactions.


These licences provide royalty revenues to the owner of the Intellectual Property,
and distribute products and technologies to licensees who might not otherwise
have had access to them.
In such transactions, the licensees may also gain rights to create improvements or
derivative works and to develop their own IP assets, which can then be cross-
licenced or licenced to others.
This creates a very productive cycle of innovation & invention and significantly
contributes to economic and social development.
Need for legal regimes protecting IPRs : Following points strongly point to strong
need for legal regimes (national & international) protecting IPRs :
Intellectual property laws gives people the right to own and profit from their
artistic, scientific and technological creations for a designated period of time.
Countries have enacted laws to protect intellectual property for two main reasons .
One, to give statutory expression to the moral and economic rights of creators in
their creations and the rights of the public in access to those creations.
Two, to promote creativity and the dissemination and application of its results,
Due to growing investments in innovation and the globalization of economic
activities,
global demand for patents has increased from 8,00,000 applications in the early
1980s to 18,00,000 in 2009.
IP contribute to economic and social development.
Intellectual property assets are used not only in business transactions, but are also
traded in their own right,
for example there are more and more online transactions (buy, sell, license) of
patents and other forms of Intellectual Property.
The buyers and sellers of intellectual property manage their intellectual property
just as any other financial assets.
Brief arguments for & against protecting IPR under a strong legal regime :
Intellectual property rights are the subject matter of many a fierce debate where
the proponents of either side profess their case ardently.

Arguments in favour of IPR Arguments against IPR

1 Incentive to innovate : IPR Poses serious roadblocks to innovation :


provides an incentive to IPRs may occasionally retard innovation. eg
innovate, as intellectual in order to improve on an existing product
property rights increase the that is patented, a person would have to
expected returns from an seek the patentees permission. A good deal
innovation by increasing of great art would not have been created

CONTENTS | Module-1 | Module-2 | Module-3 | Module-4 http://duralex.bhatt.net.in/


https://www.facebook.com/groups/Dharmo.Rakshati.Rakshitah/ Page - 28 of 196

Arguments in favour of IPR Arguments against IPR

profitability. Hence more under a strict copyright regime.


resources will be devoted to the Illustration : Shakespeare took the works
particular line of work. of others and created greater works. Under
todays copyright regime Shakespeares legal
bills would have been staggering!

2 Lockes Moral desert theory : Leggett's objection : If you assert an


Every man has a property in his exclusive right to a particular idea you
own person. ie the fruits of a cannot be sure the very same idea did not at
mans labour belongs to him. the same moment enter some other mind.
Individual must be permitted Thus these rights can only be justified if they
the fruits of his mental and are implemented in such a way that rights of
physical labour. an individual are protected without infringing
on another.

3 Kant & Hegel : Personality Palmers criticism : Palmer counters this


theory : If ones artistic by saying that if a work of art were part of
expressions are synonymous an individuals personality, then they would
with ones personality, then they cease to exist after the person died.
are deserving of protection just
as much as the physical person
is deserving of protection since
in a sense they are a part of
that physical person.

4 Bentham & Mill : Utilitarian Weak argument : Utilitarian arguments can


theory : According to this be cut for as well as against need to protect
theory, objective of any policy IPRs. The utility gains from increased
should be the attainment of the incentives for innovation must be weighed
greatest good for the greatest against the losses incurred from
number. monopolisation and their diminished
diffusion. Benefits gained cannot be
measured against the losses suffered.

5 Incentive to produce : IPR Creates artificial scarcity : IPR creates


gives researchers and inventors artificial scarcity through a monopoly on
incentive to continue invent various products (which implies a restricted
better and more efficient output and higher prices). Illustration :
products. since its establishment in 1875, the US
company AT & T collected patents in order to
ensure its monopoly on telephones. This
slowed down the introduction of radio for
about 20 years.

CONTENTS | Module-1 | Module-2 | Module-3 | Module-4 http://duralex.bhatt.net.in/


https://www.facebook.com/groups/Dharmo.Rakshati.Rakshitah/ Page - 29 of 196

A. Advantages of legal regimes protecting IPRs : Importance of IPRs :


Knowledge and inventions have played an important role in economic growth of the
countries. The progress & welfare of humanity rest on its capacity to create and
invent new works in the areas of technology and culture. There are several compelling
reasons for legal regimes protecting IPRs.
Intellectual property rights reward creativity and human endeavor, which fuel the
progress of humankind.
eg None of the multi-billion dollar film, recording, publishing and software
industries which bring pleasure to millions of people worldwide would not
exist without copyright protection.
The intellectual property regime helps strike a balance between the interests of
innovators and the public interest, providing an environment in which creativity and
invention can flourish, for the benefit of all.
Without the rewards provided by IPRs,
researchers and inventors would have little incentive to continue producing
better and more efficient products for consumers.
Consumers would have no means to confidently buy products or services without
reliable, international trademark protection and enforcement mechanisms to
discourage counterfeiting and piracy.
The promotion and protection of intellectual property,
spurs economic growth,
creates new jobs and industries,
and enhances the quality and enjoyment of life.
Legal regime for protection of IPR ensure that products are authentic, and of the
high-quality that consumers recognize and expect.
The legal protection of new creations encourages the commitment of additional
resources for further innovation.
An efficient and equitable intellectual property regime can help all countries to
realize intellectual propertys potential as a catalyst for economic development and
social and cultural well-being.
B. Disadvantages of legal regimes protecting IPR :
Sometimes, IPR may be granted to frivolous works that truly dont deserve any
protection under the law.
For example, http://www.nydailynews.com/news/national/amazon-patents-
technique-seamless-white-background-article-1.1808939
Dubbed as the silliest patent ever awarded, on 18-Mar-2014 the U.S.
Patent and Trademark Office granted Patent No. 8,676,045 titled Studio
Arrangement to Amazon.

CONTENTS | Module-1 | Module-2 | Module-3 | Module-4 http://duralex.bhatt.net.in/


https://www.facebook.com/groups/Dharmo.Rakshati.Rakshitah/ Page - 30 of 196

Amazons patent claim details,


exactly how to position lighting, the camera and the subject to create a
near perfect seamless white background.
The patent may already be infringed by millions of people .
Protecting IPRs can be very expensive . IPR lawyers, court costs, settlement fees,
filing fees and numerous other costs can mount very quickly, making protection of
IPRs expensive even for very large companies.
Sometimes, an innovator may not be allowed to harvest the fruits of creativity ,
if such intellectual products already happens to have been patented previously
by somebody else
Sometimes protection of IPRs may lead to the effect of slowing down innovation
and progress. For example,
Had the patent system prevailed during the time when wheel was invented
it would have restricted others from freely using the invention,
this would have seriously restricted the development of mankind as many
important, whether small or big improvements, immediately followed after the
invention of wheel.
The only party in this copyright and patent game, which consistently benefits from
the current situation, are the Intellectual Property lawyers. ;-)
C. Logical conclusion on the extent to which IPRs shall be protected by a
legal regime :
Intellectual property is different from other forms of property in the sense that it is
created, rather than acquired. This intangible product of a human brains may be as
valuable as his tangible properties like land or goods.
Therefore IPRs are a reward for the creative effort of the creator. And there is
strong justification of giving innovators an incentive.
The main thrust of the entire field of IPR law is that a third party shall not be
permitted to make a harvest for what he has not sowed.
Legal regime protecting IPRs gives the creator a proprietary rights in his original
creations, and an unauthorized use of such creations by other persons is a violation
of creators ownership.
The justification being,
if we want to ensure that innovators continue their activities and research that
lead to growth in the sciences and the arts,
then we have to build a solid barrier against violations of the rights of the
innovators subsisting in their creations.
At the inception of legal regimes to protect IPRs, ie 19th century,
the law was designed to reward the inventor for making his work public and

CONTENTS | Module-1 | Module-2 | Module-3 | Module-4 http://duralex.bhatt.net.in/


https://www.facebook.com/groups/Dharmo.Rakshati.Rakshitah/ Page - 31 of 196

thereby allowing the society to access something that would otherwise have
remained secret.
Protection in the form of intellectual property rights (IPRs) was the price paid
by society to the inventor so that the latter would make his work public.
ie there was greater focus on the individual right of the inventor,
and lesser focus on societal needs.
In todays world, the justification for protecting intellectual property is essentially
utilitarian (need to be useful or practical).
If everybody is free to access new knowledge, inventors have little incentive to
commit resources to producing it.
However, IPR holder also has a duty to be sensitive to meet public needs, at
affordable price.
And in case he fails in this duty, then the legal regime shall provides for
compulsory licensing, or even revocation of patent (or other IPR), in extreme
cases.
Illustration : 2012 : In the case of Natco Pharma Limited v. Bayer Corporation,
the Indian Patents Office granted its first compulsory license, for the
manufacture and sale of Bayers patented cancer drug Nexavar.
In this case Bayers inability to make Nexavar available to nearly 98 per cent
of the Indian public was held by the Controller to amount to a failure to satisfy
the reasonable requirements of the public.
This case set the precedent for making expensive patented drugs available for
compulsory licensing under the Patents Act.
Way To Go ! Creative Commons license (CC) :
Creative Commons license is a fit response to the increasingly high level of
capitalization of intellectual property rights.
CC license is an alternate model to protect intellectual property with less stress
on the commercial component.
CC license is one of several public copyright licenses that enable the free
distribution of an otherwise copyrighted work.
CC license is a collection of eleven possible conditions under which the author
can protect his work,
and in the meantime also provide free reproduction of copyright works for
educational and other socially useful purposes.
Copyleft : Copyleft is a form of copyright license. It is similar to Creative
Commons.
Copyleft can be used for publishing all kinds of copyrighted works such as
texts, music, films, computer software, plays, photographs, etc.
In case of default copyright publishing all rights are reserved in favour of the

CONTENTS | Module-1 | Module-2 | Module-3 | Module-4 http://duralex.bhatt.net.in/


https://www.facebook.com/groups/Dharmo.Rakshati.Rakshitah/ Page - 32 of 196

owner of copyright,
whereas in copyleft licensing only some rights belonging to the owner are
reserved while others are waived off in favour of the users.
The concept of copyleft is derived from copyright and in fact copyleft is just a
play on the word copyright
to describe the practice of using copyright law to remove restrictions on
distributing copies and modified versions of a work for others and requiring
that the same freedoms be preserved in modified versions.
The copyleft concept uses copyright law creatively to ensure that open content
remains open even when it is modified and redistributed by subsequent
developers.

This is Module-1. Menu ---> CONTENTS | Module-1 | Module-2 | Module-3 | Module-4

Discuss : Competing rationales of legal regimes for protection of IP.


ANSWER :
Refer :
http://epgp.inflibnet.ac.in/epgpdata/uploads/epgp_content/law/08._intellectual_pr
operty_law/03._theoretical_justifications_for_intellectual_property/et/5793_et_03_
et.pdf
https://www.icsi.edu/docs/webmodules/Publications/9.4%20Intellectual
%20Property%20Rights.pdf
http://shodhganga.inflibnet.ac.in/bitstream/10603/62276/7/chapter%202.pdf
http://ccs.in/sites/default/files/ipr_india.pdf
Moot question here is :
Which of the IPR philosophy/ theory,
best describes rational (reasoning) for granting and protecting IPRs ?
Outline :
Meaning of Competing rationals of legal regimes for protection of IP.
List of main theoretical and philosophical rationals/ justifications for protection of IP
1. John Stuart Mills utilitarian theory,
2. John Lockes labour theory,
3. George Wilhelm Friedrich Hegels personality theory.
Conclusion.
Meaning of Competing rationals of legal regimes for protection of IP :
What is meant by legal regime?
A legal regime is a system

CONTENTS | Module-1 | Module-2 | Module-3 | Module-4 http://duralex.bhatt.net.in/


https://www.facebook.com/groups/Dharmo.Rakshati.Rakshitah/ Page - 33 of 196

of principles and rules governing something,


and which is created by law.
A legal regime is a framework of legal rules and regulations.
A legal regime is what is established by or founded upon law; lawful.
A legal regime is what is recognized, enforceable, or having a remedy at law
rather than in equity.
What is meant by rational?
rational means,
A person who can draw logical conclusions.
Behavior guided by reasoning and not by emotions .
A reasoning process based on,
logic, systematic methods and pragmatic appraisal of theory, philosophy, etc
in drawing a conclusion.
Thus, Competing rationals for the legal regimes for protection of IP means,
logical and systematic discussion on, competing theoretical & philosophical
justifications for protecting IP.
List of main theoretical and philosophical rationals/ justifications for protection of IP :
1. John Stuart Mills utilitarian theory,
2. John Lockes labour theory,
3. George Wilhelm Friedrich Hegels personality theory.
Note : Proponents of above three theories had not developed the theories specifically
to justify IPR.
rather these theories were used much later by academics to justify IPR .
1. Utilitarian theory :
Theory :
Utilitarian theory is often used to justify IPR.
John Stuart Mill developed a conception of utility, which permits acts if it
produces greatest happiness amongst a vast majority of the population and
ensures absence of pain.
ie If the aggregation of happiness is less than the aggregate of all pain produced
by an act then such an act is not to be pursued.
Thus, Mills utilitarianism is taken as something that maximizes the welfare or
well-being of all the people. An act is desirable or acceptable if it seeks to
maximize the well-being of all and minimize pain.
Application in IPR :
IPR works mainly on the incentive system and on the belief that providing
protection to the innovation or any new creation would encourage people to

CONTENTS | Module-1 | Module-2 | Module-3 | Module-4 http://duralex.bhatt.net.in/


https://www.facebook.com/groups/Dharmo.Rakshati.Rakshitah/ Page - 34 of 196

openly disclose their innovation without any fear that others would appropriate
their creativity.
As per utilitarian theory, awarding property rights to innovators and producers of
creative work is a mere means to an end,
because, ultimately the society benefits as the new creation or innovation is
more openly disclosed.
Patents : Mill agreed that patent monopolies are appropriate compared to
earlier practices of rewards or privileges granted by the monarchy and the
government.
He argued that granting rewards or privileges by government involved
discretion on the part of the authority which may not necessarily recognise
certain inventions.
Patent regime that allowed monopoly to the inventor in exchange for
disclosure of the invention was thought be a good system.
It struck a fine balance between protecting the inventors interest and sharing
or disseminating information about new technologies.
Trademarks : Utilitarian theory is also used to justify the existence of the
modern trademark law.
A utilitarian would argue that protecting marks would reduce search costs,
avoid confusion and ensure quality by weeding out counterfeit goods from the
market. This maximizes the well-being of all people in the society and reduces
harm.
Marks used on the goods serve as a source identifier. It communicates to the
customer about the source of origin of the goods and also to an extent about
the quality of the goods.
If marks are not protected then any person may use it to pass off his goods as
that of others. This would harm the businesses and also confuse the
consumers who associate the mark with certain quality and originating from a
particular business.
Copyrights : Utilitarian justification works well to justify the existence of the
copyright regime as it protects the works of authors, composers, sculptors,
painters, photographers.
A utilitarian would argue that a monopoly for a short duration would encourage
more people to disseminate their work which would enable the public to read,
enjoy and appreciate creative works.
Lack of protection would not give authors enough incentive to be engaged in
creative works.
A copyright regime provides enough incentives to the author as it enables him
to disseminate his work without the fear of someone appropriating his work. It
also enables the author to earn through his creative works.

CONTENTS | Module-1 | Module-2 | Module-3 | Module-4 http://duralex.bhatt.net.in/


https://www.facebook.com/groups/Dharmo.Rakshati.Rakshitah/ Page - 35 of 196

Criticism :
Utilitarian justification of IPR on the other hand is criticised because,
utilitarian theory does not balance private and public interests.
Rather, unmindful of balance of equity equity, it blindly tries to maximize
benefit.
Private intellectual property restricts the use of ideas.
Patent regime based on the incentive system restricts the people other than the
first person to register the idea, from freely using that idea.
Granting property rights to intangible ideas impedes the flow of knowledge and
also prevents or delays downstream innovation.
Though there is a system of disclosure under the patent system it still provides
exclusive ownership to the person who owns the patent, and he can restrict
others from freely using the invention for a certain period of time (twenty
years).
Illustration :
Had the patent system prevailed during the time when wheel was invented it
would have restricted others from freely using the invention, this would have
seriously restricted the development of mankind as many important, whether
small or big improvements, immediately followed after the invention of wheel.
Thus, the IPR as such does not exactly allow free flow of ideas and the
restriction on the diffusion of technology in a way impedes innovations based on
the original patents.
The negative effect it has on the market has to be taken into consideration.
By providing patent protection for twenty years, what it invariably does is it
eliminates competitors in the market and creates a monopoly situation and the
society may not benefit from such inventions.
2. Labour Theory :
Theory :
One of the powerful basis for justifying IPR lies in the belief that
a person deserves the fruits of his labour .
This would mean that an IPR would belong to the person who created it because
it involves his labour and all the benefits derived from it would be the fruits of
his labour.
John Locke in his Second Treatise of Government asserts that all that is in
nature is provided by God and it is available to all men as it is held in common
for the benefit of all.
So no individual can have prior claim over substances that are available in
nature as it is meant for the enjoyment of the entire humanity.

CONTENTS | Module-1 | Module-2 | Module-3 | Module-4 http://duralex.bhatt.net.in/


https://www.facebook.com/groups/Dharmo.Rakshati.Rakshitah/ Page - 36 of 196

Locke asserts that when individual exerts his labour over resources he can claim
it as his property because he has added value through his labour.
Lockes version of this labour justification is based on the assumption that each
individual has prior property rights in ones body.
According to Locke, labour exerted by an individual is his own property as he is
the owner of his body.
Therefore, it is impossible to separate labour and its product .
Locke would readily grant patent right if an individual exerts labour to come up
with an innovative product or machine that is useful.
Illustration :
James Watt used his labour to develop and improve steam engines that could be
used in locomotives. Watts invention added value and spurred the growth of the
economy.
Lockes labour theory can be used to justify patent protection for Watts steam
engine as he had mixed his labour to develop a new technology.
As per Locke, an author, painter, photographer or musician who has exerted his
labour to express his original thoughts in the form of a book, paint, photo or
music album should readily be granted a copyright.
Condition : Lockes justification of property rights in labour is subject to a particular
condition.
According to him, property rights can be allowed only if it does not deny the
others of resources existing in nature. For example,
Locke would have an objection if natural phenomenon or natural resources
(like genes) are monopolized even if an individual has exerted labour to
identify or discover its existence as it would prevent others from using it for a
certain period of time.
If there are enough resources available in nature then property rights can be
claimed over resources when a person exerts labour over it.
An individual who has exerted his labour over the scarce resource cannot claim
property rights in it.
Criticism :
Lockes theory is subject to certain limitations and criticisms.
According to Robert Nozick, fruits of labour is usually valuable and property
rights enables the labourer to appropriate this value.
However, Locke allowed private property right over natural resources, only if
there is enough left for others from the common pool of resources.
For instance, if natural resources are scarce and too many people are dependent
on its availability, then if a person has mixed his labour he cannot claim property
rights in it as it would deprive others of it.

CONTENTS | Module-1 | Module-2 | Module-3 | Module-4 http://duralex.bhatt.net.in/


https://www.facebook.com/groups/Dharmo.Rakshati.Rakshitah/ Page - 37 of 196

Illustration :
Patents over certain cancerous genes like the BRCA1 and BRCA2 would limit
others from using the genes for developing a diagnostic kit.
Locke would certainly disapprove of such patents if there is not enough left for
others.
According to Locke, natural resources do not have any value until an individual
exerts his labour on it.
However, this is not plausible when labour is mixed with naturally existing
substance that does not result in transformation of the substance.
Case of patent Neem Trees use as insecticide :
W.R. Grace claimed a patent for Neem extract. All that Grace did was to
identify exactly which chemical component in the Neem plant had the
insecticide properties (azadirachtin) and extracted that component and claimed
patent over it.
Though there is labour involved on the part of Grace , i.e in identifying exactly
which chemical component had the insecticide properties, it can be said that
chemical component with insecticide property already existed in nature and
its properties would have remained the same regardless of whether Grace
identified its properties.
Here the labour is mixed with what already existed in nature.
Lockes labour theory would grant patent over Neem to Grace.
Even while, Grace had obtained information about the uses of Neem tree
from the Indian farming community.
According to labour theory, even when multiple people might have contributed
to the evolution of a technology, only the value added by the most recent
contributor is to be rewarded.
eg, Prior to Watts invention of steam engine, there were many others who
contributed in the evolution of steam engine but unfortunately it was only Watt
who was able to get the full market value for improvement on the steam
engine.
3. Personality theory :
Theory :
According to this theory, any work or an invention would belong to its author or
inventor because it is the manifestation of the creator or inventors personality.
According to George Wilhelm Friedrich Hegel, personality is the reflection of an
individuals will.
He identifies the personality as the wills struggle to actualize itself .
Hegel gives prominence to the will compared to external property, which is a
manifestation of the will.

CONTENTS | Module-1 | Module-2 | Module-3 | Module-4 http://duralex.bhatt.net.in/


https://www.facebook.com/groups/Dharmo.Rakshati.Rakshitah/ Page - 38 of 196

Thus, when an individual expresses himself through his work it is nothing but an
external manifestation of his personality.
Labour is the means by which the will occupies the object. Hegel believes that
this external manifestation which is seen as property cannot be alienated at any
point in time as it is a reflection of the self.
Application in IPR :
Hegels personality theory can be used to justify claims by authors, musician,
artists, sculptors, photographers etc. in their work.
An authors personality is manifested through his or her work. Books written by
authors are external objects through which their inner personality (i.e their
feelings, emotions, experiences and imaginations) is manifested.
ie, Harry Potter series is an external manifestation of J.K. Rowlings personality.
The book itself is an expression of the will and part of the personality and as
such justifies right to property.
Similarly, an innovative technology is a manifestation of the inventors will and
as such merits a property right.
Criticism :
The problem in applying personality justification to intellectual property would be
in identifying whether there is a personality stake in the particular object which
is the external manifestation of the will.
Further, Hegels proposition that works are external manifestation of a
personality also poses problem within the realm of IPR.
According to Hegels Personality Theory :
The writer or painter, in copying or imitating, has used his skills and abilities
and expressed his will and as such would have a property right in it.
This would be counter-productive to an IPR regime as it does not recognise the
imitator as a property right holder.
Conclusion :
As highlighted above,
ALL these 3 theories give different justifications for granting IPRs,
and ALL these 3 theories have certain inherent weaknesses.
The utilitarian theory justifies IPR on the basis that it maximizes the benefit to the
society.
Providing patent and copyright protection is seen as an incentive that facilitates
disclosure by inventors, authors and artists.
Without such protection inventors and artists would always be under the fear
that others in the society would engage in unauthorized copying of their work.
Lockes labour theory is an extension of the natural rights theory which can be

CONTENTS | Module-1 | Module-2 | Module-3 | Module-4 http://duralex.bhatt.net.in/


https://www.facebook.com/groups/Dharmo.Rakshati.Rakshitah/ Page - 39 of 196

used to justify property rights in intangible objects on the ground that a person has
exerted his labour in creating that intangible object.
Hegels personality theory justifies property protection for intangible ideas and
expressions on the basis that it is an extension of the creator or innovators
personality.

This is Module-1. Menu ---> CONTENTS | Module-1 | Module-2 | Module-3 | Module-4

Discuss : The Berne Convention.


ANSWER :
Refer :
http://www.wipo.int/treaties/en/ip/berne/summary_berne.html
https://en.wikipedia.org/wiki/Berne_Convention
Outline :
History.
Three basic principles
Principle of "national treatment"
Principle of "automatic" protection
Principle of "independence" of protection
Minimum standards of protection
Works to be protected
Rights to be protected
Duration of protection
Limitations and exceptions on economic rights
Salient feature of Berne Convention.
Intro :
1886 : The Berne Convention for the Protection of Literary and Artistic Works,
usually known as the Berne Convention,
is an international agreement governing copyright, which was first accepted in
Berne, Switzerland, in 1886.
The Convention is open to all States.
The Berne Convention formally mandated several aspects of modern copyright law;
it introduced the concept that a copyright exists the moment a work is "fixed",
rather than requiring registration.
it also enforces a requirement that countries recognize copyrights held by the
citizens of all other parties to the convention.

CONTENTS | Module-1 | Module-2 | Module-3 | Module-4 http://duralex.bhatt.net.in/


https://www.facebook.com/groups/Dharmo.Rakshati.Rakshitah/ Page - 40 of 196

The Berne Convention deals with the protection of works and the rights of their
authors.
It is based on three basic principles
and contains a series of provisions determining
the minimum protection to be granted,
as well as special provisions available to developing countries that want to
make use of them.
History :
The Berne Convention was developed at the instigation of Victor Hugo of the
Association Littraire et Artistique Internationale.
It was influenced by the French "right of the author" (droit d'auteur),
which contrasts with the Anglo-Saxon concept of "copyright" which only dealt
with economic concerns.
1886 : The first version of the Berne Convention treaty was signed by Belgium,
France, Germany, Haiti, Italy, Liberia, Spain, Switzerland, Tunisia, and the United
Kingdom.
1896 : Revised at Paris,
1908 : Revised at Berlin,
1914 : Completed at Berne,
1928 : Revised at Rome,
1948 : Revised at Brussels,
1967 : Revised at Stockholm, and
1971 : Revised at Paris.
Three basic principles :
(a) Principle of "national treatment" :
Works originating in one of the Contracting States
must be given the same protection in each of the other Contracting States as
the latter grants to the works of its own nationals.
Note : Here, Works originating in one of the Contracting States means,
works the author of which is a national of such a State
or works first published in such a State.
(b) Principle of "automatic" protection :
Protection must not be conditional upon compliance with any formality .
(c) Principle of "independence" of protection :
Protection is independent of the existence of protection in the country of origin
of the work.
If, however, a Contracting State provides for a longer term of protection than

CONTENTS | Module-1 | Module-2 | Module-3 | Module-4 http://duralex.bhatt.net.in/


https://www.facebook.com/groups/Dharmo.Rakshati.Rakshitah/ Page - 41 of 196

the minimum prescribed by the Convention and the work ceases to be protected
in the country of origin,
then, protection may be denied once protection in the country of origin ceases.
Minimum standards of protection :
The minimum standards of protection relate to
the works and rights to be protected,
and the duration of protection:
(a) Works to be protected :
Protection must include
every production in the literary, scientific and artistic domain,
whatever the mode or form of its expression" (Article 2(1) of the Convention).
(b) Rights to be protected :
Subject to certain allowed reservations, limitations or exceptions, the following
are among the rights that must be recognized as exclusive rights of
authorization :
the right to translate,
the right to make adaptations and arrangements of the work,
the right to perform in public dramatic, and/or musical works,
the right to recite literary works in public,
the right to communicate to the public the performance of such works,
the right to broadcast,
the right to make reproductions in any manner or form,
the right to use the work as a basis for an audiovisual work, and the right to
reproduce, distribute, perform in public or communicate to the public that
audiovisual work.
The Convention also provides for "moral rights", that is,
the right to claim authorship of the work
and the right to object to any mutilation, deformation or other derogatory
action in relation to the work,
that would be prejudicial to the author's honor or reputation .
(c) Duration of protection :
As to the duration of protection, the general rule is that
protection must be granted until the expiration of the 50 th year after the
author's death.
There are, however, exceptions to this general rule.
Exception-1. In the case of anonymous or pseudonymous works,

CONTENTS | Module-1 | Module-2 | Module-3 | Module-4 http://duralex.bhatt.net.in/


https://www.facebook.com/groups/Dharmo.Rakshati.Rakshitah/ Page - 42 of 196

the term of protection expires 50 years after the work has been lawfully
made available to the public,
except if the pseudonym leaves no doubt as to the author's identity
Note : In case the author discloses his or her identity during 50 years of
anonymous work
then the general rule applies, ie protection must be granted until the
expiration of the 50th year after the author's death.
Exception-2. In the case of audiovisual (cinematographic) works,
the minimum term of protection is 50 years
after the making available of the work to the public ("release")
or (in case of un-released work) from the creation of the work .
Exception-3. In the case of works of applied art and photographic works,
the minimum term is 25 years from the creation of the work .
Duration of related rights :
Related rights enjoy shorter terms, normally 50 years after the performance,
recording or broadcast has taken place.
Limitations and exceptions on economic rights :
The Berne Convention allows certain limitations and exceptions on economic rights,
that is, cases in which protected works may be used
without the authorization of the owner of the copyright,
and without payment of compensation.
These limitations are commonly referred to as "free uses" of protected works,
and are set forth in
Article 9(2) - reproduction in certain special cases,
Article 10 - quotations & use of works by way of illustration for teaching purpose,
Article 10bis - reproduction of newspaper or similar articles and use of works for
the purpose of reporting current events, and
Article 11bis(3) - temporary recordings for broadcasting purposes.
Salient feature of Berne Convention :
Developing countries are permitted,
to implement non-voluntary licenses for translation and reproduction of works in
connection with educational activities.
without the authorization of the right holder,
subject to the payment of remuneration to be fixed by the law .
Under the TRIPS Agreement, the three basic principles of Berne Convention,
also bind even those World Trade Organization (WTO) Members not party to the
Berne Convention.

CONTENTS | Module-1 | Module-2 | Module-3 | Module-4 http://duralex.bhatt.net.in/


https://www.facebook.com/groups/Dharmo.Rakshati.Rakshitah/ Page - 43 of 196

The TRIPS Agreement imposes an obligation of "most-favored-nation treatment",


under which advantages accorded by a WTO Member to the nationals of any
other country,
must also be accorded to the nationals of all WTO Members.

This is Module-1. Menu ---> CONTENTS | Module-1 | Module-2 | Module-3 | Module-4

Discuss : Universal Copyright Convention.


ANSWER :
Refer :
https://en.wikipedia.org/wiki/Universal_Copyright_Convention
Intro :
1952 Universal Copyright Convention (UCC), adopted in Geneva, Switzerland,,
is one of the two principal international conventions protecting copyright;
the other is the Berne Convention.
The Berne Convention states also became party to the UCC,
so that their copyrights would exist in non-Berne convention states as well .
WHY another convention?
The UCC was developed by UNESCO as an alternative to the Berne Convention for
those states which disagreed with aspects of the Berne Convention,
but still wished to participate in some form of multilateral copyright protection .
These states included
developing countries
as well as the United States and most of Latin America .
WHY developing countries were out of Berne Convention?
Developing countries thought that,
strong copyright protections granted by the Berne Convention overly benefited
Western, developed, copyright-exporting nations,
WHY USA & Latin Americans were out of Berne Convention?
USA & many of the Latin American countries were already members of the
Buenos Aires Convention, a Pan-American copyright convention that was weaker
than the Berne Convention.
The US laws,
only provided copyright protection for a fixed, renewable term,
and required that in order for a work to be copyrighted it must contain a
copyright notice and be registered at the Copyright Office.

CONTENTS | Module-1 | Module-2 | Module-3 | Module-4 http://duralex.bhatt.net.in/


https://www.facebook.com/groups/Dharmo.Rakshati.Rakshitah/ Page - 44 of 196

The Berne Convention, on the other hand,


provided for copyright protection for a single term based on the life of the
author,
and did not require registration or the inclusion of a copyright notice for
copyright to exist.
Thus the United States would have to make several major modifications to its
copyright law in order to become a party to Berne Convention.
At the time the United States was unwilling to do so.
The UCC permits those states which had a system of protection similar to the United
States for fixed terms at the time of signature to retain them.
Eventually, the United States became willing to participate in the Berne convention,
and in 1989, changed its national copyright law as required.
Under the Second Protocol of the UCC, protection under U.S. copyright law is
expressly required for works published
by the United Nations,
by UN specialized agencies
and by the Organization of American States (OAS).
The same requirement applies to other contracting states as well.
Anxiety of Berne Convention states :
Berne Convention states were concerned that,
the existence of the UCC would encourage parties to leave the Berne Convention
and adopt the UCC instead.
So the UCC included a clause,
stating that parties which were also Berne Convention parties
need not apply the provisions of the ??? Convention to any former Berne
Convention state which renounced the Berne Convention after 1951.
ie any state which adopts the Berne Convention is penalised,
if it then decides to renounce it and use the UCC protections instead,
since its copyrights might no longer exist in Berne Convention states.
Conclusion :
Since almost all countries are either members or aspiring members of the WTO,
and are thus conforming to the Agreement on Trade-Related Aspects of
Intellectual Property Rights Agreement (TRIPS),
the UCC has lost significance.

This is Module-1. Menu ---> CONTENTS | Module-1 | Module-2 | Module-3 | Module-4

CONTENTS | Module-1 | Module-2 | Module-3 | Module-4 http://duralex.bhatt.net.in/


https://www.facebook.com/groups/Dharmo.Rakshati.Rakshitah/ Page - 45 of 196

Discuss : World Intellectual Property Rights Organization (WIPO).


ANSWER :
Refer :
http://www.wipo.int/edocs/pubdocs/en/intproperty/450/wipo_pub_450.pdf
https://en.wikipedia.org/wiki/World_Intellectual_Property_Organization
What is the World Intellectual Property Organization (WIPO) ?
WIPO is, a global forum for intellectual property services, policy, information and
cooperation.
WIPO is one of the 17 specialized agencies of the United Nations.
WIPO is a self-funding agency of the United Nations,
WIPO has 191 member states and its headquarters is at Geneva, Switzerland .
Established in 1970, WIPO is an international organization dedicated to ensure
that rights of creators & owners of intellectual property are protected worldwide,
and that inventors and authors are recognized and rewarded for their ingenuity .
Mission : WIPOs mission is,
to lead the development of a balanced and effective international intellectual
property (IP) system that enables innovation and creativity for the benefit of all.
This international protection acts as a spur to human creativity, pushing back the
limits of science and technology and enriching the world of literature and the arts.
By providing a stable environment for marketing products protected by intellectual
property, it also oils the wheels of international trade.
WIPO works closely with its Member States and other constituents to ensure that,
the intellectual property system remains a flexible tool for prosperity and well-
being of member states, and
help realize full potential of created works for present and future generations.
History of WIPO :
1893 : The predecessor to WIPO was the United International Bureaux for the
Protection of Intellectual Property,
which had been established in 1893,
to administer the Berne Convention for the Protection of Literary and Artistic
Works,
and the Paris Convention for the Protection of Industrial Property .
The bureaus mandate was limited to promote the protection of intellectual
property.
1970 : WIPO was formally created by the Convention Establishing the World
Intellectual Property Organization, which entered into force on April 26, 1970.
Under Article 3 of this Convention, WIPO seeks to

CONTENTS | Module-1 | Module-2 | Module-3 | Module-4 http://duralex.bhatt.net.in/


https://www.facebook.com/groups/Dharmo.Rakshati.Rakshitah/ Page - 46 of 196

"promote the protection of intellectual property throughout the world" .


WIPO became a specialized agency of the UN in 1974.
Revised mandate : Article-1 of the agreement between UN & WIPO :
WIPO is responsible,
for promoting creative intellectual activity
and for facilitating the transfer of technology related to industrial property to
the developing countries
in order to accelerate economic, social and cultural development,
subject to the competence and responsibilities of the United Nations and its
organs.
How does WIPO promote the protection of intellectual property?
As part of the United Nations system of specialized agencies, WIPO serves,
as a forum for its Member States to establish and harmonize rules and practices
for the protection of intellectual property rights.
WIPO also services global registration systems for trademarks, industrial designs
and appellations of origin, and a global filing system for patents.
These systems are under regular review by WIPOs Member States and other
stakeholders to determine how they can be improved to better serve the needs of
users and potential users.
Many industrialized nations have intellectual property protection systems that are
centuries old.
Among under-developed and developing countries, many are in the process of
building up their patent, trademark and copyright legal frameworks and intellectual
property systems.
WIPO plays a key role in helping these countries,
to evolve through treaty negotiation;
legal and technical assistance;
and training in various forms, including in the area of enforcement .
WIPO works with its Member States to make available information on intellectual
property and outreach tools for a range of audiences
from the grassroots level through to the business sector and policymakers to
ensure its benefits are well recognized, properly understood and accessible to
all.
How is WIPO funded?
WIPO is a largely self-financed organization, generating more than 90 percent
of its annual budget through
its widely used international registration and filing systems,
as well as through its publications and arbitration and mediation services .

CONTENTS | Module-1 | Module-2 | Module-3 | Module-4 http://duralex.bhatt.net.in/


https://www.facebook.com/groups/Dharmo.Rakshati.Rakshitah/ Page - 47 of 196

The remaining funds come from contributions by Member States.


WIPOnet :
WIPO has established WIPOnet, a global information network.
The project seeks to link over 300 intellectual property offices (IP offices) in all
WIPO Member States.
In addition to providing a means of secure communication among all connected
parties, WIPOnet is the foundation for WIPO's intellectual property services.

This is Module-1. Menu ---> CONTENTS | Module-1 | Module-2 | Module-3 | Module-4

Discuss : TRIPS and the UNESCO.


ANSWER :
Refer :
https://en.wikipedia.org/wiki/TRIPS_Agreement
http://lawtimesjournal.in/trips-respect-copyright/
https://www.wto.org/english/tratop_e/trips_e/intel2_e.htm
http://www.indianmba.com/Faculty_Column/FC535/fc535.html
TRIPS Agreement :
Intro :
The Agreement on Trade-Related Aspects of Intellectual Property Rights
(TRIPS) is an international legal agreement
between all the member nations of the World Trade Organization (WTO).
TRIPS sets down minimum standards for the regulation by national governments
of many forms of intellectual property (IP)
as applied to nationals of other WTO member nations.
The TRIPS agreement introduced intellectual property law into the international
trading system for the first time and remains the most comprehensive
international agreement on intellectual property to date.
TRIPS specifies (i) standards, (ii) enforcement procedures & remedies, and (iii)
dispute resolution procedures.
History :
TRIPS was negotiated at the end of the Uruguay Round of the General
Agreement on Tariffs and Trade (GATT) in 1994
and is administered by the WTO.
TRIPS came into effect on 1 January 1995.
TRIPS Agreement was the culmination of a program of intense lobbying by the
United States, supported by the European Union, Japan and other developed

CONTENTS | Module-1 | Module-2 | Module-3 | Module-4 http://duralex.bhatt.net.in/


https://www.facebook.com/groups/Dharmo.Rakshati.Rakshitah/ Page - 48 of 196

nations.
Economic coercion :
Developed nations strategy of linking trade policy to intellectual property
standards)
played an important role in defeating the competing policy positions of
developing countries like Brazil, Thailand, India and Caribbean Basin states.
The GATT became the basis for the establishment of the World Trade
Organization.
Since, ratification of TRIPS is a compulsory requirement of World Trade
Organization membership,
any country seeking to obtain hard access to the numerous international
markets opened by the World Trade Organization must enact the strict
intellectual property laws mandated by TRIPS.
For this reason, TRIPS is the most important multilateral instrument for the
globalization of intellectual property laws.
Even, states like Russia and China that were very unlikely to join the Berne
Convention have found the prospect of WTO membership a powerful enticement.
Doha Declaration :
In 2001, developing countries, concerned that developed countries were insisting
on an overly narrow reading of TRIPS, initiated a round of talks that resulted in
the Doha Declaration.
The Doha declaration is a WTO statement that clarifies the scope of TRIPS,
stating for example that TRIPS can and should be interpreted in light of the
goal "to promote access to medicines for all".
Scope :
TRIPS requires WTO members to provide legal regime to protect following IPRs :
copyright rights, covering content producers including performers, producers
of sound recordings and broadcasting organizations;
geographical indications, including appellations of origin;
industrial designs;
integrated circuit layout-designs;
patents;
new plant varieties;
trademarks;
etc.
TRIPS requires that, the legal regime protecting IPRs,
shall meet the objectives to contribute to the promotion of technological
innovation

CONTENTS | Module-1 | Module-2 | Module-3 | Module-4 http://duralex.bhatt.net.in/


https://www.facebook.com/groups/Dharmo.Rakshati.Rakshitah/ Page - 49 of 196

and to the transfer and dissemination of technology,


to the mutual advantage of producers and users of technological knowledge
and in a manner conducive to social and economic welfare,
and to a balance of rights and obligations.
Features :
TRIPS Agreement is sometimes referred to as a Berne and Paris-plus agreement .
The three main features of the Agreement are :
1. Standards.
2. Enforcement procedures & remedies.
3. Dispute settlement.
1. Standards :
The Agreement sets out the minimum standards of protection to be provided
by each Member
in respect of each of the main areas of intellectual property covered by the
TRIPS Agreement,
Standards define each of the main elements of protection :
the subject-matter to be protected,
the rights to be conferred and permissible exceptions to those rights,
and the minimum duration of protection.
The Agreement also requires that the substantive obligations of
the main conventions of the WIPO, ie
the Paris Convention for the Protection of Industrial Property (Paris
Convention), and
the Berne Convention for the Protection of Literary and Artistic Works.
must be complied with in their most recent versions.
Moreover, the TRIPS Agreement also adds a substantial number of additional
obligations on matters
where the pre-existing conventions are silent or were seen as being
inadequate.
2. Enforcement procedures & remedies :
Unlike other agreements on intellectual property, TRIPS has a powerful
enforcement mechanism.
States can be disciplined through the WTO's dispute settlement mechanism.
TRIPS Agreement provides for domestic procedures and remedies for the
enforcement of intellectual property rights by States. These include,
civil and administrative procedures and remedies that must be available so
that right holders can effectively enforce their rights.

CONTENTS | Module-1 | Module-2 | Module-3 | Module-4 http://duralex.bhatt.net.in/


https://www.facebook.com/groups/Dharmo.Rakshati.Rakshitah/ Page - 50 of 196

3. Dispute settlement :
The Agreement makes disputes between WTO Members about the respect of
the TRIPS obligations subject to the WTOs dispute settlement procedures.
Other features :
TRIPS provides for most-favoured-nation treatment and some general rules,
to ensure that procedural difficulties in acquiring or maintaining IPRs do not
nullify the substantive benefits that should flow from the Agreement.
The obligations under the Agreement apply equally to all Member countries,
but developing countries will have a longer period to phase them in.
Special transition arrangements operate in the situation where a developing
country does not presently provide product patent protection in the area of
pharmaceuticals.
Implementation in developing countries :
There were strong apprehensions that the TRIPS standard of requiring all countries
to create strict IPR regime was detrimental to the development of poorer countries.
hence, developing countries were allowed extra time to implement the applicable
changes to their national laws, in two tiers of transition according to their level
of development.
Lack of legal and technical expertise needed to draft legislation that implements
flexibilities,
has often led to developing nations copying IP legislation of developed nations
or relying on technical assistance from WIPO .
This, according to critics, encourages developing nations to implement stronger
intellectual property monopolies, to the advantage of developed nations.
Studies, Banerjee and Nayak, shows that TRIPS has a positive effect on R&D
expenditure of Indian pharmaceutical firms.
Criticism :
Since TRIPS came into force, it has been subject to criticism from developing
countries, academics, and non-governmental organizations.
Though some of this criticism is against the WTO generally, many advocates of
trade liberalisation also regard TRIPS as poor policy.
TRIPS results in wealth concentration, by way of moving money from people in
developing countries to copyright and patent owners in developed countries)
Coercive device of imposition of artificial scarcity on the developing nations forced
them to go for stronger IPR regime against their national welfare.
Lengthy patent periods under TRIPs unduly slows the entry of generic substitutes
and competition to the market.
The importance of TRIPS in the process of generation and diffusion of knowledge

CONTENTS | Module-1 | Module-2 | Module-3 | Module-4 http://duralex.bhatt.net.in/


https://www.facebook.com/groups/Dharmo.Rakshati.Rakshitah/ Page - 51 of 196

and innovation has been overestimated by its supporters.


TRIPS failed to accelerate investment and technology flows to low-income nations,
which infact was a likely benefit advanced by WTO members in the lead-up to
the agreement's formation.
The 2002 Doha Declaration affirmed that the TRIPS agreement should not prevent
members from taking measures necessary to protect public health.
However, despite this recognition, less-developed countries have argued that
TRIPS's flexible provisions, such as compulsory licensing, are near-on impossible
to exercise due to infant domestic manufacturing and technology in less
developed countries.
Conclusion :
It is generally observed that a strong patent regime has often been found
detrimental to the process of industrial development in particular and scientific
advancement in general.
TRIPS may prove to be a breeding ground for cost inefficient process technologies.
It would be better that the Indian government,
instead of fulfilling its obligation in haste,
take advantage of the concessions given in the TRIPs agreement.
and make such amendments in the act which adequately safeguard and protect
the interests of the domestic industries and market.
UNESCO :
<Work on this. https://en.wikipedia.org/wiki/UNESCO >
Copyright has even greater potential to encourage creativity in the beginning of the
21st century.
Committed to promoting copyright protection since its early days, UNESCO has over
time grown concerned with ensuring general respect for copyright in all fields of
creation and cultural industries.
UNESCO conducts awareness-raising and capacity-building projects, in addition to
information, training and research in the field of copyright law.
It is particularly involved in developing new initiatives to fight against piracy.
UNESCO endeavours to make a contribution to the international debate on this issue,
taking into account the development perspective and paying particular attention to
the need of maintaining the fair balance between the interests of authors and the
interest of the general public of access to knowledge and information.

This is Module-1. Menu ---> CONTENTS | Module-1 | Module-2 | Module-3 | Module-4

Discuss : (i) International legal regime protecting IPRs, (ii) Indian legal regime

CONTENTS | Module-1 | Module-2 | Module-3 | Module-4 http://duralex.bhatt.net.in/


https://www.facebook.com/groups/Dharmo.Rakshati.Rakshitah/ Page - 52 of 196

protecting IPRs.
Discuss the development of Intellectual Property Law in India in the
international regime. (Dec-2015)
Explain the status and position of IPRs in India in context with the International
Regime. (Apr-2016)
ANSWER :
Refer :
http://lawtimesjournal.in/analysis-national-ipr-policy/
http://shodhganga.inflibnet.ac.in/bitstream/10603/62276/7/chapter%202.pdf
Outline :
(i) International legal regime protecting IPRs
TRIPS
The territoriality of IPRs
Theories governing enforcement of international treaties : Monism vs Dualism
Core obligations imposed by international IP agreements
Enforcement provisions
(ii) Indian legal regime protecting IPRs :
Indias International Trade Obligations
Statutory Framework governing Intellectual Property regime in India
Specific steps taken by India in compliance with international treaties :
Administrative Mechanism of the protection
IPR Enforcement Mechanism in India.
(i) International legal regime protecting IPRs :
TRIPS : <detailed discussion elsewhere in this doc>
Before Berne Convention (ie 19th century), domestic intellectual property laws
provided no legal protection to IP products created in other nations,
thereby permitting those products to be exploited by free riders operating
outside the state in which the products were created.
The unfairness of this result prompted governments in the late nineteenth
century to consider an international approach to protect IPRs.
The importance of intellectual property was recognized in
1883 Paris Convention for the Protection of Industrial Property,
and 1886 the Berne Convention for the Protection of Literary and Artistic
Works.
Both these treaties are administered by the World Intellectual Property
Organization (WIPO).

CONTENTS | Module-1 | Module-2 | Module-3 | Module-4 http://duralex.bhatt.net.in/


https://www.facebook.com/groups/Dharmo.Rakshati.Rakshitah/ Page - 53 of 196

1994 : International efforts in protecting IPRs, culminated in an international


treaty TRade-related aspects of Intellectual Property rights (TRIPS),
between all the member nations of the World Trade Organization (WTO).
TRIPS Agreement is sometimes referred to as a Berne and Paris-plus agreement .
The Trade-Related Aspects of Intellectual Property Rights (TRIPS) Agreement
is generally considered as the cornerstone of global intellectual Property Laws
as it lays down the minimum standards for all World Trade Organization (WTO)
member countries.
TRIPS is often talked about as a package deal whereby developing countries
agreed to the TRIPS standards in exchange for the other carrots associated with
WTO membership, which definitely included increased access to foreign markets.
The territoriality of IPRs :
Because of the limited scope of international IPR agreements,
no international IPRs are available to inventors and creators who seek to
market their products across borders (with the limited exception of the
European Union).
IPRs are territorial in nature and are acquired and enforced on a country-by-
country basis under territorially-circumscribed national IPR laws.
The "territorial" nature of intellectual property refers to the fact that
countries enact their own intellectual property laws, typically by statute, and
these intellectual property laws have no application or force outside the
country in which they are enacted.
The inventor must comply with all of the requirements that each country
imposes for granting patent rights to the new variety.
Recent international agreements have achieved some modest forms of
procedural harmonization,
but they have not altered the fundamental premise that
national laws rather than international treaties are the immediate source of
nearly all private rights in intellectual property products.
Two basic principles can be deduced from territorial nature of IPRs are
First, where national laws differ as to the scope or content of the protection
they provide to intellectual property products, the rights enjoyed by the
owners of those products will vary in different national jurisdictions.
Second, territoriality implies that each nation has the right to decide on the
form of IPR protection to be granted within its own borders, provided that it
complies with the obligations contained in international IPR agreements to
which it is a party.
Theories governing enforcement of international treaties : Monism vs Dualism :

CONTENTS | Module-1 | Module-2 | Module-3 | Module-4 http://duralex.bhatt.net.in/


https://www.facebook.com/groups/Dharmo.Rakshati.Rakshitah/ Page - 54 of 196

Implementation of treaty-based obligations in national IPR laws can occur in one


of two ways.
Monism : Automatic incorporation :
In some nations (often referred to as "automatic incorporation" states),
treaties become binding as a part of domestic law as soon as formal
ratification procedures have been adopted.
In these nations, treaties are considered to be "self-executing" or capable of
being given "direct effect" in domestic law such that courts
and administrative agencies can construe the treaty directly and enforce
the rights it grants to the owners of intellectual property products.
Dualism : Legislative incorporation :
In other nations, however (often referred to as "legislative incorporation"
states), treaties are considered to be "non-self-executing"
and can only become binding in domestic law once the parliament or
legislature has adopted legislation to implement the treaty.
In these nations, owners of intellectual property products rely on this
domestic legislation rather than on the treaties themselves when they seek
to enforce rights granted to them under the treaties.
India follows the dualist (legislative inc) theory for the implementation of
international law at domestic level.
Accordingly, international treaties do not automatically become part of national
law. They have to be transformed into domestic law by a legislative act.
Core obligations imposed by international IP agreements :
The territorial approach to IPR protection appears at first to present myriad
difficulties for creators and owners of intellectual property products.
However, in reality, the content of each nations IPR laws are often quite similar
since they have been shaped by international IPR agreements.
These obligations have been substantially successful in eliminating differences in
national IP systems.
National treatment :
One of the cornerstones of international IPR agreements is the national
treatment principle.
National treatment bars discrimination against foreign IPR owners by requiring
that each state provide the same IPRs to private parties from other member
states as are provided to the states own nationals.
National treatment levels the playing field among treaty parties and prevents a
state from giving its own creators and inventors unfair advantages over foreign
creators and inventors.
In the absence of national treatment, for example, domestic firms could freely

CONTENTS | Module-1 | Module-2 | Module-3 | Module-4 http://duralex.bhatt.net.in/


https://www.facebook.com/groups/Dharmo.Rakshati.Rakshitah/ Page - 55 of 196

exploit intellectual property products created in other member states while


simultaneously enjoying legal protection within their own domestic markets.
Reciprocity :
The provisions of several intellectual property treaties contain a limited
exception to national treatment known as reciprocity.
Where a treaty permits reciprocity between two states, say A & B,
State A may condition the grant of legal protection to intellectual property
products from State B, upon State Bs granting of legal protection to
intellectual property products from State A.
Reciprocity is often applied to new IPRs as means of encouraging other nations
to recognize the new rights and extend their protection to foreign nationals.
Once a large number of states have recognized the new IPR, they may revise
the treaty to eliminate the reciprocity option and impose a national treatment
obligation.
Enforcement provisions :
The grant of IPRs in national laws would be meaningless without adequate and
effective mechanisms to enforce those rights.
International treaties specify the types of enforcement provisions that member
states must adopt in their national laws.
These provisions include the imposition of civil and criminal penalties against any
person whose acts infringe upon IPRs of other person.
The penalties include civil judicial proceedings for monetary damages or an
injunction to prevent the continued unauthorized use of the product and criminal
proceedings commenced by the government itself.
(ii) Indian legal regime protecting IPRs :
In the era of expanding multilateral trade and commerce,
it has become inevitable for any country to protect its intellectual property by
providing statutory rights to the creators and inventors
and thus help them fetch adequate commercial value for their efforts in the
world market.
Indias International Trade Obligations :
The Trade-Related Aspects of Intellectual Property Rights (TRIPS) Agreement
is generally considered as the cornerstone of global intellectual Property Laws
as it lays down the minimum standards for all World Trade Organization (WTO)
member countries.
TRIPS is often talked about as a package deal whereby developing countries
agreed to the TRIPS standards in exchange for the other carrots associated with
WTO membership, which definitely included increased access to foreign markets.

CONTENTS | Module-1 | Module-2 | Module-3 | Module-4 http://duralex.bhatt.net.in/


https://www.facebook.com/groups/Dharmo.Rakshati.Rakshitah/ Page - 56 of 196

India as a founder member of WTO has ratified TRIPS, and is in the process of
adjusting itself with the new international trade regime.
This acclimatization process included a lot of changes in its laws and notably
Intellectual Property Laws.
As per the agreement, all member countries including, India are to abide by the
mutually negotiated norms and standards within the stipulated time frame.
Accordingly, India has set up an Intellectual Property Right (IPR) regime, which
is WTO compatible and is well established at all levels whether statutory,
administrative or judicial.
Statutory Framework governing Intellectual Property regime in India :
India has complied with TRIPS obligations by enacting the necessary statutes
and amending the existing ones.
The general laws in relation to Intellectual Property Enforcement in India are
mainly the following :-
i) The Code of Civil Procedure - provides for the civil remedies and
enforcement through civil courts,
ii) The Indian Penal Code - provides for penal remedies.
iii) The Civil and Criminal Rules of Practice - rules of practice of the trial courts,
High Courts and the Supreme Court of India set the enforcement procedure.
Judicial precedents :
India follows common law tradition and judicial precedents do have binding
force.
Decisions of the SC and HCs, bind their respective lower judiciary.
The Intellectual Property Laws :
The most important of the Indian IPR Laws are :
Indian Copyright Act, 1957;
Trade Marks Act, 1999;
Designs Act, 2001;
Patents Act, 1970
Geographical Indications of Goods (Registration and Protection) Act 1999;
Protection of Plant varieties and Farmer's Rights Act, 2001.
Semiconductor Integrated Circuit Layout Design Act, 2000;
Above legislations are supported by following rules framed under respective
their respective statutes :-
i) The Patents Rules, 1972 as amended by the Patents (Amendment) Act,
1999
ii) The Trade Rules, 2001.

CONTENTS | Module-1 | Module-2 | Module-3 | Module-4 http://duralex.bhatt.net.in/


https://www.facebook.com/groups/Dharmo.Rakshati.Rakshitah/ Page - 57 of 196

iii) The Copyright Rules, 1958; and


iv) The Designs Rules, 2001.
Specific steps taken by India in compliance with international treaties :
Apart from above legislative act, there are well-established statutory,
administrative, and judicial frameworks for safeguarding IPRs in India.
Nomenclature IPR in India was imported from the west.
In the Ministry of Commerce and Industry,
the office of the 'Controller General of Patents, Designs and Trade Marks
(CGPDTM)'
has been set up under the Department of Industrial Policy and Promotion.
Well-known international trademarks have been afforded protection in India in
the past by the Indian courts,
despite the fact that these trade marks were not registered in India.
Computer databases and software programs have been protected under the
copyright laws in India and pursuant to this;
software companies have successfully curtailed piracy through judicial
intervention.
Although trade secrets and know-how are not protected by any specific statutory
law in India, they are protected under the common law.
The courts, under the doctrine of breach of confidentiality, have granted
protection to trade secrets.
Administrative Mechanism of the protection :
The Government has taken a comprehensive set of initiatives to streamline the
intellectual property administration in the country in view of its strategic
significance.
Matters relating to patents, designs, trademarks and geographical indications are
administered and supervised in the following way :-
i) The Patent Office (including Designs Wing)
ii) The Patent Information System (PIS)
iii) The Trade Marks Registry (TMR), and
iv) The Geographical Indications Registry (GIR)
v) The 'Copyright Office' has been set up in the Department of Education of the
Ministry of Human Resource Development, to provide all facilities including
registration of copyrights.
(vi) For, layout design of integrated circuits, 'Department of IT' in the Ministry
of Information Technology is the nodal organization.
(vii) For, Protection of Plant Varieties and Farmers' Rights Authority in
Ministry of Agriculture administers all measures and policies.

CONTENTS | Module-1 | Module-2 | Module-3 | Module-4 http://duralex.bhatt.net.in/


https://www.facebook.com/groups/Dharmo.Rakshati.Rakshitah/ Page - 58 of 196

IPR Enforcement Mechanism in India :


Following is the hierarchy of courts in cases of IPR disputes.
District Courts
High Courts
Supreme Courts
This hierarchy holds good for all IPRs, including patents, trade marks, designs
and Geographical Indications.
Remedies for Infringement under Indian Laws fall in two categories :
A. Civil Actions :
Plaintiffs can seek ad interim and interim relief, including,
injunctions,
orders for discovery and inspection,
orders for interrogatories.
B. Criminal Actions
Intellectual Property Rights can also be enforced by initiating infringement
actions through criminal prosecution.
The IP statutes provide for punishments and fines in respect of
infringements of different Ips.
Conclusion :
It is undisputed that creation of intellectual property is critical to Indias
economic development and the achievement of social and strategic goals.
The only controversial issue is how to use IP and how best to reward its creator.
India is witnessing rapid development in almost all sectors.
Indian IP law and mechanisms are also changing to adapt to global practices .
There is a need to back up intellectual property activity in India to achieve the
economic goals.
Indias IP systems and mechanisms are becoming rich and diversified which is
very much necessary for spurring and supporting technology innovation for
socio-economic benefits.

This is Module-1. Menu ---> CONTENTS | Module-1 | Module-2 | Module-3 | Module-4

Discuss : Assignment and Licensing of IP.


ANSWER :
Refer :
http://epgp.inflibnet.ac.in/epgpdata/uploads/epgp_content/law/08._intellectual_pr
operty_law/27._intellectual_property_assignment_and_licensing_/et/5795_et_27_

CONTENTS | Module-1 | Module-2 | Module-3 | Module-4 http://duralex.bhatt.net.in/


https://www.facebook.com/groups/Dharmo.Rakshati.Rakshitah/ Page - 59 of 196

et.pdf
Outline :
Need for knowledge about licensing & assignment of IP :
Transfer of IP Rights :
Assignment :
License :
Difference between Assignment and Licenses
Need for knowledge about licensing & assignment of IP :
Intellectual property (IP) is not wealth in itself but only a tool or source of it .
And this reservoir of wealth is made to flow only when IP is used to produce and
market goods and services.
The creator may utilize his IP himself but the creator himself is often unable to
utilize his own property or utilize it to its fullest extent.
When he invites others to utilize his property, he has to rely largely on the
mechanism of licensing or assignment by entering into contracts with others.
It is undeniable that IP licensing plays a major role in todays business and
economy.
Business transactions in intellectual property are highly ubiquitous.
eg Online contracts wherein we often click on I agree while downloading
music, films, computer software, books, etc or using online information,
are mostly in the form of licenses.
From off-the-shelf software to complex technology transfer arrangements, all
depend on the same licensing law principles.
Virtually every business today is confronting licensing issues on ever increasing
scales.
Hence, a solid base of knowledge about licensing & assignment of IP has never
been more important for students of law.
Transfer of IP Rights :
IPRs (eg copyright, patents, trademarks, designs, trade secrets, etc) are tradable
and can be transferred through the instrument of contract.
Mobilizing the flexibility of the instrument of contract an owner of IP could invite
others to utilize his property in following different ways,
by assignment wherein all rights in a property are assigned to another person
in return of some consideration.
by selectively granting certain rights and withhold others which is known as a
license.
The format chosen for the transaction of IP, affects and reflects the rights and
obligations of the parties.

CONTENTS | Module-1 | Module-2 | Module-3 | Module-4 http://duralex.bhatt.net.in/


https://www.facebook.com/groups/Dharmo.Rakshati.Rakshitah/ Page - 60 of 196

Assignment :
The term assignment is defined as a transfer or making over to another
of the whole of any property,
real or personal,
in possession or in action,
or of any estate or right therein.
What assignment is to intangible property,
sale is to tangible property.
ie IP is transmissible by assignment as personal or movable property.
By assigning his IP to another, the owner transfers his legal title to the assignee.
In assignment
the ownership rights of the IP pass from seller to buyer
and it is a one-time activity, conveying full rights in the underlying IP.
Usually, assignment involves compensation in the form of a lump sum payment in
one go but it might also be deferred to be made dependent on certain factors, such
as the success of the commercialization of the transferred IP.
The assignment of intellectual property permits a greater freedom to the assignee
in marketing of the products produced by the IP concerned than could be the case
with a license.
License :
A license can also be appreciated as a legal instrument through which the owner
could invite others in the utilization of his IP.
Meaning of license :

CONTENTS | Module-1 | Module-2 | Module-3 | Module-4 http://duralex.bhatt.net.in/


https://www.facebook.com/groups/Dharmo.Rakshati.Rakshitah/ Page - 61 of 196

Law confers exclusive rights to the owner of IP to do various acts.


An infringement occurs if any one of these acts is done without license.
A license, therefore, passes no interest but merely makes lawful that which
would otherwise be unlawful.
eg a patent license is a waiver by the patent owner of his right to exclude the
licensee from making, using, selling, offering for sale or importing the claimed
invention.
A license is a promise not to sue a party for actions that would otherwise constitute
infringement.
A license is a contract that allocates rights and limitations in the use of an IP right.
The extent of license rights may be,
non-exclusive, permission to use the licensed IP in some limited manner,
exclusive, permission to all but the ownership of IP.
Difference between Assignment and Licenses :
Both licenses and assignments are contracts,
and both result in transfer of intellectual property.
However, they entail very different legal and practical consequences.
They also serve very different business purposes.
Further, it may be rather difficult to state whether a particular transaction is an
assignment or license.
The relationship between assignor and assignee and between licensor and licensee
is contractual.
That means, technically, all parties can commit breach of such contract leading
to an action for breach of contract as well as for violation of underlying IP.
It is, therefore, extremely important to appreciate their precise nature and
accurate import. Differentiating between them will also bring forth some of their
salient features which are as follows :
A license is a transfer by the owner of IP to another of rights less in degree than
the property itself. For example, any transfer of a patent right short of
assignment could be described as a license.
Assignment is the transfer of the entire interest in an IP or of an undivided
portion of such entire interest.
The nature of licensees title is not proprietary,
while, nature of assignees title is proprietary.
Since the rights of a licensee are not proprietary, the license will not bind a
purchaser from the intellectual property owner acting in good faith for valuable
consideration and without notice of the license;
whereas the title of an assignee, being proprietary in nature, is good against all

CONTENTS | Module-1 | Module-2 | Module-3 | Module-4 http://duralex.bhatt.net.in/


https://www.facebook.com/groups/Dharmo.Rakshati.Rakshitah/ Page - 62 of 196

subsequent dealings, including a subsequent purchaser from his assignor.


A licensee must act within the scope of the licenseotherwise he will infringe.
An assignee or his successors may deal with the assigned property as they wish
subject only to any contractual restrictions.
A licensor cannot be sued for infringement by even the exclusive licensee.
An assignor can sued by the assignee for infringement of underlying IP.
A failure to pay royalties on part of the licensee may lead to revocation of license,
while a failure to pay royalties in case of an assignment cannot lead to
revocation of assignment. The assignor cannot get the property back after it has
been assigned.
Licensing is usually on consideration of periodic payments called royalties over the
productive life of the property.
Assignment of IP rights is usually conditional on payment of lump sum.
However, there is no hard and fast rule as to that; so, at times, a licensor may
get lump sum and an assignor may get royalties.
Even if both the licensor and the assignor are getting royalties, still there will be
a difference in their treatment under taxation laws.
Royalties paid under a license, however, are deductible business expenses of
the licensee and comprise ordinary income for the licensor.
Payments made for an assignment of an IP must be capitalized by the
assignee and may be taxed as capital gains to the assignor.

This is Module-1. Menu ---> CONTENTS | Module-1 | Module-2 | Module-3 | Module-4

Discuss : Effects of legislations of Patent, Copyright and Trademark in India.


Discuss the amendments done in various legislations relating to intellectual
properly law in India and their effects, (Nov-2014)
Explain in detail the Amendments in various legislations relating to IPRs in India.
(Apr-2016)
ANSWER :
Refer :

Amendments in various legislations relating to intellectual properly law :

This is Module-1. Menu ---> CONTENTS | Module-1 | Module-2 | Module-3 | Module-4

CONTENTS | Module-1 | Module-2 | Module-3 | Module-4 http://duralex.bhatt.net.in/


https://www.facebook.com/groups/Dharmo.Rakshati.Rakshitah/ Page - 63 of 196

Module-2 :
2) Select Aspects of the Copyright Law in India :
2.1) Historical evolution of the law
2.2) Meaning of copyright,
2.3) Availability of Copyright protection:
2.3.1) Copyright in literary and Artistic work, dramatic and musical Works
2.3.2) Copyright in Sound records and cinematograph films
2.3.3) Copyright in computer programme, Internet and database
2.4) Author and Ownership of copyright, Term of copyright, authorities under
the Copyright Act and their powers, Registration of copyright, Copyright
Society and its powers, Appeal provisions
2.5) Rights conferred by copyright, Assignment, Transmission and
relinquishment of copyright, Provisions relating to licence
2.6) Infringement of Copyright : Criteria, various types of infringement
2.7) Acts not constituting infringement, Fair use provisions, Piracy in internet
2.8) International copyright
2.9) Remedies and Penalty provisions

This is Module-2. Menu ---> CONTENTS | Module-1 | Module-2 | Module-3 | Module-4

MODULE-2 QUESTIONS :

Explain in detail the meaning of Copyright and state the Historical evolution of
the Copyright Law in India. (Apr-2016)
Discuss : Amendments in Copyright Law in India.
Explain the meaning of Copyright and discuss its essential elements, (Nov-2014)
Discuss the meaning of copyright and work in which copyright subsists under the
Copyright Act, 1957. (Nov-2011)
Explain under the Copyright Act : Meaning and characteristic of Copyright (Oct-
2013)
Discuss the essential elements of Copyrights. (Dec-2015)
Discuss availability of copyright in, (i) literary and Artistic work, dramatic and
musical Works, (ii) Sound records and cinematograph films, (iii) computer
programme, Internet and database.
Discuss : Government work (Under Copyright Act) (Dec-2015)
Explain under the Copyright Act : Author (Oct-2013)

CONTENTS | Module-1 | Module-2 | Module-3 | Module-4 http://duralex.bhatt.net.in/


https://www.facebook.com/groups/Dharmo.Rakshati.Rakshitah/ Page - 64 of 196

Discuss : (i) Ownership of copyright, (ii) First owner of copyright, (iii) Term of
copyright, (iv) Authorities under the Copyright Act and their powers, (v)
Registration of copyright, (vi) Copyright Society and its powers, (vii) Appeal
provisions.
Write note : First owner of copyright. (Nov-2011, Nov-2012)
Explain under the Copyright Act : First owner of Copyright (Oct-2013)
Write note : Term of copyright (Nov-2011, Nov-2012, Oct-2013, Dec-2015)
Explain under the Copyright Act : Powers and procedure of copyright board. (Oct-
2013, Nov-2014, Dec-2015)
Explain : Copyright Societies. (Nov-2012)
Explain under the Copyright Act : Copyright Societies (Oct-2013)
Discuss : Rights conferred by copyright, (i) Assignment & Transmission, (ii)
relinquishment of copyright, (iii) Provisions relating to licence.
Discuss the infringement of copyright under the Copyright Act, 1957. (Nov-2011)
Explain in detail the various kinds of infringement of copyrights and state the
remedies under the Copyright Law. (Apr-2016)
Write short note : Infringement of Literary Work (Dec-2015)
Discuss : (i) Acts not constituting infringement, (ii) Fair use provisions, (iii)
Piracy in internet.
Explain act not be infringement of Copyright under the Copyright Act, 1999. (Nov-
2012)
Explain : The remedies available against infringement of Copyright. (Nov-2012)
Explain under the Copyright Act : Civil remedies available against Infringement of
copyright (Oct-2013)
Explain : International Copyright. (Nov-2012)
Explain under the Copyright Act : International Copyright (Oct-2013)

This is Module-2. Menu ---> CONTENTS | Module-1 | Module-2 | Module-3 | Module-4

MODULE-2 ANSWERS :

Explain in detail the meaning of Copyright and state the Historical evolution of
the Copyright Law in India. (Apr-2016)
Discuss : Amendments in Copyright Law in India.
Explain the meaning of Copyright and discuss its essential elements, (Nov-2014)
Discuss the meaning of copyright and work in which copyright subsists under the

CONTENTS | Module-1 | Module-2 | Module-3 | Module-4 http://duralex.bhatt.net.in/


https://www.facebook.com/groups/Dharmo.Rakshati.Rakshitah/ Page - 65 of 196

Copyright Act, 1957. (Nov-2011)


Explain under the Copyright Act : Meaning and characteristic of Copyright (Oct-
2013)
Discuss the essential elements of Copyrights. (Dec-2015)
ANSWER :
Refer :
http://copyright.gov.in/documents/handbook.html <-- Key document for Copyright
http://www.wipo.int/edocs/pubdocs/en/intproperty/450/wipo_pub_450.pdf
Note : Read from Module-1 ---> Berne Convention
Berne Convention for the Protection of Literary and Artistic Works
an international treaty on copyrights.
Outline :
What are Copyright?
What are Related Rights?
Scope of the Copyright Act 1957 :
Work in which copyright subsists
Rights of the holder of (i) copyrights, and (ii) related rights
Characteristic of Copyright
Enforcement of copyright and related rights
What are the benefits of protecting copyright and related rights?
How have copyright and related rights kept up with advances in technology?
How are copyright and related rights regulated?
Brief History of Copyright Law in India
Amendments in Copyright Law in India
What are Copyright?
Creativity being the keystone of progress, no civilized society can afford to ignore
the basic requirement of encouraging the same.
The term copyright is not defined under the Indian Copyright Act, 1957.
The general connotation of the term copyright refers to the right to copy which is
available only to the author or the creator, as the case may be.
Thus, any other person who copies the original work would be amount to
infringement under the Copyright Act 1957.
Copyright laws grant authors, artists and other creators protection for their literary
and artistic creations, generally referred to as works.
Copyright ensures certain minimum safeguards of the rights of authors over their
creations.

CONTENTS | Module-1 | Module-2 | Module-3 | Module-4 http://duralex.bhatt.net.in/


https://www.facebook.com/groups/Dharmo.Rakshati.Rakshitah/ Page - 66 of 196

What are Related Rights?


A closely associated field is related rights or rights related to copyright,
that encompass rights similar or identical to those of copyright, although
sometimes more limited and of shorter duration.
The beneficiaries of related rights are :
performers (such as actors and musicians) in their performances;
producers of phonograms (for example, compact discs) in their sound
recordings; and
broadcasting organizations in their radio and television programs.
Scope of the Copyright Act 1957 :
Copyright as provided by the Indian Copyright Act 1957 is valid only within the
borders of the country.
To secure protection to Indian works in foreign countries, India has become a
member of the following international conventions on copyright and
neighbouring (related) rights :
a. Berne Convention for the Protection of Literary and Artistic works.
b. Universal Copyright Convention.
c. Convention for the Protection of Producers of Phonograms against
Unauthorised Duplication of their Phonograms.
d. Multilateral Convention for the Avoidance of Double Taxation of Copyright
Royalties.
e. Trade Related Aspects of Intellectual Property Rights (TRIPS) Agreement.
Work in which copyright subsists : (Chapter III, Sec-13 of Copyright Act 1957)
Literary works (including novels, poems, plays, reference works, newspapers,
advertisements, computer programmes, tables and compilations including
computer literary data bases)
Dramatic works
Musical works
Artistic works
Cinematograph films
Sound recordings.
Foreign Works :
The copyright of foreign works is also protected in India. Copyright of nationals
of countries who are members of
the Berne Convention for the Protection of Literary and Artistic Works,
Universal Copyright Convention
and the TRIPS Agreement are protected in India through the International

CONTENTS | Module-1 | Module-2 | Module-3 | Module-4 http://duralex.bhatt.net.in/


https://www.facebook.com/groups/Dharmo.Rakshati.Rakshitah/ Page - 67 of 196

Copyright Order,
as if such works are Indian works
Rights of the holder of (i) copyrights, and (ii) related rights :
SHORT NOTE :
Creators of works protected by copyright, and their heirs and successors
(generally referred to as right holders), have certain basic rights under
copyright law.
They hold the exclusive right to use or authorize others to use the work on
agreed terms.
Holder of (i) copyrights and (ii) related rights can authorize or prohibit,
reproduction of their work in all forms, including print form and sound
recording;
public performance and communication of their work to the public;
broadcasting of their work ;
translation of their work into other languages; and its adaptation, such as from
a novel to a screenplay for a film.
LONG NOTE :
Owner of a Copyright has following rights under the Copyright Act 1957 :
Reproduction.
Communication.
Adaptation.
Translation.
REPRODUCTION :
The Copyright confers upon the assessee the sole right to reproduce the
authored work.
In other words, no other person except the author shall make copies (one or
many) of the work or copy the substantial part of the work in any form
including sound and film recording etc without the permission of the copyright
owner.
For example, a person buys a film CD and the person makes multiple copies of
it and sells it to others. This would amount to copyright infringement.
COMMUNICATION :
Communication to the public means making any work available to general
public for the purpose of being seen or heard or otherwise enjoyed by the
public directly or by any means of display or diffusion.
It is not necessary that any member of the public actually sees, hears or
otherwise enjoys the work so made available.
For example, a cable operator may transmit a cinematograph film.

CONTENTS | Module-1 | Module-2 | Module-3 | Module-4 http://duralex.bhatt.net.in/


https://www.facebook.com/groups/Dharmo.Rakshati.Rakshitah/ Page - 68 of 196

Even while, no member of the public see the communication, still it is a


communication to the public.
The fact that the work in question is accessible to the public is enough to
say that the work is communicated to the public.
ADAPTATION :
Adaptation involves the preparation of a new work in the same or different
form based upon an already existing work.
The Copyright Act defines the following acts as adaptations :
a. Conversion of a dramatic work into a non-dramatic work
b. Conversion of a literary or artistic work into a dramatic work.
c. Re-arrangement of a literary or dramatic work
d. Depiction in a comic form or through pictures of a literary or dramatic
work
e. Transcription of a musical work or any act involving re-arrangement or
alteration of an existing work.
For example, the book Five Point Someone written by Chetan Bhagat was
made as a film named 3 Idiots in Hindi. It is noted that the concept of the
film alone was taken and not the whole of its expression. Again, the remake of
the film 3 idiots was done in Tamil in the name of Nanban. Again here
some alterations were made to suit the targeted audience and therefore, only
amounted to copying of idea and not the expression.
TRANSLATION :
Similarly, the owner has the full and sole authority to translate the work done
by him in one language to one or many other languages.
Any other person interested in doing so must get the prior permission of the
owner.
For example, a film taken in English can be dubbed or remade only by the
owner or any other person with the consent of the owner.
Characteristic of Copyright :
Copyright and related rights protection is obtained automatically without the need
for registration or other formalities.
Term of copyright protection enables both creators and their heirs and successors
to benefit financially for a reasonable period of time.
Copyright protects the expression and not the content or substance per se.
For example, an author writes about making of an aircraft. Here, the idea of
making of the plane is not protected but the only the way of expressing is
protected. The idea is protected under the Patent law and not under Copyright
Act 1957.

CONTENTS | Module-1 | Module-2 | Module-3 | Module-4 http://duralex.bhatt.net.in/


https://www.facebook.com/groups/Dharmo.Rakshati.Rakshitah/ Page - 69 of 196

Many types of works protected under the laws of copyright and related rights
require mass distribution, communication and financial investment for their
successful dissemination (for example, publications, sound recordings and films).
Hence, creators often transfer these rights to companies,
who, in turn, develop and market the works,
in return for compensation in the form of payments and/or royalties
(compensation based on a percentage of revenues generated by the work).
Duration :
Copyrights : The economic rights relating to copyright are of limited duration
as provided for in the relevant WIPO treaties
beginning with the creation and fixation of the work,
and lasting for not less than 50 years after the creators death.
National laws may also establish longer terms of protection.
Related rights : Related rights enjoy shorter terms, normally 50 years after the
performance, recording or broadcast has taken place.
Copyright and the protection of performers also include moral rights, meaning the
right to claim authorship of a work, and the right to oppose changes to the work
that could harm the creators reputation.
Enforcement of copyright and related rights :
Rights provided for under copyright and related rights laws can be enforced by
right holders through a variety of methods and fora,
including civil action suits,
administrative remedies
and criminal prosecution.
Injunctions, orders requiring destruction of infringing items, inspection orders,
among others, are used to enforce these rights.
What are the benefits of protecting copyright and related rights?
Copyright and related rights protection is an essential component in fostering
human creativity and innovation.
Giving authors, artists and creators incentives in the form of recognition and fair
economic reward increases their activity and output and can also enhance the
results.
By ensuring the existence and enforceability of rights,
individuals and companies can more easily invest in the creation, development
and global dissemination of their works.
This, in turn, helps to increase access to and enhance the enjoyment of culture,
knowledge and entertainment the world over, and also stimulates economic and
social development.

CONTENTS | Module-1 | Module-2 | Module-3 | Module-4 http://duralex.bhatt.net.in/


https://www.facebook.com/groups/Dharmo.Rakshati.Rakshitah/ Page - 70 of 196

How have copyright and related rights kept up with advances in technology?
The field of copyright and related rights has expanded enormously during the last
several decades
with the spectacular progress of technological development that has, in turn,
yielded new ways of disseminating creations by such forms of communication as
satellite broadcasting, compact discs and DVDs.
Widespread dissemination of works via the Internet raises difficult questions
concerning copyright and related rights in this global medium.
WIPO is fully involved in the ongoing international debate to shape new standards
for copyright protection in cyberspace.
In that regard, the Organization administers the WIPO Copyright Treaty (WCT) and
the WIPO Performances and Phonograms Treaty (WPPT), known as the Internet
Treaties.
These treaties clarify international norms aimed at preventing unauthorized access
to and use of creative works on the Internet.
How are copyright and related rights regulated?
Copyright and related rights protection is obtained automatically without the need
for registration or other formalities.
However, many countries provide for a national system of optional registration and
deposit of works.
These systems facilitate, for example, questions involving disputes over ownership
or creation, financial transactions, sales, assignments and transfer of rights.
Brief History of Copyright Law in India :
The evolution of Copyright Law in India is spread over three phases.
In the first phase, the law of copyright was introduced in India during the reign of
the British Rule in India via the British Copyright Act, 1911.
This Act had very different provisions in comparison to todays law.
The term of the Copyright was life time of the author plus seven years after the
death of the author.
However the total term of copyright cannot exceed the period of forty-two years.
The government could grant a compulsory licence to publish a book if the owner
of copyright, upon the death of the author, refused to allow its publication.
Registration of Copyright with the Home Office was mandatory for enforcement
of rights under the Act.
The second phase was in 1914, when the Indian legislature under the British Raj
enacted the Copyright Act of 1914.
It was almost similar to the British Copyright Act of 1911.
However the major change that was brought in this Act was the criminal

CONTENTS | Module-1 | Module-2 | Module-3 | Module-4 http://duralex.bhatt.net.in/


https://www.facebook.com/groups/Dharmo.Rakshati.Rakshitah/ Page - 71 of 196

sanction for infringement.


The 1914 Act was amended a number of times.
Subsequently, India saw the third phase of its copyright law evolution in the
introduction of the Indian Copyright Act, 1957 which was enacted in order to suit
the provisions of the Berne Convention.
This Act was enacted by Independent India and is the main Act by which we are
governed till date.
Amendments in Copyright Law in India :
Certain changes were incorporated by way of the Copyright (Amendment) Act,
2012 to curb infringement of copy rights.
The existing clause (1)(a) has been amended to provide fair dealing with any work
for the purposes of private and personal use
with an exception that of a computer programme.
Cinematograph and musical works were also brought under the ambit of the works
to which the fair use provision has been extended to.
Special needs of disabled :
Provision was made for fair use of the work aimed at the benefiting the disabled.
It facilitates reproducing, issuing of copies, adapting or communicating to the
public any work in any accessible format, for disabled persons to access works
including sharing with any person with disability for private or personal use,
research or for any other educational purposes.

This is Module-2. Menu ---> CONTENTS | Module-1 | Module-2 | Module-3 | Module-4


GO TO MODULE-2 QUESTIONS.
GO TO CONTENTS.

Discuss availability of copyright in, (i) literary and Artistic work, dramatic and
musical Works, (ii) Sound records and cinematograph films, (iii) computer
programme, Internet and database.
Discuss : Government work (Under Copyright Act) (Dec-2015)
ANSWER :
Refer :

(i) Literary and Artistic work, dramatic and musical Works :
<Read Berne convention from Module-1>
(ii) Sound records and cinematograph films :
Introduced vide 2012 amendment in the Copyrights Act 1957.

CONTENTS | Module-1 | Module-2 | Module-3 | Module-4 http://duralex.bhatt.net.in/


https://www.facebook.com/groups/Dharmo.Rakshati.Rakshitah/ Page - 72 of 196

(iii) Copyright on computer programme, Internet and database :


Outline :
Hyperlinking (making links to other web pages)
Precautions before creating hyperlinks :
Using images
Using Screenshots :
Using Logos :
Introduced vide 2012 amendment in the Copyrights Act 1957.
Whenever using information or material from the Internet, it is important to
remember that, unless explicitly stated otherwise,
the majority of such resources will be subject to copyright restrictions and will be
the property of the copyright holder.
Even if there is no copyright statement on the material you are looking at, you
must not assume that it is copyright-free.
Always look for copyright notices or terms and conditions of use stated on
resources themselves.
Just because it is easy to access, you must not assume that
information posted on the Internet is freely available to be used in any way you
choose.
Copying someone elses web page to be adapted for your own purposes is a
copyright infringement unless permission is obtained.
Hyperlinking (making links to other web pages) :
It is common practice for web pages to include links to external web sites, and
this does not usually cause a problem, but there are some risks involved if care
is not taken.
It is good practice to check for any conditions that might apply to a particular
web site, and if in doubt to contact the copyright owner.
It is wise to avoid the practice of deep-linking, that is to say hyper-linking
directly to material in someone elses site and by-passing the home page.
In doing so, there is a danger that you remove the identification of the
owner/creator of the original site, and appear to claim the content as your
own.
Precautions before creating hyperlinks :
It is advisable to create a link to the home page of a web site rather than linking
to a deep level of the site.
This helps to avoid the issues of referring to information out of context, or
seeming to pass information off as your own.
Clearly label links with the name of the web site and the individual or corporate

CONTENTS | Module-1 | Module-2 | Module-3 | Module-4 http://duralex.bhatt.net.in/


https://www.facebook.com/groups/Dharmo.Rakshati.Rakshitah/ Page - 73 of 196

author. This will help inform your users of what they are looking at and avoid the
issue of passing the information off as your own.
When you are creating hypertext links you should be careful not to authorise
users to make copies of the site you are referencing unless this is expressly
permitted.
Take care that you do not quote or link to another site in a derogatory manner,
by quoting out of context or making an inference that is not directly supported
by evidence.
Avoid the use of frames and do not link from frames to another external site
without the specific permission of the owner of the site you wish to link to. If you
do link to an external web site make sure that it opens in a separate frame to
avoid any confusion or possible misinterpretation.
Using images :
Images on the internet are not copyright free , and care should be taken in their
use. There are a number of sources of royalty-free images and pictures on the
internet, and these sites will state quite clearly the terms and conditions of their
use.
Many websites do NOT allow their work to be copied even for non-commercial
purposes.
If it is not clear from the site that the rights holder is happy for the image to be
used for your particular purpose, you should always seek permission before you
do so.
Always acknowledge your source;
Never alter the image.
Using Screenshots :
There may well be multiple copyrights in screenshots, including fonts, graphics
etc.
If using these for learning and teaching purposes you must avoid any alteration
to the original, and any misleading labelling.
Using Logos :
Care must be taken with use of company logos, particularly where these are
used to click through to a web page.
Such use, without permission, would infringe the companys trademark. There
have been several high-profile legal suits resulting from such uses.
(iv) Government work :
Definition : Sec-2(k) of the Copyright Act, 1957 :
Government work means
a work which is made or published by or under the direction or control of

CONTENTS | Module-1 | Module-2 | Module-3 | Module-4 http://duralex.bhatt.net.in/


https://www.facebook.com/groups/Dharmo.Rakshati.Rakshitah/ Page - 74 of 196

(i) the Government or any department of the Government;


(ii) any Legislature in India;
(iii) any court, tribunal or other judicial authority in India;

This is Module-2. Menu ---> CONTENTS | Module-1 | Module-2 | Module-3 | Module-4

Explain under the Copyright Act : Author (Oct-2013)


ANSWER :
Refer :

Definition : Sec-2(d) of the Copyright Act, 1957 :
author means,
(i) in relation to a literary or dramatic work,
the author of the work;
(ii) in relation to a musical work,
the composer;
(iii) in relation to an artistic work other than a photograph,
the artist;
(iv) relation to a photograph,
the person taking the photograph;
(v) in relation to a cinematograph film or sound recording,
the producer;
(vi) in relation to any literary, dramatic, musical or artistic work which is
computer-generated,
the person who causes the work to be created;

This is Module-2. Menu ---> CONTENTS | Module-1 | Module-2 | Module-3 | Module-4

Discuss : (i) Ownership of copyright, (ii) First owner of copyright, (iii) Term of
copyright, (iv) Authorities under the Copyright Act and their powers, (v)
Registration of copyright, (vi) Copyright Society and its powers, (vii) Appeal
provisions.
Write note : First owner of copyright. (Nov-2011, Nov-2012)
Explain under the Copyright Act : First owner of Copyright (Oct-2013)
Write note : Term of copyright (Nov-2011, Nov-2012, Oct-2013, Dec-2015)
Explain under the Copyright Act : Powers and procedure of copyright board. (Oct-

CONTENTS | Module-1 | Module-2 | Module-3 | Module-4 http://duralex.bhatt.net.in/


https://www.facebook.com/groups/Dharmo.Rakshati.Rakshitah/ Page - 75 of 196

2013, Nov-2014, Dec-2015)


Explain : Copyright Societies. (Nov-2012)
Explain under the Copyright Act : Copyright Societies (Oct-2013)
ANSWER :
Refer :
http://www.saprlaw.com/taxblog/copyright_final.pdf
(i) Ownership of copyright :
Generally, the creator or the author of the work is the owner of the work and
therefore entitled to get the copyright for the work. However,
Where the author of the work is employed by another person, the work belongs
to the employer of the author.
Where creation of the works is incidental, but not the purpose, the work belongs
to the authors.
But in practice, out of the contractual agreement between the employer and the
employee, the creation during the course of employment would be belonging to the
employer.
Multiple authors :
There may be a situation where a particular final work involves many
copyrightable sub-divisions such as film wherein many works such as music,
lyrics, dramatic works etc are copyrightable.
The authors in the creation of such work are many such as :
a. In the case of a musical work, the composer.
b. In the case of a cinematograph film, the producer.
c. In the case of a sound recording, the producer.
d. In the case of a photograph, the photographer.
e. In the case of a computer generated work, the person who causes the work
to be created.
f. In the case of Script, the writer. etc.
(ii) First owner of the copyright :
Essence of Sec-17(1)(a, b, c) :
in the case of a literary, dramatic or artistic work made by the author in the
course of his employment or contract of service
or a photograph taken, or a painting or portrait drawn, or an engraving or a
cinematograph film made, for valuable consideration,
in so far as the copyright relates to the publication of the work or reproduction of
the work, for the purpose of its being so published is concerned,
the said employer shall be (in absence of any agreement to the contrary)

CONTENTS | Module-1 | Module-2 | Module-3 | Module-4 http://duralex.bhatt.net.in/


https://www.facebook.com/groups/Dharmo.Rakshati.Rakshitah/ Page - 76 of 196

the first owner of the copyright in the work


but in all other respects,
the author shall be the first owner of the copyright in the work.
Essence of Sec-17(1)(d, dd) : in the case of
a Government work, or a public undertaking work
such Government (or public undertaking) shall be, in the absence of any
agreement to the contrary, the first owner of the copyright therein;
Illustration :
A film producer hires a composer to create a sound track, for some
consideration,
Here,
so far as the the publication & reproduction rights are concerned
depending upon the terms of agreement,
film producer may be the first owner of the copyright on the sound track,
while, in all other respects,
music composer may be the first owner of the copyright.
(iii) Term of copyright :
Generally copyright lasts for Life + 60 years in India. Provisions for term of
copyright for different kinds of works are as follows,
Original literary, dramatic, musical and artistic works,
60-year from the year following the death of the author. In case of joint
authorship, the date has reference to author who dies last.
Posthumous Work
60 years from the date of demise of the owner
Anonymous and pseudonymous publications
60 years from beginning of the calendar year following the year of publication.
Photographs
60 years from the beginning of the calendar year next following the year in
which the photograph is published
Work of Government, Public undertaking and International Organisation
60 years from the year next to the year of publication.
Cinematographic film
60 year from the post calendar year of the release of the Film.
(iv) Authorities under the Copyright Act and their powers :
The Copyright Act 1957 provides for two authorities :
(a) Registrar of Copyrights, {Sec-9 & 10}

CONTENTS | Module-1 | Module-2 | Module-3 | Module-4 http://duralex.bhatt.net.in/


https://www.facebook.com/groups/Dharmo.Rakshati.Rakshitah/ Page - 77 of 196

(b) Copyright Board, {Sec-11 & 12}


(a) Registrar of Copyrights :
Sec-9 : Copyright Office :
(1) There shall be established for the purposes of this Act an office to be called
the Copyright Office.
(2) The Copyright Office shall be under the immediate control of the Registrar
of Copyrights who shall act under the superintendence and direction of the
Central Government.
Sec-10 : Registrar and Deputy Registrars of Copyrights :
(1) The Central Government shall appoint a Registrar of Copyrights and may
appoint one or more Deputy Registrars of Copyrights.
Powers of the Registrar of Copyrights :
The Registrar of Copyrights has the powers of a civil court when trying a suit
under the Code of Civil Procedure in respect of the following matters, namely,
a. summoning and enforcing the attendance of any person and examining
him on oath;
b. requiring the discovery and production of any document;
c. receiving evidence on affidavit;
d. issuing commissions for the examination of witnesses or documents;
e. requisitioning any public record or copy thereof from any court or office;
f. any other matters which may be prescribed.
(b) Copyright Board :
There are no special courts for the purpose of dealing with copyright cases.
Since, regular courts lack knowledge and expertise in the field of copyright.
there is a Copyright Board to adjudicate certain cases pertaining to copyright.
The government has set up a Copyright Enforcement Advisory Council (CEAC) to
adjudicate certain matters relating to copyright.
Sec-11 : Composition :
Sec-11 of the The Copyright Act 1957 provides for a quasi-judicial body called
the Copyright Board consisting of,
a Chairman,
and two or more, but not exceeding fourteen, other members
for adjudicating certain kinds of copyright cases.
The Chairman of the Board is of the level of a judge of a High Court.
Sec-12 : Powers and procedure of Copyright Board :
(1) The Copyright Board has power to regulate its own procedure, including
the fixing of places and times of its sittings

CONTENTS | Module-1 | Module-2 | Module-3 | Module-4 http://duralex.bhatt.net.in/


https://www.facebook.com/groups/Dharmo.Rakshati.Rakshitah/ Page - 78 of 196

(2) The Copyright Board may exercise and discharge its powers and functions
through Benches constituted by the Chairman of the Copyright Board from
amongst its members.
(7) The Copyright Board
shall be deemed to be a civil court for the purposes of sections 345 and 346
of the CrPC 1973,
and all proceedings before the Board shall be deemed to be judicial
proceedings within the meaning of sections 193 and 228 of the IPC.
The Board has power to :
i. hear appeals against the orders of the Registrar of Copyright;
ii. hear applications for rectification of entries in the Register of Copyrights;
iii. adjudicate upon disputes on assignment of copyright;
iv. grant compulsory licences to publish or republish works;
v. grant compulsory licence to produce and publish a translation of a literary
or dramatic work in any language after a period of seven years from the
first publication of the work;
vi. hear and decide disputes
as to whether a work has been published
or about the date of publication
or about the term of copyright of a work in another country;
vii. fix rates of royalties in respect of sound recordings under the cover-
version provision; and
viii. fix the resale share right in original copies of a painting, a sculpture or a
drawing and of original manuscripts of a literary or dramatic or musical
work.
(v) Registration of copyright :
Copyright is automatic once the original work is created and it does not require
any formality.
However, certificate of registration of copyright and the entries made therein
serve as prima facie evidence in a court of law with reference to dispute relating
to ownership of copyright.
Procedure for registration : Chapter VI of the Copyright Rules, 1956 sets out the
procedure for the registration under the Copyright Act. The procedure for
registration is as follows :
a. Application for registration is to be made on Form IV ( Including Statement
of Particulars and Statement of Further Particulars) as prescribed in the first
schedule to the Rules ;
b. Separate applications should be made for registration of each work;

CONTENTS | Module-1 | Module-2 | Module-3 | Module-4 http://duralex.bhatt.net.in/


https://www.facebook.com/groups/Dharmo.Rakshati.Rakshitah/ Page - 79 of 196

c. Each application should be accompanied by the requisite fee prescribed in


the second schedule to the Rules ; and
d. The applications should be signed by the applicant or the advocate in whose
favour a Vakalatnama or Power of Attorney has been executed. The Power of
Attorney signed by the party and accepted by the advocate should also be
enclosed.
Each and every column of the Statement of Particulars and Statement of Further
Particulars should be replied specifically.
Both published and unpublished works can be registered.
Copyright in works published before 21 st January, 1958, i.e., before the
Copyright Act, 1957 came in force, can also be registered, provided the works
still enjoy copyright.
Three copies of published work may be sent along with the application.
If the work to be registered is unpublished, a copy of the manuscript has to be
sent along with the application for affixing the stamp of the Copyright Office in
proof of the work having been registered.
In case two copies of the manuscript are sent, one copy of the same duly
stamped will be returned, while the other will be retained, as far as possible, in
the Copyright Office for record and will be kept confidential.
Also it would also be open to the applicant to send only extracts from the
unpublished work instead of the whole manuscript and ask for the return of the
extracts after being stamped with the seal of the Copyright Office.
When a work has been registered as unpublished and subsequently it is
published, the applicant may apply for changes in particulars entered in the
Register of Copyright in Form V with prescribed fee.1.4
Advantage of Registration :
Registration establishes a public record of the copyright claim.
Before an infringement suit may be filed in court, registration is necessary for
works.
Registration establishes sufficient evidence in court concerning the validity of
the copyright and the facts stated in the copyright certificate.
If registration is made, statutory damages and attorney's fees will be available
to the copyright owner in court actions. Otherwise, only an award of actual
damages and profits is available to the copyright owner.
Registration allows the owner of the copyright to record the registration with
the Indian Customs for protection against the importation of infringing copies.
(vi) Copyright Society : Collective management organizations (CMOs) :
Sec-34 : Administration of rights of owner by copyright society :
(1) Subject to such conditions as may be prescribed,

CONTENTS | Module-1 | Module-2 | Module-3 | Module-4 http://duralex.bhatt.net.in/


https://www.facebook.com/groups/Dharmo.Rakshati.Rakshitah/ Page - 80 of 196

(a) a copyright society may accept


from an author and other owners of right
exclusive authorisation to administer any right in any work
by issue of licences or collection of licence fees or both; and
(b) an author and other owners of right
shall have the right to withdraw such authorisation without prejudice to the
rights of the copyright society under any contract.
(2) It shall be competent for a copyright society ( powers of society)
to enter into agreement with any foreign society or organisation administering
rights corresponding to rights under this Act,
to entrust to such foreign society or organisation
the administration in any foreign country of rights administered by the said
copyright society in India,
or for administering in India the rights administered in a foreign country by
such foreign society or organisation:
(3) Subject to such conditions as may be prescribed, a copyright society may
(i) issue licences under section 30 in respect of any rights under this Act;
(ii) collect fees in pursuance of such licences;
(iii) distribute such fees among owners of rights after making deductions for its
own expenses;
(iv) perform any other functions consistent with the provisions of section
Need for Copyright Societies :
Many authors and performers do not have the ability or means to pursue the
legal and administrative enforcement of their copyright and related rights,
especially given the increasingly global use of literary, music and performance
rights.
As a result, the establishment and enhancement of collective management
organizations (CMOs), or societies, is a growing and necessary trend in many
countries.
Features of Copyright Societies :
A copyright society is a registered collective administration society.
Such a society is formed by copyright owners as a group.
The minimum membership required for registration of a society is seven.
Ordinarily, only one society is registered to do business in respect of the same
class of work.
Functions of Copyright Societies :
A copyright society can issue or grant license in respect of any work in which

CONTENTS | Module-1 | Module-2 | Module-3 | Module-4 http://duralex.bhatt.net.in/


https://www.facebook.com/groups/Dharmo.Rakshati.Rakshitah/ Page - 81 of 196

copyright subsists or in respect of any other right given by the Copyright Act.
eg Certain rights of producers of sound recordings and broadcasting
organizations are sometimes managed collectively as well.
Basically the a copyright society performs the following functions :
i. Issue licences in respect of the rights administered by the society.
ii. Collect fees in pursuance of such licences.
iii. Distribute such fees among owners of copyright after making deductions for
the administrative expenses.
These societies can also provide their members with efficient administrative
support and legal expertise in,
for example, collecting, managing and disbursing royalties gained from the
national and international use of a work or performance.
(vii) Appeal provisions :

This is Module-2. Menu ---> CONTENTS | Module-1 | Module-2 | Module-3 | Module-4

Discuss : Rights conferred by copyright, (i) Assignment & Transmission, (ii)


relinquishment of copyright, (iii) Licence.
ANSWER :
Refer :
https://www.icsi.edu/docs/webmodules/Publications/9.4%20Intellectual
%20Property%20Rights.pdf
http://epgp.inflibnet.ac.in/epgpdata/uploads/epgp_content/law/08._intellectual_pr
operty_law/27._intellectual_property_assignment_and_licensing_/et/5795_et_27_
et.pdf
http://www.saprlaw.com/taxblog/copyright_final.pdf
Note : This answer is specific to Copyright only.
For general discussion on transfer, assignment, licensing of IPRs, see Module-1.
Outline :
(i) Assignment & Transmission
Mode of Assignment
Other provisions related to assignment
(ii) Relinquishment of copyright
(iii) Licence
Voluntary Licensing
Compulsory Licensing

CONTENTS | Module-1 | Module-2 | Module-3 | Module-4 http://duralex.bhatt.net.in/


https://www.facebook.com/groups/Dharmo.Rakshati.Rakshitah/ Page - 82 of 196

Compulsory Copyright Licence for Benefit of Disabled


Cancellation of Copyright License
Difference between Assignment and License
Intro :
Copyright is a bundle of rights. A copyrighted work may be used in numerous
ways.
For example, a novel may be translated, serialized, dramatized or turned into a
cinematograph film.
Each of these uses of the novel or any interest therein may be licensed to
different licensees.
The concept of copyright brings to ones mind associations with activities like book
publishing, music recording and publishing, film making and distribution, computer
software and advertising.
(i) Assignment & Transmission :
Assignment of copyright can be attempted in respect of
works that have already been created
and also for works that are not yet created.
That means future works can also be assigned.
But in respect of future works the assignment shall take effect only when the
work comes in existence.
There is no specific form or words prescribed for a copyright assignment.
However, a contract for copyright assignment of copyright has to be in writing.
The owner of the copyright in an existing work or the prospective owner of the
copyright in a future work may assign to any person the copyright.
Sec-18 of the Copyright Act provides for
assignment of copyright in an existing work as well as future work.
In both the cases an assignment may be made of the copyright
either wholly or partially
and generally or subject to limitations
and for the whole period of copyright or a part thereof.
However, in case of assignment of copyright in any future work, the assignment
has the real effect only when the work comes into existence.
Sections 17 and 18 of the Copyright Act, 1957 show where the copyright vests.
Sec-17 : If a work is done by an author for a consideration for a publisher,
the copyright in it would normally vest in the publisher subject to any contract
to the contrary, as is provided by Section 17 of the said Act.
Sec-18 : Under Sec-18, the copyright could be assigned, and if it is so done it

CONTENTS | Module-1 | Module-2 | Module-3 | Module-4 http://duralex.bhatt.net.in/


https://www.facebook.com/groups/Dharmo.Rakshati.Rakshitah/ Page - 83 of 196

would be vested in the purchaser.


(Khemraj Shrikrishnadass v. M/s Garg & Co. and Another AIR 1975 Del 130.)
Mode of Assignment :
Sec-19 of the Act provides that an assignment of copyright should be in writing
signed by the owner of the copyright.
Mere acceptance of remuneration or delivery of manuscript does not constitute
an assignment of copyright.
Oral assignment is invalid and it is impermissible in law.
Essentials of assignment :
The assignment of copyright should specify,
the assigned work,
rights including duration,
territorial extent of assignment
and the amount of royalty.
Other provisions related to assignment :
Copyright is different from the material object which is the subject of the
copyright.
ie the transfer of the material object does not necessarily involve a transfer of
the copyright.
In the absence of duration and territorial extent,
the assignment remains valid for a period of five years and within the territory
of India.
In case assignee does not exercise his rights within a period of one year from
the date of assignment,
the assignment in respect of such rights shall be deemed to have lapsed after
the expiry of said period, unless otherwise specified in the assignment.
The assignment of copyright in any work
contrary to the terms and conditions of the rights already assigned to a
copyright society in which the author of the work is a member
is void.
Setty v. Dr. Suryakantha U. Kamath K.A. Venugopala Setty v. Dr. Suryakantha
U. Kamath AIR 1992 Kar 1.
(ii) Relinquishment of copyright :
http://www.legalserviceindia.com/articles/copy_owner.htm
Sec-21 : The author of a work may relinquish
all or any of the rights comprised in the copyright in the work
by giving notice in the prescribed from to the Registrar of Copyrights

CONTENTS | Module-1 | Module-2 | Module-3 | Module-4 http://duralex.bhatt.net.in/


https://www.facebook.com/groups/Dharmo.Rakshati.Rakshitah/ Page - 84 of 196

and thereupon such rights shall, subject to the following conditions, cease to
exist from the date of the notice.
On receipt of a notice, the Registrar of Copyrights shall cause it to be published in
the Official Gazette and in such other manner as he may deem fit.
The relinquishment of all or any of the copyrights,
shall not affect any rights subsisting in favour of any person on the date of the
notice.
(iii) Licence :
Sec-30 : Licensing of copyright can be attempted in respect of
works that have already been created
and also for works that are not yet created.
But in respect of future works the license shall take effect only when the work
comes in existence.
There is no specific form or words prescribed in the Copyright Act for a copyright
license.
However, a contract for copyright license of copyright has to be in writing.
The copyright owner may grant a license and transfer some or all of his rights to
others to exploit his work for monetary benefits.
A license is different from an assignment,
licensee gets certain rights subject to the conditions specified in the license
agreement
but the ownership of those rights is not vested with him
while in case of an assignment
the assignee becomes the owner of the interest assigned to him.
A license may be exclusive or of non-exclusive type.
Chapter VI : Sections 30-32B : <read bare act for details>
Section 30 deals with licences by owners of copyright;
Section 31 provides for compulsory licence in works withheld from public;
Section 31A deals with compulsory licences in unpublished Indian works;
Section 31B deals with Compulsory Licence for the benefit of disabled;
Section 31C deals with statutory licence for cover versions;
Section 31D deals with statutory licence for broadcasting of literary and musical
works and sound recording;
Section 32 deals with licences to produce and publish translations;
Section 32A provides for licence to reproduce and publish works for certain
purposes;
Section 32B deals with termination of licences.

CONTENTS | Module-1 | Module-2 | Module-3 | Module-4 http://duralex.bhatt.net.in/


https://www.facebook.com/groups/Dharmo.Rakshati.Rakshitah/ Page - 85 of 196

Voluntary Licensing :
The owner of the Copyright in any existing or future work may grant any interest
in the work by way of license.
As regards the future works the license shall take effect only when the works
comes into existence.
For a license to be valid it must be in writing and signed by either the owner or
his duly authorized agent.
Where a person to whom a license relating to copyright in any future work dies
before the work comes into existence, his legal representative shall be entitled
to the benefit of the license.
A License Agreement generally contains the following particulars :
Identification of the work licensed
Duration of the license
Territorial extent of the license
Amount of royalty payable
Conditions relating to revision, extension and/or termination of license
Any dispute in respect of the license shall be settled by the Copyright Board or
by way of Arbitration.
Allowability of sub-licensing etc.
Compulsory Licensing :
Compulsory Licensing can be
invoked under certain circumstances with respect to both published works and
unpublished works.
obtained for the purposes of production and publication or translation of the
work.
The procedure for obtaining compulsory licensing with respect to the Indian
works and foreign works is different.
Compulsory licensing on Published Works :
With respect to the Indian works published or performed in public, compulsory
licenses can be obtained by making a complaint to the Copyright Board on the
ground that the owner has :
Refused to re-publish or allow the republication of the work
or has refused to allow the performance of the work in public
and by reason of such refusal the work is withheld from the public.
or Refused to allow the communication of the work to the public
by broadcast of the work or work in the sound recording
on such terms, which the complainant considers reasonable.

CONTENTS | Module-1 | Module-2 | Module-3 | Module-4 http://duralex.bhatt.net.in/


https://www.facebook.com/groups/Dharmo.Rakshati.Rakshitah/ Page - 86 of 196

or Refused to allow the performance of the work in pubic


and by reason of such refusal work is withheld from public;
Compulsory licensing on Unpublished Works :
Compulsory licenses can also be obtained with respect to the unpublished
works by making an application to the Copyright board in the following
circumstances :
Author is dead
Author is unknown
Author cannot be traced
Author cannot be found
Requirement : Before making an application in respect of an unpublished work
the applicant is required to publish his proposal in one issue of a daily
newspaper in the English language having circulation in major part of the
country
and also in one issue of any daily newspaper in that language.
Compulsory licensing with regard to copyright is mostly paper realism as books
or films are seldom licensed compulsorily.
Recently, the compulsory licensing was enforced in the field of Patents which
were not welcomed by the foreign counterparts as this would reduce their
income.
This law supported by conventions such as Vienna Conventions and TRIPS has to
be taken advantage to include the foreign works and the foreign books must be
made available to the general public at a lesser cost.
Sec-31(B) : Compulsory Copyright Licence for Benefit of Disabled :
(1) Any person working for the benefit of persons with disability on a profit basis
or for business may apply to the Copyright Board,
in such form and manner and accompanied by such fee as may be prescribed,
for a compulsory licence to publish any work in which copyright subsists for
the benefit of such persons
and the Copyright Board shall dispose of such application as expeditiously as
possible
(2) The Copyright Board may,
inquire to establish the credentials of the applicant and satisfy itself that the
application has been made in good faith.
(3) If the Copyright Board is satisfied,
after giving to the owners of rights in the work a reasonable opportunity of
being heard
and after holding such inquiry as it may deem necessary,

CONTENTS | Module-1 | Module-2 | Module-3 | Module-4 http://duralex.bhatt.net.in/


https://www.facebook.com/groups/Dharmo.Rakshati.Rakshitah/ Page - 87 of 196

that a compulsory licence needs to be issued to make the work available to


the disabled,
it may direct the Registrar of Copyrights to grant to the applicant such a
licence to publish the work.
(4) Every compulsory licence issued under this section shall specify
the means and format of publication,
the period during which the compulsory licence may be exercised
and in the case of issue of copies, the number of copies that may be issued
including the rate or royalty:
Provided that where the Copyright Board has issued such a compulsory licence it
may,
on a further application and after giving reasonable opportunity to the owners
of rights,
extend the period of such compulsory licence and allow the issue of more
copies as it may deem fit.
Cancellation of Copyright License :
The Copyright Board can cancel the license granted on any of the following
grounds :
The licensee has failed to produce and publish the translation of the work
within the specified period or the extended period.
The license was obtained by fraud or misrepresentation as to any essential
fact.
The licensee has contravened any of the terms and conditions of the license.
Difference between Assignment and License : Assignment of copy right and
copyright license are two forms of contract involved in the exploitation of copyright
work by a third party.
An assignment involves the disposal of the copyright. The author (assigner)
assigns the copyright to another person (assignee) or transfers the ownership of
the copyright.
License
usually involves licensing of some of the rights and not the whole.
can be exclusive or non exclusive.
is an authorization of an act without which authorization would be an
infringement.
Assignee will be the owner of the copyright as regard rights so assigned . The
owner will be the owner of the copyright of remaining rights. The assignment could
be for whole duration of the copyright or for a short duration.
In case of Licensing, the ownership shall always vest with the owner (Licensor).

CONTENTS | Module-1 | Module-2 | Module-3 | Module-4 http://duralex.bhatt.net.in/


https://www.facebook.com/groups/Dharmo.Rakshati.Rakshitah/ Page - 88 of 196

In case of assignment, the assignee himself can take action against third party.
The licensee can join with the owner of the copyright and as a party to the
infringement, and take an action for infringement against third party but a
bonafide purchaser in good faith and for consideration of the proprietors interest
without notice of previous licensee is unaffected by it.
Assignee is responsible to protect such rights which accrue as a result of
assignment by the previous owner of IPR.
The licensee himself is NOT responsible to protect copyright.
He can, at the best, sue the licensor for damages for breach of contract if the
latter does not protect his interest.
Failure of assignee to pay royalties, does NOT enable assignor to revoke
assignment.
A failure to pay royalties enables the licensor to revoke the license.
Under Section 30 of the Copyright Act, if the assignee dies before the work comes
into existence, the owner of the copyright has the power to assign his entire rights
or assign only some of the rights to legal representatives of the assignee.
if the licensee in the case of future work dies before the work comes in to
existence his legal representatives shall be entitled to such works, in the
absence of any provision to the contrary.

This is Module-2. Menu ---> CONTENTS | Module-1 | Module-2 | Module-3 | Module-4

Discuss the infringement of copyright under the Copyright Act, 1957. (Nov-2011)
Explain in detail the various kinds of infringement of copyrights and state the
remedies under the Copyright Law. (Apr-2016)
Write short note : Infringement of Literary Work (Dec-2015)
Discuss : (i) Acts not constituting infringement, (ii) Fair use provisions, (iii)
Piracy in internet.
Explain act not be infringement of Copyright under the Copyright Act, 1999. (Nov-
2012)
Explain : The remedies available against infringement of Copyright. (Nov-2012)
Explain under the Copyright Act : Civil remedies available against Infringement of
copyright (Oct-2013)
ANSWER :
Refer :
http://lawtimesjournal.in/infringement-of-copyright/
http://lawtimesjournal.in/copyright-and-the-internet/
http://www.saprlaw.com/taxblog/copyright_final.pdf

CONTENTS | Module-1 | Module-2 | Module-3 | Module-4 http://duralex.bhatt.net.in/


https://www.facebook.com/groups/Dharmo.Rakshati.Rakshitah/ Page - 89 of 196

Indian Copyright Act, 1957;


Copyright Rules, 1958;
Outline :
Infringement of copyright
Exceptions and limitations of copyright : Compulsory licensing
Contributory Infringement
Acts not constituting infringement
Copyright infringements while using Internet and database
Defences available in case of copyright infringement
Fair use provisions : Doctrine of fair dealing
Intro :
Copyright is a form of protection provided by the laws of India to the authors of
original works of authorship
including literary, dramatic, musical, artistic, architectural and certain other
intellectual works.
This protection is available to both published and unpublished works.
Copyright is created at the instance a work is fixed.
Material in the public domain is intellectual property that does not come under
copyright laws.
Nearly all work before the 20th C. is not copyrighted.
Infringement of copyright :
A copyright owner cannot enjoy his rights unless infringement of the same is
stringently dealt with by the Courts.
Copyright infringement is reproducing, distributing, displaying or performing a
work, or to make derivative works, without permission from the copyright holder.
It is often called "piracy".
Enforcement of copyright is generally the responsibility of the copyright holder.
Generally the copyright holder can only get money damages IF the owner registers
the copyright.
Plagiarism :
Plagiarism is the act of stealing and passing off the ideas, words, or other
intellectual property produced by another as ones own.
Plagiarism is also an infringement of copyright.
For example, using someone elses words in a research paper without citing the
source, is an act of plagiarism.
Exceptions and limitations of copyright : Compulsory licensing :
In Copyright Laws exceptions and limitations are

CONTENTS | Module-1 | Module-2 | Module-3 | Module-4 http://duralex.bhatt.net.in/


https://www.facebook.com/groups/Dharmo.Rakshati.Rakshitah/ Page - 90 of 196

provisions which, in public interest, permit the use of copyrighted works without
prior authorization or a license from its owner.
The principle of conditional grants to proprietary rights in any intellectual property
is to promote public interest. This is universally recognized and incorporated in
intellectual property system.
Protection and enforcement of intellectual property rights must :
be conducive to social and economic welfare;
safeguard an individuals fundamental rights; and
Promote commerce, competition and innovation.
Generally, exceptions and limitations to copyright are subject to a three-step test
set out in the Berne Convention for the Protection of Literary and Artistic Works.
Briefly stated, the Berne Convention provides that an exception or limitation to
copyright is permissible only if :
it covers special cases
it does not conflict with the normal exploitation of the work; and
it does not unreasonably prejudice the legitimate interests of the author.
Standard exceptions and limitations vary from country to country in their number
and scope.
Contributory Infringement :
Contributory infringement is where,
the copyrighted work is duplicated by another person without the consent of the
owner or existence of any lawful excuse by another with the aid of another.
This may be simply put a abatement to an offence.
For example,
a person has a Rs.1000 note and takes a color Xerox in a shop.
Here, the person is an infringer
and the Xerox shop is abettor who commits contributory infringement.
a book or compact disc is copyrighted which can be easily ascertained,
any person who helps in the offence of infringement like making duplicates
copies, translation, adaptation, communication to public etc, would amount to
contributory infringement.
In the A&M Records, Inc. v. Napster, Inc. 21,
the Defendant maintained a central unit which enabled two or more remote
computers to share all the music files in other system.
The defendant was held vicariously liable and for contributing to the
infringement.
In the case of Sony Corp. of America v. Universal City Studios, Inc. 22, the

CONTENTS | Module-1 | Module-2 | Module-3 | Module-4 http://duralex.bhatt.net.in/


https://www.facebook.com/groups/Dharmo.Rakshati.Rakshitah/ Page - 91 of 196

Supreme Court of the United States which ruled that


the making of individual copies of complete television shows for purposes of time
shifting does not constitute copyright infringement though the lower courts
considered it to be a contributory infringement, but it is only a fair use.
Acts not constituting infringement :
The act/ copying by defendant may not always amount to infringement.
Some examples of acts which do not constitute Infringement under the Copyright
Act are as follows :
Fair dealing such as criticisms, personal use, newspaper report, review etc.
Non-paying Audience
Research or Private study
Judicial Proceedings
Exclusive work of member of House of Legislature
Non-Corporate matter for institution purpose
Question Papers
With Consent
Issue being Current Topic such as economic, social, political Etc.
Made less than 3 copies
Available in Official Gazette.
Report of committee or Commission
After the expiry of Copyright.
Adaptation of Computer Program
Copyright infringements while using Internet and database :
<Read Copyright on computer programme, Internet and database in this doc>
Defences available in case of copyright infringement : Some of the defenses available
for the defendant are as follows :
Not Copyrightable
Fair Dealings
Education
Permitted acts
Consent
Public Interest
Libraries and Archieves
Computer programs
Adaptation
Statutory License etc.

CONTENTS | Module-1 | Module-2 | Module-3 | Module-4 http://duralex.bhatt.net.in/


https://www.facebook.com/groups/Dharmo.Rakshati.Rakshitah/ Page - 92 of 196

Fair use provisions : Doctrine of fair dealing :


The term fair dealing has not been defined in the Act.
It is a legal doctrine, which allows a person to make limited use of copyrighted
work without the permission of the owner.
Whether a persons use of copyright material is fair would depend entirely upon
the facts and circumstances of a given case.
The line between fair dealing and infringement is a thin one.
In India, there are no set guidelines that define the number of words or passages
that can be used without permission from the author.
Only the Court applying basic common sense can determine this.
It may however be said that the extracted portion should be such that it does not
affect the substantial interest of the Author.
Fair dealing is a significant limitation on the exclusive right of the copyright owner.
It has been interpreted by the courts on a number of occasions by judging the
economic impact it has on the copyright owner.
However, where the economic impact is not significant, the use may constitute
fair dealing.
The fair nature of the dealing depends on the following four factors :
the purpose of use
the nature of the work
the amount of the work used, and
the effect of use of the work on the original
In India the provisions of Section 52 of the Copyright Act, 1957 provide for certain
acts, which would not constitute an infringement of copyright.
Sec-52 : Certain acts not to be infringement of copyright
(1) The following acts shall not constitute an infringement of copyright, namely
(a) a fair dealing with any work, not being a computer programme, for the
purposes of-
(i) private or personal use, including research;
(ii) criticism or review, whether of that work or of any other work;
(iii) the reporting of current events and current affairs, including the
reporting of a lecture delivered in public.
(d) reproduction for the purpose of a judicial proceeding or of a report of a
judicial proceeding;
(e) reproduction or publication of a literary, dramatic, musical or artistic work
in any work prepared by the Secretariat of a Legislature exclusively for the use
of the members of that Legislature;

CONTENTS | Module-1 | Module-2 | Module-3 | Module-4 http://duralex.bhatt.net.in/


https://www.facebook.com/groups/Dharmo.Rakshati.Rakshitah/ Page - 93 of 196

(f) the reproduction of any literary, dramatic or musical work in a certified


copy made or supplied in accordance with any law for the time being in force;
(g) the reading or recitation in public of any reasonable extract from a
published literary or dramatic work;
(h) the publication in a collection, mainly composed of non-copyright matter,
bona fide intended for the use of educational institutions,
the making of sound if made by or with the license or consent of the owner of
the right in the work
In the case of Kartar Singh Giani v. Ladha Singh, the High court held that :
two points have been urged in connection with the meaning of the expression
fair, in fair dealing
(1) that in order to constitute unfairness there must be an intention to
compete and to derive profit from such competition and
(2) that unless the motive of the infringer were unfair in the sense of being
improper the dealing would be fair.
Fair dealing of internet/ electronic materials :
The Copyright, Designs and Patents Act 1988 permits individuals to make a
single copy of a reasonable proportion of literary, dramatic, musical and
artistic works for their own private study or research for non-commercial
purposes under the terms of fair dealing.
Although this concept was initially developed with printed materials in mind, it is
a useful rule of thumb when copying internet/ electronic materials which are not
otherwise governed by specific licences.
Case-laws :
In Hindustan Pencils Ltd v Alpna Cottage Industries the Copyright Board of Goa
held that
where the similarities between the artistic works of the parties are fundamental
and substantial in material aspects, it would amount to copyright violation and
the defendants copyright is liable to be expunged from the register of copyright.
However, where the first party himself is shown to have adopted or imitated a
trademark and copyright of a third party,
then Courts can resolutely decline to step in aid of this party because honesty
of action is the crux of the matter and Courts protection is extended only on
the principle that damage to a party which is the dishonest user of someone
elses IP.
The Board further referred the decision of the apex court in R.G. Anand v M/S
Delux Films where the Court observed :
in order to be actionable the copy must be a substantial and material one
which at once leads to the conclusion that the defendant is guilty of the act of

CONTENTS | Module-1 | Module-2 | Module-3 | Module-4 http://duralex.bhatt.net.in/


https://www.facebook.com/groups/Dharmo.Rakshati.Rakshitah/ Page - 94 of 196

piracy.
In Khajanchi Film Exchange v state of MP the appellants apprehending the violation
of their copyright in the film, prayed for the writ of Mandamus without first
exhausting the alternative remedy available under the Copyright Act. The Division
Bench of the Madhya Pradesh High Court Observed :
Petition was filed on mere apprehension that appellants would be deprived of
their rights which did not exist when claim for mandamus was made.
Mandamus can be granted only when default, commission, or omission takes
place which had not happened in this case.
In Jolen Inc v Shoban Lal Jain the Madras High Court held that
latches and acquiescence is a good defence to an action for copyright
infringement.
the plaintiff having allowed the defendant to carry on the business under the
trade name of the plaintiff for 7 years is prima facie guilty of acquiescence and it
cannot claim for relief of injunction against the defendant as the balance of
convenience is in favour of him.

This is Module-2. Menu ---> CONTENTS | Module-1 | Module-2 | Module-3 | Module-4

Explain : International Copyright. (Nov-2012)


Explain under the Copyright Act : International Copyright (Oct-2013)
ANSWER :
Refer :

This is Module-2. Menu ---> CONTENTS | Module-1 | Module-2 | Module-3 | Module-4

CONTENTS | Module-1 | Module-2 | Module-3 | Module-4 http://duralex.bhatt.net.in/


https://www.facebook.com/groups/Dharmo.Rakshati.Rakshitah/ Page - 95 of 196

Module-3 :
3) Intellectual Property Rights in Trademarks and Design :
3.1) The rationale of protection of trademarks as (a) an aspect of commercial
and (b) of consumer rights, Definition and concept of trademarks in goods
& services
3.2) Registration, Distinction between trademark and property mark The
doctrine of honest current user, Doctrine of deceptive similarity
Protection of well-known marks
3.3) Passing off and infringement of trademarks : (Definitions, Concept and
Distinction)
3.4) Standards of proof in passing off action
3.5) Remedies and Penalty provisions
3.6) Industrial Designs, Designs Act, 2000 : Authorities, Procedure for
registration of designs, Controller and Registrar : powers and duties,
Assignment and transmission of designs, Powers of the Central
Government, Copyright on registration of Industrial Designs : related
provisions, Patents to designs, Piracy of registered designs, Remedial
aspects, Appeal provisions,

This is Module-3. Menu ---> CONTENTS | Module-1 | Module-2 | Module-3 | Module-4

MODULE-3 QUESTIONS :

Explain under the Trademark Act, 1999 : Meaning of Trademark (Oct-2013)


Write short note : False Trade Mark (Dec-2015)
Write note : Trade description - False trade description. (Nov-2011)
Discuss under the Trade Mark Act, 1999 : Trade description. (Nov-2012)
Discuss : The rationale of protection of trademarks as (a) an aspect of
commercial and (b) of consumer rights.
State the definition and concept of trademarks in goods and services with case-
laws. (Apr-2016)
Discuss under the Trade Mark Act, 1999 : Collective marks. (Nov-2012, Dec-2015)
Discuss the registration under the Trade Marks Act, 1999. (Nov-2011)
Explain under the Trademark Act, 1999 : Application for registration (Oct-2013)
Explain under the Trademark Act, 1999 : Register of Trademark (Oct-2013)
How to oppose registration under Trade Marks Act ? (Dec-2015)
Discuss under the Trade Mark Act, 1999 : Effect of registration. (Nov-2012)

CONTENTS | Module-1 | Module-2 | Module-3 | Module-4 http://duralex.bhatt.net.in/


https://www.facebook.com/groups/Dharmo.Rakshati.Rakshitah/ Page - 96 of 196

Explain under the Trademark Act, 1999 : Alteration of registered Trademark (Oct-
2013)
Write note : Registered Users. (Nov-2011)
Explain under the Trademark Act, 1999 : Registration as registered user (Oct-
2013)
Discuss : Absolute and Relative grounds for refusal of registration .
Discuss under the Trade Mark Act, 1999 : Absolute grounds for refusal of
registration. (Nov-2012, Oct-2013)
Discuss under the Trade Mark Act, 1999 : Prohibition of registration of some
names. (Nov-2012, Oct-2013)
Discuss under the Trade Mark Act, 1999 : Relative grounds for refusal of
registration. (Nov-2012)
Discuss under the Trade Mark Act, 1999 : Deceptive similar. (Nov-2012, Nov-
2014, Dec-2015)
Discuss : Distinction between trademark and property mark.
Discuss under the Trade Mark Act, 1999 : Assignment of Trade mark. (Nov-2012,
Nov-2014, Dec-2015)
Discuss : Licensing of Trade Marks.
Explain : Infringement of Trademark (Oct-2013)
Discuss : (i) False Trade Mark, (ii) False Trade Description, (iii) Falsifying and
falsely applying trade marks.
State the definitions and concept of passing off and infringement of trademarks and
distinguish between passing off and infringement of trademarks. (Apr-2016)
Discuss : Standards of proof in passing off action.
Discuss the procedure for infringement of action and passing of action under the
Trade Marks Act, 1999. (Nov-2011)
Discuss : Remedies and Penalty provisions under the Trade Marks Act, 1999.
Discuss under the Design Act, 2000 : Industrial Design. (Nov-2012, Oct-2013, Nov-
2014)
Explain under the Design Act : Owner of Design (Oct-2013)
Explain the provisions relating to registration under the Designs Act of 2000. (Nov-
2011)
Write Short note : Procedure for registration of designs (Nov-2014)
Discuss under the Design Act, 2000 : Prohibition on registration of Designs.
(Nov-2012)
Discuss under the Design Act, 2000 : About cancellation of registration of Design.
(Nov-2012)
Discuss : Powers of the Central Government under Design Act, 2000.

CONTENTS | Module-1 | Module-2 | Module-3 | Module-4 http://duralex.bhatt.net.in/


https://www.facebook.com/groups/Dharmo.Rakshati.Rakshitah/ Page - 97 of 196

Discuss authorities under Designs Act 2000 and their powers and duties : (i)
Controller, (ii) Registrar.
Discuss Design Controller under the Designs Act, 2000. (Nov-2011, Nov-2012, Nov-
2014, Dec-2015)
Explain the controls over Registration of Designs. (Dec-2015)
Explain under the Design Act : Rectification of register (Oct-2013)
Discuss : Assignment, transmission and licensing of designs.
Discuss : Nature of Copyright in contrast to Design Right .
Discuss : Dart Industries Inc & Anr. vs Techno Plast & Ors.
Discuss : Samsonite Corp vs Vijay Sales.
Discuss : Patents to designs.
Discuss : (i) Piracy of registered designs, (ii) Remedial aspects, (iii) Appeal
provisions.
Discuss under the Design Act, 2000 : Piracy of Registered Designs. (Nov-2012, Oct-
2013, Nov-2014)

This is Module-3. Menu ---> CONTENTS | Module-1 | Module-2 | Module-3 | Module-4

MODULE-3 ANSWERS :

Explain under the Trademark Act, 1999 : Meaning of Trademark (Oct-2013)


Write short note : False Trade Mark (Dec-2015)
Write note : Trade description - False trade description. (Nov-2011)
Discuss under the Trade Mark Act, 1999 : Trade description. (Nov-2012)
ANSWER :
Refer :
http://www.ipindia.nic.in/writereaddata/Portal/IPOGuidelinesManuals/1_32_1_tmr-
draft-manual.pdf <--- Key document for Trade Marks.
https://en.wikipedia.org/wiki/Indian_trademark_law
http://www.wipo.int/edocs/pubdocs/en/intproperty/450/wipo_pub_450.pdf
http://www.lawyersclubindia.com/articles/Criminal-liability-of-a-party-making-
false-statement-8181.asp?
utm_source=newsletter&utm_content=news&utm_medium=email&utm_campaign
=nl_May
https://www.murgitroyd.com/heavens-to-murgitroyd/trade-mark-trademark-or-
trade-mark/

CONTENTS | Module-1 | Module-2 | Module-3 | Module-4 http://duralex.bhatt.net.in/


https://www.facebook.com/groups/Dharmo.Rakshati.Rakshitah/ Page - 98 of 196

http://www.majmudarindia.com/pdf/Limits%20or%20effects%20of%20registered
%20trademark.pdf
Outline : Basics of trademarks.
Intro
The Trade Mark Act 1999 :
What is a Trade Mark?
Benefits of trade marks
Nice Agreement : Classification of Trademarks
Madrid System for registration of Trademarks :
Scope of trademark protection :
Use of TM, SM and symbols :
Intro :
A trademark is a distinctive sign that identifies certain goods or services produced
or provided by an individual or a company.
Its origin dates back to ancient times when craftsmen reproduced their signatures,
or marks, on their artistic works or products of a functional or practical nature.
Over the years, these marks have evolved into todays system of trademark
registration and protection.
The system helps consumers to identify and purchase a product or service based
on whether its specific characteristics and quality as indicated by its unique
trademark meet their needs.
The Trade Mark Act 1999 :
Indian trademark law statutorily protects trademarks as per Trademark Act 1999
and also under the common law remedy of passing off.
Statutory protection of trademark is administered by the Controller General of
Patents, Designs and Trade Marks,
which is a government agency reporting to the Department of Industrial Policy
and Promotion (DIPP), under the Ministry of Commerce and Industry.
The law of trademark provides for,
the mechanism of registration, & the rights acquired by registration,
protection of trademark and prevention of fraudulent trademark.
modes of transfer and assignment of the rights,
nature of infringements,
penalties for such infringement
and remedies available to the owner in case of such infringement.
What is a trademark?
Term Trade means,

CONTENTS | Module-1 | Module-2 | Module-3 | Module-4 http://duralex.bhatt.net.in/


https://www.facebook.com/groups/Dharmo.Rakshati.Rakshitah/ Page - 99 of 196

exchange (something, eg product/ services) for something else, typically as a


commercial transaction.
Term Mark is not defined in the Indian TM Act 1999. However Section 68(1) of
the Trade Marks Act 1994 of England
defines a Mark as a device , brand , heading , label , ticket , name , signature ,
brand , letter , numeral or any combination thereof.
Thus, Trade Mark means,
a sign/symbol that depicts/identifies product or services that a business
enterprise is engaged in.
Trade Marks can be seen as serving two main purposes :
1. To protect business reputation and goodwill.
2. To protect consumers from deception, that is to prevent the buying public
purchasing inferior goods or services in the mistaken belief that they originate
from or are provided by another trader.
Essence of Sec-2(1)(zb) : Trade Marks Act 1999 :
trade mark means
a mark capable of being represented graphically
and which is capable of distinguishing the goods or services of one person
from those of others
and may include shape of goods, their packaging and combination of colours;
A trade mark can include,
a device, brand, heading, label, ticket, name, signature, word, letter, numeral,
shape of goods, packaging or combination of colors or any such combinations.
How to spell trade mark correctly?
Is it one word? Two words? Or something else?
The spelling of trade mark is completely down to which country you are from.
Trade Mark is typically the British spelling.
Trademark is the spelling used in America and used by the World Intellectual
Property Organisation (WIPO).
In India, it is spelled as Trade Mark,
as in Trade Marks Act, 1999.
<Also read False Trade Mark, False Trade Description, Falsifying and falsely applying
trade marks in this doc>
Benefits of trademarks :
Trademark protection ensures that the owners of marks have the exclusive right
to use them to identify goods or services,
or to authorize others to use them in return for payment.

CONTENTS | Module-1 | Module-2 | Module-3 | Module-4 http://duralex.bhatt.net.in/


https://www.facebook.com/groups/Dharmo.Rakshati.Rakshitah/ Page - 100 of 196

The period of protection varies, but a trademark can be renewed indefinitely upon
payment of the corresponding fees.
Trademark protection is legally enforced by courts that have the authority to stop
trademark infringement.
In a larger sense, trademarks promote initiative and enterprise worldwide by
rewarding their owners with recognition and financial profit.
Trade Mark protection discourages the unfair competitors ,
such as counterfeiters, to use similar distinctive signs to market inferior or
different products or services.
The trade mark system enables people with skill and enterprise to produce and
market goods and services in the fairest possible conditions, thereby facilitating
international trade.
Nice Agreement : Classification of Trademarks :
"Nice Agreement Concerning the International Classification of Goods and Services
for the Purposes of the Registration of Marks"
is a multilateral treaty (1957 Nice, France) administered by WIPO.
Classification (under Nice Agreement) of goods & service
is called Nice Classification
India follows Nice Classification of Trade Marks.
Accordingly, different goods and services have been classified into 45 Trademark
Classes (1 to 34 cover goods, and 35 to 45 cover services).
The Nice Agreement is open to states who are parties to the "Paris Convention for
the Protection of Industrial Property".
The idea behind this system is to specify and limit the extension of the intellectual
property right by determining which goods or services are covered by the mark,
and to unify classification systems around the world.
Madrid System for registration of Trademarks :
It is an international registration system for trademarks administered by WIPO.
It avoids need to register separate applications with each national or regional
office.
The system is governed by two treaties :
the Madrid Agreement Concerning the International Registration of Marks,
and the Madrid Protocol.
India joined the Madrid Protocol on July 08, 2013. Hence, Indian Trade Mark
registration also has effect in other countries of the Madrid Union.
The term of registration under the Madrid Protocol is 10 years from the date of
application. It is further renewable.
Scope of trademark protection :

CONTENTS | Module-1 | Module-2 | Module-3 | Module-4 http://duralex.bhatt.net.in/


https://www.facebook.com/groups/Dharmo.Rakshati.Rakshitah/ Page - 101 of 196

Almost all countries in the world register and protect trademarks.


Each national or regional office maintains a Register of Trademarks containing full
application information on all registrations and renewals, which facilitates
examination, search and potential opposition by third parties.
The effects of the registration are, however, limited to the country (or, in the case
of regional registration, countries) concerned.
Use of TM, SM and symbols :
A trademark is designated by the following ways :
TM (for an unregistered trade mark);
SM (for an unregistered service mark);
(for a registered trademark, or service mark)
The use of TM and SM symbols notifies the public that the company is claiming
exclusive ownership of the trademark/ servicemark.
Both TM and SM symbols can generally mean that,
one claims to be the proprietor of such a mark.
Often, TM & SM symbolds are used after filing of an application for trade mark/
service mark registration.
The symbol, can be used only AFTER the trademark is registered and the
registration certificate is issued.
Moreover, one may use the registration symbol only in connection with the goods
and/ or services in respect of which the trademark is registered.

This is Module-3. Menu ---> CONTENTS | Module-1 | Module-2 | Module-3 | Module-4


GO TO MODULE-3 QUESTIONS.
GO TO CONTENTS.

Discuss : The rationale of protection of trademarks as (a) an aspect of


commercial and (b) of consumer rights.
ANSWER :
Refer :

This is Module-3. Menu ---> CONTENTS | Module-1 | Module-2 | Module-3 | Module-4

State the definition and concept of trademarks in goods and services with case-
laws. (Apr-2016)

CONTENTS | Module-1 | Module-2 | Module-3 | Module-4 http://duralex.bhatt.net.in/


https://www.facebook.com/groups/Dharmo.Rakshati.Rakshitah/ Page - 102 of 196

ANSWER :
Refer :

This is Module-3. Menu ---> CONTENTS | Module-1 | Module-2 | Module-3 | Module-4

Discuss under the Trade Mark Act, 1999 : Collective marks. (Nov-2012, Dec-2015)
ANSWER :
Refer :

Sec-2(1)(g) collective mark means
a trade mark distinguishing the goods or services
of members of an association of persons (not being a partnership)
from those of others;
Collective marks, are owned by an association whose members use them to indicate
products with a certain level of quality and who agree to adhere to specific
requirements set by the association.
Such associations might represent, for example, accountants, engineers or
architects.

This is Module-3. Menu ---> CONTENTS | Module-1 | Module-2 | Module-3 | Module-4

Discuss the registration under the Trade Marks Act, 1999. (Nov-2011)
Explain under the Trademark Act, 1999 : Application for registration (Oct-2013)
Explain under the Trademark Act, 1999 : Register of Trademark (Oct-2013)
How to oppose registration under Trade Marks Act ? (Dec-2015)
Discuss under the Trade Mark Act, 1999 : Effect of registration. (Nov-2012)
Explain under the Trademark Act, 1999 : Alteration of registered Trademark (Oct-
2013)
Write note : Registered Users. (Nov-2011)
Explain under the Trademark Act, 1999 : Registration as registered user (Oct-
2013)
ANSWER :
Refer :
http://www.ablemindconsulting.com/registration_procedur.htm

CONTENTS | Module-1 | Module-2 | Module-3 | Module-4 http://duralex.bhatt.net.in/


https://www.facebook.com/groups/Dharmo.Rakshati.Rakshitah/ Page - 103 of 196

http://www.mondaq.com/india/x/127680/Trademark/Trademarks+Law+In+India+
Everything+You+Must+Know
http://www.ssrana.in/Intellectual%20Property/Trademarks/FAQ-on-Madrid-
Protocol-India.aspx
http://www.lrswami.com/page/what-is-a-trademark
http://www.legalserviceindia.com/trademarks-copyrights/trade
%20markmainpage.htm
Outline :
Need for Trade Mark registration :
What kinds of trademarks can be registered?
Characteristics of a good Trade Mark
Registration procedure :
Need for am agent/ attorney to file for a trademark registration :
Precautions before application :
Application :
Review by the Trade Marks Office :
Preliminary Approval, Publication, Show Cause hearing or Rejection :
Opposition to registration of a Trade Mark
Issue of registration certificate
Term of Trademark Registration :
Register of Trademarks : Rectification/ alteration/ cancellation of Trademark
Rectification/ alteration of Trademark
Cancellation/ revocation of a registered trade mark
Renewal of registration :
Registration as registered user :
Need for Trade Mark registration :
Registration of a Trademark is not mandatory in India but it is advisable to do so
The Trade Marks Act 1999 envisages registration of the Mark before affording
protection.
Section 27 of the Act states that,
no infringement action is maintainable against an unregistered trade mark.
Thus it is in the best interests of the owner/proprietor that the trade marks are
registered for their effective and profitable utilisation.
Benefits of a registered trademark :
Identifies the origin of goods & services.
Advertises goods & services.

CONTENTS | Module-1 | Module-2 | Module-3 | Module-4 http://duralex.bhatt.net.in/


https://www.facebook.com/groups/Dharmo.Rakshati.Rakshitah/ Page - 104 of 196

Guards the commercial goodwill of a trader.


Protects the innocent public from buying goods of second-rate quality.
What kinds of trademarks can be registered?
Trademarks may be,
one or a combination of words, letters and numerals.
they may also consist of drawings, symbols or three-dimensional signs, such as
the shape and packaging of goods.
Following marks can be registered under the Indian trademark law :-
Names
Invented / Coined Words
Numerals
Letters
2D/ 3D symbols
Devices
Combination of colors
Shape of goods
Slogans
Signature
Sound Marks
Smell Marks
Internet domain names
In some countries, non-traditional marks may be registered for distinguishing
features such as
holograms, motion, color and non-visible signs (sound, smell or taste).
Collective marks, owned by an association, can also be registered.
Certification Marks :
Certification marks are given for compliance with defined standards but are not
confined to any membership.
They may be granted to anyone who can certify that their products meet certain
established standards.
Some examples of recognized certification are,
the internationally accepted ISO 9000 quality standards,
and Eco-labels for products with reduced environmental impact.
Characteristics of a good Trade Mark :
Should be distinctive and should not have deceptive similarity.
Laudatory word/words such as best, perfect etc to be avoided

CONTENTS | Module-1 | Module-2 | Module-3 | Module-4 http://duralex.bhatt.net.in/


https://www.facebook.com/groups/Dharmo.Rakshati.Rakshitah/ Page - 105 of 196

It should have close relation with product/ service it is to represent.


It should be short
It should appeal to the eye/ ear.
If it is word mark,
it should be easy to pronounce and remember.
it should be easy to spell correctly and remember easily.
it should not be descriptive
In case of a symbol,
it should be capable of being described easily.
It should not belong to the class of marks prohibited for registration
It should satisfy the requirements of registration.
Registration procedure :
The registration process is the same for both trademarks and servicemarks.
The registration procedure in India is based on the 'first to file' system . It is
therefore important that the rights holder apply for the registration of its mark as
soon as possible.
The registration of a trademark in India typically takes about 2 to 3 years, subject
to the trademark not being opposed by a third party.
The Office of the Controller General of Patents, Trade Marks, Industrial Designs and
Geographical Indications is the appropriate office for filing of a trademark
application in India.
This office has branches in Mumbai, Delhi, Chennai, Ahmedabad and Kolkata.
A trademark application may be filed in any of these offices based on the territorial
jurisdiction.
Need for an agent/ attorney to file for a trademark registration :
The question whether an attorney is required to file for a trademark registration
is debatable
but if measured on a weighing scale, the side to appoint an attorney becomes
heavier.
Yes, hiring an attorney is NOT an essential requirement for filing a trademark ,
especially when all the laws and rules are properly laid down for a trademark
registration. Any layman who wants to register a trademark can apply himself.
But in reality, appointing an attorney will ease the process to register a
trademark.

CONTENTS | Module-1 | Module-2 | Module-3 | Module-4 http://duralex.bhatt.net.in/


https://www.facebook.com/groups/Dharmo.Rakshati.Rakshitah/ Page - 106 of 196

Precautions before application :


Before applying for Trade Mark, it is advisable to determine if the trademark is
eligible for registration.
Applicant shall ensure that,
any third party is NOT already using the trademark.
it is distinctive, so that consumers can distinguish it from trademarks
identifying other products, as well as identify a particular product with it.
it must neither mislead nor deceive customers nor violate public order or
morality. (deceptive similar)

CONTENTS | Module-1 | Module-2 | Module-3 | Module-4 http://duralex.bhatt.net.in/


https://www.facebook.com/groups/Dharmo.Rakshati.Rakshitah/ Page - 107 of 196

the rights applied for are NOT the same as, or similar to, rights already
granted to another trademark owner.
Thorough search of Trade Mark Register shall be carried, otherwise,
there may be objection by the Trade Mark Office, or
3rd parties (who claim similar or identical rights) may oppose the application
A trademark search can be conducted in India on the Indian Trademark
Registrys Website at : http://www.ipindia.nic.in/
Application :
An application for registration of a trademark must be filed with the appropriate
national or regional trademark office.
The application must contain a clear reproduction of the sign filed for
registration, including any colors, forms or three-dimensional features.
It must also contain a list of the goods or services to which the sign would apply.
Review by the Trade Marks Office :
After the application has been filed,
the Trade Marks Office reviews it to ensure that it is complete in all respects
and thereafter allots an application number to the applications.
If the trademark is registered, the application number becomes the registration
number.
Preliminary Approval, Publication, Show Cause hearing or Rejection :
During the process of examination the Trade Marks Office determines if the
trademark is barred for registration,
either under absolute grounds for refusal and/or relative grounds for refusal as
prescribed in The Trade Marks Act, 1999.
Accordingly, they issue an examination report and
the Applicant must respond to the objections that have been raised in the
examination report within a period of one month from the issuance of the
examination report.
Thereafter and based on the response to the examination report that has been
filed by the Applicant,
the Registrar of Trade Marks determines if the application should be,
refused,
accepted for advertisement,
accepted subject to certain limitations
or put up for a "show cause" hearing,
On hearing, the application might be accepted, rejected or accepted subject to
certain limitations.

CONTENTS | Module-1 | Module-2 | Module-3 | Module-4 http://duralex.bhatt.net.in/


https://www.facebook.com/groups/Dharmo.Rakshati.Rakshitah/ Page - 108 of 196

Should the application be rejected the Applicant can approach the Intellectual
Property Appellate Board to appeal the order of the Registrar of Trade Marks.
Opposition to registration of a Trade Mark :
Should a rights holder of an existing trademark come across,
a trademark which is similar (or deceptively similar) to their mark,
or a trademark published in the Trade Marks Journal, which is similar (or
deceptively similar) to their mark,
they may oppose the impugned mark within three months of the publication in
the journal.
The opposition proceedings in India maybe initiated by any party in order to
maintain the purity of the Register of Trade Marks, regardless of whether they
have any locus standi.
Therefore, any third party who is of the opinion that the advertised mark should
not be allowed to register, can initiate an opposition proceeding by filing a Notice
of Opposition in the appropriate office of the Trade Marks Office.
The Notice of Opposition should be sent to the Trade Marks Office in triplicate .
Issue of registration certificate :
Within three months of the publication of the trademark in the Trade Marks
Journal,
should the trademark not be opposed by a third party,
it will proceed for registration
and the Trade Marks Registry will accordingly issue a registration certificate.
Term of Trademark Registration :
Trademark protection in India is perpetual subject to renewal of the registration
after every 10 years.
The application for renewal can be filed six months before the expiry of the
validity period of the trademark.
Register of Trademarks : Rectification/ alteration/ cancellation of Trademark :
There is a common misnomer that once a trademark is registered it is safe and
protected for good.
There are certain limits on effect of registered Trade Marks as envisaged by the
provisions of Chapter IV of the TM Act 1999.
In case a registered trademark owned by a third party infringes the rights of the
rights holder,
the rights holder can initiate (i) rectification, or (ii) cancellation proceedings
against the registered owner.
Rectification/ alteration of Trademark :
An aggrieved person may file an application before the Registrar of Trademarks

CONTENTS | Module-1 | Module-2 | Module-3 | Module-4 http://duralex.bhatt.net.in/


https://www.facebook.com/groups/Dharmo.Rakshati.Rakshitah/ Page - 109 of 196

or to the Intellectual Property Appellate Board (IPAB),


for varying the registration of the trademark
on the ground of any contravention or failure to observe a condition entered
on the Register in relation thereto.
Cancellation/ revocation of a registered trade mark :
1. An aggrieved person may file an application before the Registrar of
Trademarks or to the Intellectual Property Appellate Board (IPAB),
for cancellation of the registration of the trademark
on the ground of any contravention or failure to observe a condition entered
on the Register in relation thereto.
2. The application for cancellation can also be filed for removal of an entry made
in Register,
without sufficient cause
or wrongly remaining on the Register
and for correction of any error or defect in any entry in the Register.
3. A registered trade mark can be cancelled/ revoked by the Trade Mark Office
for following reasons :
(a) Non Use :
A registered trade mark can be revoked if the genuine use has been
suspended for an uninterrupted period of five years and there are no proper
reasons for non use.
Such marks are usually referred to as ghost marks .
(b) It has become a generic name :
A registered Trade Mark can be revoked because of the acts or the inactivity
of the proprietor which has resulted in the mark becoming a common name
in trade for a product or service for which it is registered.
eg In Re Xerox Case , the trademark XEROX became a generic name and
substituted the word Photocopy . It became such a part of the common
public domain that instead of saying photocopying people started using
Xeroxing and thereby the mark lost its distinctiveness and was subsequently
revoked.
(c) It is Misleading :
A registered trade mark can be revoked because of misleading nature of the
registered trade mark (particularly in respect of the nature, quality or
geographical origin of the goods or services in question).
(d) Honest Concurrent Use :
If two users had been using the same trade mark in the course of their
business for a period of five years then no infringement action will be

CONTENTS | Module-1 | Module-2 | Module-3 | Module-4 http://duralex.bhatt.net.in/


https://www.facebook.com/groups/Dharmo.Rakshati.Rakshitah/ Page - 110 of 196

maintainable on the unregistered user by the registered user.


Thus, registration of a trade mark is not the end all and be all of the matter.
There are certain limits to registration, and in order to safeguard the product from
falling within the ambit of those limits , an efficient and proactive trade mark
management is an absolute must.
Renewal of registration :
The trademark is initially registered for a period of 10 years, which is calculated
from the date of filing of the application
and in case of convention application, from the date of priority.
The registration is required to be renewed within 6 months before the date of
expiry of the registration, i.e., 10 years from the date of the application or
subsequent renewals.
The failure in renewing the trademark within the stipulated period of time and a
grace period of maximum 1 year granted for restoration of the trademark,
automatically leads to removal of the trademark from the Register of
Trademarks.
Registration as registered user :

This is Module-3. Menu ---> CONTENTS | Module-1 | Module-2 | Module-3 | Module-4

Discuss : Absolute and Relative grounds for refusal of registration.


Discuss under the Trade Mark Act, 1999 : Absolute grounds for refusal of
registration. (Nov-2012, Oct-2013)
Discuss under the Trade Mark Act, 1999 : Prohibition of registration of some
names. (Nov-2012, Oct-2013)
Discuss under the Trade Mark Act, 1999 : Relative grounds for refusal of
registration. (Nov-2012)
Discuss under the Trade Mark Act, 1999 : Deceptive similar. (Nov-2012, Nov-
2014, Dec-2015)
ANSWER :
Refer :
https://www.lexology.com/library/detail.aspx?g=26e3d590-7347-4e7b-8d5d-
d03f3228d4da
https://spicyip.com/2015/12/supreme-court-says-no-trademark-registration-for-
names-of-holy-books.html
Outline :

CONTENTS | Module-1 | Module-2 | Module-3 | Module-4 http://duralex.bhatt.net.in/


https://www.facebook.com/groups/Dharmo.Rakshati.Rakshitah/ Page - 111 of 196

A Mark shall not be registered as a Trade Mark if it


1. Absolute grounds / prohibition :
is prohibited under the Emblems and Names Act, 1950
or hurts religious susceptibilities of class/ section of citizens of India; or
or comprises/contains scandalous/obscene matter which is against the
morality of the public;
or is prohibited under Sec-9 and Sec-13 or the TM Act 1999.
2. Relative grounds : (honest concurrent use)
causes confusion in the public, in view of an earlier mark;
or is deceptively similar to an earlier mark;
3. Other grounds :
there is passing off of Copyright Infringement
or it has connection with a Living or Recently Deceased Person
1. Absolute grounds for refusal of Trade Mark :
Note :
Prohibition of registration of some names
is same as Absolute grounds for refusal of Trade Mark.
A Mark shall not be registered as a Trade Mark if it
(A) is prohibited under the Emblems and Names Act, 1950
(B) hurts religious susceptibilities of class/ section of citizens of India;
(C) comprises/contains scandalous/obscene matter which is against the morality of
the public;
(D) is prohibited under Sec-9 and Sec-13 or the TM Act 1999.
Sec-9 and Sec-13 provides for Prohibition of registration of some names or in
other words, Absolute grounds for refusal of Trade Mark.
Sec-9 : Such trademarks which are :
Incapable of distinguishing the goods/services of the applicant with those of
others;
or may designate kind, quality, purpose, value, geographical origin;
or have been commonly used in the current language or established
practices of trade;
shall not be registered.
Exception :
If before the date of application, the mark has acquired distinctiveness by use
or it is a well-known mark, it cannot be denied registration.
3-Dimensional Mark or Shape of Goods may be registered as a trademark.

CONTENTS | Module-1 | Module-2 | Module-3 | Module-4 http://duralex.bhatt.net.in/


https://www.facebook.com/groups/Dharmo.Rakshati.Rakshitah/ Page - 112 of 196

However, a Mark shall not be registered as a Trade Mark if :


it consists exclusively of the shape of goods itself or the shape of goods
necessary to obtain a technical result; or
the shape which adds substantial value to the goods
ie the shape should have visual appeal to add value and not be of a functional
nature to fall within the purview of this provision.
Explanation :
The nature of goods or services in relation to which trademark is used or
proposed to be used,
shall not be a ground of refusal of registration.
Sec-13 : Chemical element/compound :
No word,
which is the name of a chemical element/ compound (not mixture)
or which is declared by the World Health Organization and notified by the
Registrar of Trade Marks as an international non-proprietary name /
deceptively similar to such names
shall be registered as a trademark.
Even if the aforementioned word has acquired distinctiveness as a trademark it
still cannot be registered as a trademark.
2. Relative grounds for refusal of Trade Mark : (honest concurrent use)
A Mark shall not be registered as a Trade Mark if it
(honest concurrent use) causes confusion in the public, in view of an earlier
mark;
(honest concurrent use) or is deceptively similar to an earlier mark;
Essence of Sec-11 : Honest concurrent use,
A Trade Mark shall not be registered if :
(1) there is a likelihood of confusion for the public,
because the trademark being applied for is identical with an earlier
trademark
and the goods/ services of the two marks are similar; or
(2) the trademark being applied for is similar to an earlier trademark
and the goods/ services of the two marks are identical.
Sec-2(1)(h) deceptively similar
A mark shall be deemed to be deceptively similar to another mark if it so nearly
resembles that other mark as to be likely to deceive or cause confusion;
In all Sec-11 of the TM Act 1999 lists 11 relative grounds for refusal of a TM.
Some of these relative grounds are discussed below.

CONTENTS | Module-1 | Module-2 | Module-3 | Module-4 http://duralex.bhatt.net.in/


https://www.facebook.com/groups/Dharmo.Rakshati.Rakshitah/ Page - 113 of 196

Sec-11 : Honest concurrent use :


(1) Causes confusion :
Save as provided in section 12, trade mark shall not be registered if, because
of-
(a) its identity with an earlier trade mark and similarly of goods or services
covered by the trade mark, or
(b) its similarity to an earlier trade mark and the identity or similarity of
the goods of services covered by the trade mark.
there exists a likelihood of confusion on the part of the public, which includes
the likelihood of association with the earlier trade mark.
(2) Deceptive similar :
A trade mark which
(a) is identical with or similar to an earlier trade mark, and
(b) is to be registered for goods or services which are not similar to those
for which the earlier trade mark is registered in the name of a different
proprietor.
Shall not be registered if
the earlier trade mark is a well-known trade mark in India
and use of the later mark without due cause would take unfair advantage of
or be detrimental to the distinctive character or repute of the earlier trade
mark.
Meaning of earlier :
Sec-11(4) : The term Earlier Trade Mark,
means a Registered mark or a convention application (from a citizen of a
country/group of countries with which India has a treaty/ agreement)
for which the date of application was earlier than the trademark sought to be
registered.
3. Other grounds for refusal of a Trade Mark :
A Mark shall not be registered as a Trade Mark if it
(A) there is passing off of Copyright Infringement, or
(B) it has connection with a Living or Recently Deceased Person.
(A) Passing off of Copyright Infringement : <Also discussed elsewhere in this doc>
Essence of Sec-11 (3) :
A trademark shall not be registered,
if its use is liable to be prevented by virtue of,
Law of Passing off,
or Law of Copyright.

CONTENTS | Module-1 | Module-2 | Module-3 | Module-4 http://duralex.bhatt.net.in/


https://www.facebook.com/groups/Dharmo.Rakshati.Rakshitah/ Page - 114 of 196

(B) Connection with a Living or Recently Deceased Person :


Sec-14 : If an application for registration is in respect of a trademark,
which falsely suggests a connection with a living person or a person who died
within 20 years prior to the date of application,
then the Trade Mark Registrar may, prior to proceeding with the application,
require the applicant to furnish consent in writing of such living person; or
legal representative of the deceased person.
The Registrar may refuse to proceed with the application if such consent is not
furnished.

This is Module-3. Menu ---> CONTENTS | Module-1 | Module-2 | Module-3 | Module-4

Discuss : Distinction between trademark and property mark.


ANSWER :
Refer :
http://rajdeepandjoyeeta.com/trademark/
http://www.lawweb.in/2016/07/what-is-difference-between-trade-mark_14.html
https://www.legalcrystal.com/cases/search/name:trade-mark-and-property-mark
The concept of a trade mark is distinct from that of a property mark.
The difference between a Trade Mark and a Property Mark has been elaborated by the
Supreme Court in the case of Sumat Prasad Jain Vs. Sheojanam Prasad (Dead) & Ors.
Facts :
Respondent evolved a formula of manufacturing a scent Basant Bahar, which
became very popular.
He applied for registration of the Trade Mark, but the application was not
granted due to certain technical defects.
The appellant had also put up sale of a scent prepared by him and sold it as
Pushp Raj Scent.
This scent did not become popular and so he started putting up his said scent
under the name of Basant Bahar in receptacles, similar to those of the
Respondent, except the name of the manufacturer.
Legal battle :
The Trial Court convicted the Appellant under s. 482[6] and s. 486[7] I.P.C.
On appeal, the Addl. Session Judge set aside the said order of conviction and
sentence.
The complainant filed an appeal in the High Court.
Pending the appeal, the complainant died.

CONTENTS | Module-1 | Module-2 | Module-3 | Module-4 http://duralex.bhatt.net.in/


https://www.facebook.com/groups/Dharmo.Rakshati.Rakshitah/ Page - 115 of 196

The High Court allowed the appeal on the ground that,


though in the complaint, the Complainant has used expressions, such as,
Trade Mark, counterfeiting his Trade Mark etc,
in substance the complainant averred counterfeiting of property mark and
accordingly set aside the order of acquittal.
The Supreme Court held that (A) the concept of Trade Mark is distinct from that
of a property mark.
A Trade Mark means
a mark used in relation to goods
for the purpose of indicating a connection between the goods and some
person having the right as proprietor to use that mark.
The function of a Trade Mark is
to give an indication to the purchaser as to the manufacture or quality of
the goods,
to give an indication to his eye of the trade source from which the goods
come or the trade hands through which they passed on their way to the
market.
On the other hand, a property mark, defined by S.479 of the Penal Code,
means
a mark used for denoting that a movable property belong to a particular
person.
Thus, the distinction between a Trade Mark and a property mark is that
whereas the former denotes the manufacture or quality of the goods to
which it is attached,
the latter denotes the ownership in them.
In other words a Trade Mark concerns the goods themselves,
while a property mark concerns the proprietor.
A property mark attached to the movable property of a person remains even if
part of such property goes out of his hands and ceases to be his.
The Supreme Court held that (B) to succeed on the charge under s. 482 and s.
486 of IPC, the complainant had to establish that
the appellant (original accused) marked the scent manufactured and sold by
him or the packets and receptacles containing such scent or used packets or
receptacles bearing that mark
and that he did so in a manner calculated to cause it to be believed that the
goods so marketed or scent contained in the, packets so marked belonged
to the complainant.
For the purpose of s.486, he had further to establish that the appellant had

CONTENTS | Module-1 | Module-2 | Module-3 | Module-4 http://duralex.bhatt.net.in/


https://www.facebook.com/groups/Dharmo.Rakshati.Rakshitah/ Page - 116 of 196

sold or exposed for sale or had in his possession for sale, goods having a mark
calculated to cause it to be believed that the scent, was the scent
manufactured by and belonging to the (original) complainant.
Coming to the facts of the case, it held that
the name Basant Bahar with the same picture, the same inscriptions, and
the same receptacles, was the property mark denoting that the scent in
question was the one manufactured and belonging to the complainant.
From the finding arrived at by the Trial Court, it must follow that
the appellant marked his scent aid the packets in which it was packed with
the same name, the same picture and the same inscriptions
with the intention of causing it to be believed that the scent so marked, or
the scent contained in the said packets, was the one manufactured by and
sold in the market by the complainant.
The appellant (original accused) thus committed the offence of both
using a false property mark
and of selling goods marked with a counterfeit property mark.
The High Court was right in setting aside the order of acquittal passed by the
Additional Sessions Judge and in restoring the order of conviction and sentence
passed by the Trail Court.

This is Module-3. Menu ---> CONTENTS | Module-1 | Module-2 | Module-3 | Module-4

Discuss under the Trade Mark Act, 1999 : Assignment of Trade mark. (Nov-2012,
Nov-2014, Dec-2015)
Discuss : Licensing of Trade Marks.
ANSWER :
Refer :
http://epgp.inflibnet.ac.in/epgpdata/uploads/epgp_content/law/08._intellectual_pr
operty_law/27._intellectual_property_assignment_and_licensing_/et/5795_et_27_
et.pdf
Note : This answer is specific to Trade Marks only.
For general discussion on transfer, assignment, licensing of IPRs, see Module-1.
Outline :
Assignment of Trade Mark :
Licensing of Trade Mark :
Assignment of Trade Mark :
<work on this>

CONTENTS | Module-1 | Module-2 | Module-3 | Module-4 http://duralex.bhatt.net.in/


https://www.facebook.com/groups/Dharmo.Rakshati.Rakshitah/ Page - 117 of 196

Licensing of Trade Mark :


In trade mark licensing,
the owner of a mark gives permission to place his mark on manufactured goods
or services belonging to someone else.
The Trade Marks Act contains elaborate provisions as to licensing of registered
trade mark,
while licensing of unregistered trade marks is governed by common law.
Where the license contract is registered under the Trade Marks Act,
the licensee is known as registered user,
registration of a licensee as a registered user is NOT mandatory.
Trade mark licensing is based on the legal fiction that
use of a trade mark by a licensee will be a deemed to be
use by the trade mark owner and for the trade mark owners sole benefit.
Therefore, no application can be filed by anyone for revocation of the trade mark
on the grounds of non-use, if the licensee has used the trade mark in that period.
Further all goodwill generated by the licensee around the licensed trade mark shall
belong to the registered proprietor.
Quality control is a special feature of trade mark licensing which distinguishes it
from other forms of IP licensing.
Quality control by the proprietor of trade mark over the use of the licensed mark
is an independent requirement both under common law and statutory law as to
trade marks.
Under this requirement the licensor is required to control the quality of the
products/ services of the licensee.
Such quality control could be achieved in the following manners :
By specification of formulae, standards, methods, directions, instructions, etc.
to be followed by the licensee
By inspection of manufacturing processes, facilities, products, packaging,
services, advertising, etc. of the licensee
By analyzing the samples of the licensees products
The law does not demand goods of a particular quality from a licensor, but it
certainly requires that the goods of the owners licensee must match with the
quality of goods produced by the owner himself.
Trade mark licensing is the basis of numerous business practices and in many of
these practices licensing of trade marks is hybridized with licensing of other tools.
The business practices potentially involving a trade mark license are
franchising, merchandising, technology transfer and software licensing.
Out of these, trade mark licensing can blend, almost imperceptibly, into franchising

CONTENTS | Module-1 | Module-2 | Module-3 | Module-4 http://duralex.bhatt.net.in/


https://www.facebook.com/groups/Dharmo.Rakshati.Rakshitah/ Page - 118 of 196

which essentially combines trade mark license with the provision of marketing or
promotional assistance and controls over the manufacturing methods employed by
the franchisee.

This is Module-3. Menu ---> CONTENTS | Module-1 | Module-2 | Module-3 | Module-4

Explain : Infringement of Trademark (Oct-2013)


Discuss : (i) False Trade Mark : Falsifying and falsely applying trade marks, (ii)
False Trade Description.
State the definitions and concept of passing off and infringement of trademarks and
distinguish between passing off and infringement of trademarks. (Apr-2016)
Discuss : Standards of proof in passing off action.
Discuss the procedure for infringement of action and passing of action under the
Trade Marks Act, 1999. (Nov-2011)
ANSWER :
Refer :
http://thedemandingmistress.blogspot.in/2014/08/distinction-between-falsification-
of.html
http://www.lawyersclubindia.com/articles/Criminal-liability-of-a-party-making-
false-statement-8181.asp?
utm_source=newsletter&utm_content=news&utm_medium=email&utm_campaign
=nl_May
http://www.mondaq.com/india/x/127680/Trademark/Trademarks+Law+In+India+
Everything+You+Must+Know
http://www.nishithdesai.com/fileadmin/user_upload/pdfs/Research
%20Papers/Intellectual_Property_Law_in_India.pdf
http://www.ssrana.in/Intellectual%20Property/Infringement/Difference-between-
Infringement-and-Passing-Off-in-India.aspx
Outline :
What is a trademark?
What is trade description?
Infringement of registered trade marks
Meaning of applying trade marks and trade descriptions :
False Trade Mark : Falsifying and falsely applying trade marks :
False Trade Description :
Scope of Sec-29 and Sec-102 :

CONTENTS | Module-1 | Module-2 | Module-3 | Module-4 http://duralex.bhatt.net.in/


https://www.facebook.com/groups/Dharmo.Rakshati.Rakshitah/ Page - 119 of 196

Burden of proof :
Passing-off :
Standards of proof in passing off action.
Difference between passing-off and infringement of trade mark :
Intro :
Chapter XII of the Trade Marks Act 1999 deals with Offences and Penalties.
Sec-29 defines infringement of registered trade marks.
Sec-102 defines falsification of a trade mark and falsely applying a trade mark.
What is a trademark?
Essence of Sec-2(1)(zb) : Trade Marks Act 1999 :
trade mark means
a mark capable of being represented graphically
and which is capable of distinguishing the goods or services of one person
from those of others
and may include shape of goods, their packaging and combination of colours;
A trade mark can include,
a device, brand, heading, label, ticket, name, signature, word, letter, numeral,
shape of goods, packaging or combination of colors or any such combinations.
What is trade description?
Sec-2(1)(za) : Trade Marks Act 1999 :
"trade description" means any description, statement or other indication, direct
or indirect,-
...
(iv) as to the place or country in which any goods or services were made,
produced or provided, or
(v) as to the indication of the identity of the manufacturer or of the person
providing the services of the person for whom the goods are manufactured or
services are provided, or
(vi) as to the mode of manufacturing any goods or providing services, or
(vii) as to the material of which any goods are composed, or
...
Infringement of registered trade marks :
In many countries, a trademark receives protection without registration.
In India, registration of a Trademark is not mandatory, but it is advisable.
The Trade Marks Act 1999 envisages registration of the Mark before affording
protection.

CONTENTS | Module-1 | Module-2 | Module-3 | Module-4 http://duralex.bhatt.net.in/


https://www.facebook.com/groups/Dharmo.Rakshati.Rakshitah/ Page - 120 of 196

Section 27 of the Act states that,


no infringement action is maintainable against an unregistered trade mark.
When does the infringement occur ?
Infringement of registered trade mark may occur in following conditions :
1. as provided under Sec-29 : Infringement of registered trade marks
2. as provided under Sec-11 : Relative grounds for refusal of registration.
1. Infringement of registered trade marks u/s 29 :
Sec-29 :
Trademark infringement occurs when
one party uses a trademark that is identical or confusingly similar to a
trademark owned by another party,
in relation to products or services which are identical or similar to the
products or services of the other party.
in such manner as to render the use of the mark likely to be taken as
being used as a trade mark.
THUS, essential ingredients for initiation of an infringement action are,
the allegedly infringing mark must be either identical or deceptively similar
to the registered trademark;
the goods / services in relation to which the allegedly infringing mark is
used must be specifically covered by the registration of the registered
trademark;
the use of the allegedly infringing mark must be in the course of trade; and
the use must be in such a manner as to render the use likely to be taken as
being used as a trademark.
Relative grounds for refusal of registration u/s 11 :
Sec-11(1) : A registered trademark is also infringed by use of a mark when,
because of
marks identity with registered trademark and similarity with goods /
services covered by registration; or
marks similarity with registered trademark and identity with goods /
services covered by registration; or
marks identity with registered trademark and identity with goods /
services covered by registration
it is likely to cause confusion on the part of the public (in case 3 above,
confusion is presumed), or which is likely to have an association with the
registered trademark.
Sec-11(2) : An identical or similar mark is used with respect to goods or
services which are not similar to those for which a registered trademark is

CONTENTS | Module-1 | Module-2 | Module-3 | Module-4 http://duralex.bhatt.net.in/


https://www.facebook.com/groups/Dharmo.Rakshati.Rakshitah/ Page - 121 of 196

registered, such use amounts to infringement


if a registered trademark has reputation in India and the use of the mark
without due cause takes unfair advantage of or is detrimental to the
distinctive character or repute of the registered trademark
Who can sue for Infringement ?
The registered proprietor, his heirs and the registered user(s) can sue for
infringement.
An assignee of a registered trademark can also sue for infringement.
A passing off suit can be converted into a combined action of infringement and
passing off,
if the registration of the trademark is obtained before the final hearing of the
passing off suit.
In India, infringement of trade mark can be addressed under civil laws as well as
under criminal laws.
Significantly, infringement of a trademark is a cognizable offence and criminal
proceedings can be initiated against the infringers.
Meaning of applying trade marks and trade descriptions :
Sec-101(1) : A person shall be deemed to apply a trade mark or mark or trade
description to goods or services who
(a) applies it to the goods themselves or use it in relation to services, or
(b) applies it to any package in or with which the goods are sold, or
(c) places, encloses or annexes any good which are sold, in or with any package
to which a trade mark or mark or trade description has been applied, or
(d) . . . (e) . . .
False Trade Mark : Falsifying and falsely applying trade marks :
Sec-102 of the TM Act 1999 :
(1) A person shall be deemed to falsify a trade mark who, either ,
(a) without the assent of the proprietor of the trade mark makes that trade
mark or a deceptively similar mark; or
(b) falsifies any genuine trade mark, whether by alteration, addition,
effacement or otherwise.
(2) A person shall be deemed to falsely apply to goods or services a trade
mark who, without the assent of the proprietor of the trade mark ,
(a) applies such trade mark or a deceptively similar mark to goods or services
or any package containing goods;
(b) . . .
(3) Any trade mark
falsified as mentioned in (1) above,

CONTENTS | Module-1 | Module-2 | Module-3 | Module-4 http://duralex.bhatt.net.in/


https://www.facebook.com/groups/Dharmo.Rakshati.Rakshitah/ Page - 122 of 196

or falsely applied as mentioned in (2) above,


is a false trade mark.
Thus,, the act of manufacture of the mark without the consent of the proprietor of
the trade mark is deemed as falsification of the trade mark.
False Trade Description :
Sec-2(1)(i) of the Trade Marks Act 1999 :
false trade description means
(I) a trade description which is untrue or misleading in a material respect as
regards the goods or services to which it is applied; or
(II) . . . (III) . . . (IV) . . . (V) . . .
Burden of proof :
Sec-102(4) :
In any prosecution for falsifying a trade mark or falsely applying a trade mark to
goods or services,
the burden of proving the assent of the proprietor shall lie on the accused.
Scope of Sec-29 and Sec-102 :
It must be noted that the prohibition under Section 102 is not restricted to the
class of goods or services in connection with which the proprietor of the genuine
trade mark uses the trade mark.
In other words, a person who applies/uses a false trade mark in any class of
goods or services could attract the prohibition of Section 102.
To this extent, the scope of the bar under Section 102 is wider than Section 29.
Passing-off :
For an action of passing off, registration of a trademark is irrelevant. It is based on
property in goodwill acquired by use of the mark by the plaintiff.
Passing off is a common law tort used to enforce unregistered trademark rights.
THUS, if the mark in question has become well known in India, the user of such a
trademark is not without recourse and may seek a remedy by means of a passing
off action.
Passing off essentially occurs where
the reputation in the trademark of party A is misappropriated by party B,
such that party B misrepresents as being the owner of the trademark or having
some affiliation/nexus with party A,
thereby damaging the goodwill of party A.
The purpose of this tort is to protect commercial goodwill and to ensure that ones
business reputation is not exploited.
Since business goodwill is an asset and therefore a species of property, the law

CONTENTS | Module-1 | Module-2 | Module-3 | Module-4 http://duralex.bhatt.net.in/


https://www.facebook.com/groups/Dharmo.Rakshati.Rakshitah/ Page - 123 of 196

protects it against encroachment as such.


Standards of proof in passing off action :
In a passing off action, the plaintiff must establish that,
the mark, name or get-up - the use of which by the defendant is subject of the
action -
is distinctive of his goods in the eyes of the public or class of public
and that his goods are identified in the market by a particular mark or symbol.
Note : A passing off suit can be converted into a combined action of infringement
and passing off,
if the registration of the trademark is obtained before the final hearing of the
passing off suit.
Difference between passing-off and infringement of trade mark :

Infringement Passing off

1 Statutory remedy Common Law remedy

2 Registration of trademark is required Registration of trademark is not a


prerequisite

3 Plaintiff is only required to establish Plaintiff is not only required to


that infringing mark is deceptively establish deceptive similarity of two
similar to the registered mark in contesting mark but also require to
respect of similar goods / services and prove deception or confusion among
no further proof is required afterwards public and likelihood of injury to the
as there is presumption of confusion plaintiff's goodwill

4 Prosecution under Criminal remedies Prosecution under Criminal remedies is


is easier relatively harder

5 Benefit of instituting the suit under Such benefit is not available and the
Section 134 of the Trade Marks Act, regular rules of jurisdiction provided
1999 is available wherein the under Section 20 of Civil Procedure
registered proprietor or registered Code, 1908 apply i.e. Passing off action
user of the trademark can institute has to be filed where the Defendant
the suit where they actually and resides or carried on business or cause
voluntarily resides or carries on of actions has arisen.
business or personally work for gain

Summary :
Enforcement mechanisms are expected to boost the protection of marks in India
and reduce infringement and contravention of trademarks.

This is Module-3. Menu ---> CONTENTS | Module-1 | Module-2 | Module-3 | Module-4

CONTENTS | Module-1 | Module-2 | Module-3 | Module-4 http://duralex.bhatt.net.in/


https://www.facebook.com/groups/Dharmo.Rakshati.Rakshitah/ Page - 124 of 196

Discuss : Remedies and Penalty provisions under the Trade Marks Act, 1999.
ANSWER :
Refer :
https://www.icsi.edu/docs/webmodules/Publications/9.4%20Intellectual
%20Property%20Rights.pdf
http://www.ssrana.in/Intellectual
%20Property/Trademarks/Trademark_Protection.aspx
http://www.ssrana.in/Intellectual%20Property/Infringement/Difference-between-
Infringement-and-Passing-Off-in-India.aspx
http://www.mondaq.com/india/x/127680/Trademark/Trademarks+Law+In+India+
Everything+You+Must+Know
Outline :
Relief granted by Courts in Civil Suits for Infringement and Passing off :
Penalties in case of Criminal case for infringement and passing off :
Remedies under the Customs Laws for infringement and passing-off :
Penalty for falsification and falsely applying a trademark
Penalty for selling goods or providing services to which false trade mark or false
trade description is applied.
Intro :
An action for Infringement for registered trademark
or tort of Passing off for both registered as well as unregistered trademark
are essentially two ways of achieving the same objective i.e. protection of
goodwill attached with a mark.
In India a combined action for infringement and passing off is permissible.
However, both are technically two different concepts.
Civil Law, Criminal Law as well as Customs Law remedies are available in the
Infringement or Passing off action.
Relief granted by Courts in Civil Suits for Infringement and Passing off :
A suit can be initiated either under the law of passing off or for infringement under
the Trade Marks Act, 1999
depending on whether the trade mark is unregistered, pending registration or
registered respectively.
Jurisdiction and Venue :
The suit for passing off and/or infringement can be initiated either in the District
Court or in the High Court depending on the valuation of the suit.
The suit can be at the place where the rights holder or one of the rights holders

CONTENTS | Module-1 | Module-2 | Module-3 | Module-4 http://duralex.bhatt.net.in/


https://www.facebook.com/groups/Dharmo.Rakshati.Rakshitah/ Page - 125 of 196

actually and voluntarily reside or work for gain or carries on business.


Elements of the Complaint :
In the complaint, the rights holder is required to demonstrate that
(a) the alleged infringing act involves a mark that is identical or similar to a
trade mark of the rights holder;
(b) the infringing representation of a trade mark is being used in connection
with goods or services and might lead to confusion in public regarding the
origin of the infringing goods/services;
(c) the unlawful act interfered with the trade mark holder's rights of exclusive
use or caused the rights holder economic loss
Remedies under civil laws :
Sec-135 expressly stipulates that the relief which a Court may grant in any suit
for infringement or for passing off referred to in Sec-134, may include
following :
permanent and interim injunction,
appointment of a local commissioner,
for search, seizure and preservation of
infringing goods, account books and preparation of inventory, etc.
delivery of the infringing goods for destruction,
restraining the infringer from
disposing of or dealing with the assets in a manner
which may adversely affect plaintiff's ability to recover damages, costs or
other pecuniary remedies which may be finally awarded to the plaintiff.
damages or account of profits, and cost of the legal proceedings.
Ex-parte Interim Injunction :
The order of interim injunction may be passed ex parte or after notice.
This injunction is normally granted at the early stages of the trial
provided that the rights holder is able to establish his rights before the
Court and prove the gravity of the offence, merits immediate consideration.
Penalties in case of Criminal case for infringement and passing off :
The Trade Marks Act, 1999 provides for remedies for infringement under the
criminal laws too.
The police have the power to suo motu conduct raids and seizure operations.
However, the use of such powers by the police is minimal.
IF the TM right holder is aware of the details (name, address, dates of
infringement, etc.) of the infringers,
a complaint can be lodged with the police authorities and raids organized

CONTENTS | Module-1 | Module-2 | Module-3 | Module-4 http://duralex.bhatt.net.in/


https://www.facebook.com/groups/Dharmo.Rakshati.Rakshitah/ Page - 126 of 196

accordingly.
ELSE
it is advisable to procure a general search and seizure warrant from the local
magistrate and thereafter organize search and seizure operations in that area.
In a criminal proceeding, the litigation is between the State and the infringer and
therefore the rights holder has a limited role to play.
Penalties under criminal laws :
In case of a criminal action for infringement or passing off, following penalty
provisions are there :
Sec-103 : Penalty for applying false trade marks, trade descriptions :
imprisonment ranging from a term from 6 months to 3 years,
and fine ranging from INR 50,000 to INR 200,000.
Sec-104 : Penalty for selling goods or providing services to which false trade
mark or false trade description is applied.
imprisonment ranging from a term from 6 months to 3 years,
and fine ranging from INR 50,000 to INR 200,000.
Remedies under the Customs Laws for infringement and passing-off :
In addition to the civil and the criminal remedies mentioned herein above,
there are also provisions under the customs law which prohibit the importation
of infringing goods in India.
The Customs authorities have promulgated guidelines (known as the Intellectual
Property Rights (Imported Goods) Enforcement Rules, 2007)
under which the rights holder can record their registered trade marks with the
Customs authorities.
These guidelines authorize the Custom officials to seize goods infringing the trade
marks of the rights holder at the border without obtaining any orders from the
Court.
These rules also empower the Custom officers to destroy the suspected goods
under official supervision or dispose them outside the normal channels of
commerce
after it has been determined that the goods detained have infringed the trade
marks of the rights holder
and that no legal proceeding is pending in relation to such determination.
These rules also prohibit the re-exportation of the goods infringing trade marks in
an unaltered state.

This is Module-3. Menu ---> CONTENTS | Module-1 | Module-2 | Module-3 | Module-4

CONTENTS | Module-1 | Module-2 | Module-3 | Module-4 http://duralex.bhatt.net.in/


https://www.facebook.com/groups/Dharmo.Rakshati.Rakshitah/ Page - 127 of 196

Discuss under the Design Act, 2000 : Industrial Design. (Nov-2012, Oct-2013, Nov-
2014)
ANSWER :
Refer :
http://www.ipindia.nic.in/writereaddata/Portal/IPOGuidelinesManuals/1_30_1_man
ual-designs-practice-and-procedure.pdf <--- Key document for Designs.
http://www.wipo.int/edocs/pubdocs/en/intproperty/450/wipo_pub_450.pdf
http://www.nishithdesai.com/fileadmin/user_upload/pdfs/Research
%20Papers/Intellectual_Property_Law_in_India.pdf
https://www.icsi.edu/docs/webmodules/Publications/9.4%20Intellectual
%20Property%20Rights.pdf
Industrial Design, the concept :
Meaning of Design :
What is Not a Design?
Meaning of Industrial Design :
Importance of Industrial Design: Need to protect Design: Benefits of Design Right:
How can industrial designs be protected?
Meaning of Design Right :
How extensive is industrial design protection?
Intro :
Under the TRIPS Agreement, minimum standards of protection of industrial designs
have been provided for.
As a developing country, India has already amended its national legislation to
provide for these minimal standards.
Industrial designs in India are protected under the Designs Act, 2000.
The Designs Rules, 2001 have been framed under the Designs Act.
The Designs Act incorporates the minimum standards for the protection of
industrial designs, in accordance with the TRIPS agreement.
It provides for the introduction of an international system of classification, as per
the Locarno Classification.
Meaning of Design :
Sec-2(d) : Design Act 2000 :
design means
only the features of shape, configuration, pattern, ornament or composition of
lines or colours
applied to any article
whether in two dimensional or three dimensional or in both forms,

CONTENTS | Module-1 | Module-2 | Module-3 | Module-4 http://duralex.bhatt.net.in/


https://www.facebook.com/groups/Dharmo.Rakshati.Rakshitah/ Page - 128 of 196

by any industrial process or means,


whether manual mechanical or chemical, separate or combined,
which in the finished article appeal to and are judged solely by the eye.
However, design does not include
any mode or principle of construction,
or anything which is in substance a mere mechanical device,
and does not include
any trademark, or property mark, or any artistic work.
In order to obtain registration under the Designs Act,
the design must be applied to an article.
a mere painting or its presentation on paper is not entitled for registration.
What is Not a Design?
As stated in the definition of the design above, design does not include :
(i) any trademark, or
(ii) any property mark, or
(iii) any artistic work,
Artistic Work means
A painting, sculpture, drawing (including a diagram, map, chart or plan), an
engraving or a photograph, whether or not any such work possesses artistic
quality, architecture
Meaning of Industrial Design :
Hardly any other subject matter within the realm of intellectual property is as
difficult to explain as industrial designs.
Industrial designs refer to
creative activity which results in the ornamental or formal appearance of a
product.
Industrial designs are an element of intellectual property.
As per WIPO
In a legal sense, an industrial design constitutes the ornamental or aesthetic
aspect of an article.
A design may consist of
three-dimensional features, such as the shape or surface of an article,
or two-dimensional features, such as patterns, lines or color.
Industrial designs are applied to a wide variety of industrial products and
handicrafts :
from technical and medical instruments, to watches, jewelry and other luxury

CONTENTS | Module-1 | Module-2 | Module-3 | Module-4 http://duralex.bhatt.net.in/


https://www.facebook.com/groups/Dharmo.Rakshati.Rakshitah/ Page - 129 of 196

items;
from house wares and electrical appliances to vehicles and architectural
structures;
from textile designs to leisure goods.
An industrial design is,
primarily of an aesthetic nature,
any technical features of the article to which it is applied are not protected by
the design registration.
however, technical features could be protected by a patent.
Importance of Industrial Design : Need to protect Design : Benefits of Design Right :
Best to begin with an illustration :
Apple iPhones are manufactured in China.
But, China is able to capture paltry 2-5% of its value while overwhelming part is
cornered by USA.
This is mainly attributed to value added by Designing and Research, which is
based in USA.
Thus, importance of design protection cant be overstressed.
Industrial designs are what make an article attractive and appealing;
hence, they add to the commercial value of a product and increase its
marketability.
When an industrial design is protected, the owner the person or entity that has
registered the design is assured an exclusive right and protection against
unauthorized copying or imitation of the design by third parties.
This helps to ensure a fair return on investment.
An effective system of protection also benefits consumers and the public at large,
by promoting fair competition and honest trade practices,
encouraging creativity and promoting more aesthetically pleasing products.
Protecting industrial designs helps to promote economic development by
encouraging creativity in the industrial and manufacturing sectors, as well as in
traditional arts and crafts.
Designs contribute to increase in exports.
Industrial designs can be relatively simple and inexpensive to develop and protect.
They are reasonably accessible to small and medium-sized enterprises as well as to
individual artists and craftsmakers, in both developed and developing countries.
Today, industrial design has become an integral part of consumer culture where
rival articles compete for consumer's attention.
It has become important therefore, to grant to an original industrial design

CONTENTS | Module-1 | Module-2 | Module-3 | Module-4 http://duralex.bhatt.net.in/


https://www.facebook.com/groups/Dharmo.Rakshati.Rakshitah/ Page - 130 of 196

adequate protection.
How can industrial designs be protected?
In most countries, an industrial design must be registered in order to be protected
under industrial design law.
As a rule, to be registrable, the design must be new or original.
Countries have varying definitions of such terms, as well as variations in the
registration process itself.
Generally, new means that no identical or very similar design is known to have
previously existed.
Once a design is registered, a registration certificate is issued.
The term of protection granted is generally five years, with the possibility of further
renewal, in most cases for a period of up to 15 years.
Depending on the particular national law and the kind of design,
an industrial design may also be protected under copyright law, as a work of
applied art with a much longer term of protection than the standard 10 or 15
years under registered design law.
In some countries, protections under copyright and industrial design are mutually
exclusive,
ie, once owners choose one kind of protection, they can no longer invoke the
other.
Under certain circumstances an industrial design may also be protectable under
unfair competition law, although the conditions of protection and the rights and
remedies available can differ significantly.
Meaning of Design Right :
Design Right refers to
a property right for a novel or original design that is given to the proprietor of a
validly registered design.
In principle, owner of a registered industrial design or of a design patent
has the right to prevent third parties from making, selling or importing articles
bearing or embodying a design
which is a copy, or substantially a copy, of the protected design,
when such acts are undertaken for commercial purposes.
To be protected under most national laws, an industrial design must be new/
original and non-functional.
How extensive is industrial design protection?
Generally, industrial design protection is limited to the country in which protection
is granted.
The Hague Agreement Concerning the International Registration of Industrial

CONTENTS | Module-1 | Module-2 | Module-3 | Module-4 http://duralex.bhatt.net.in/


https://www.facebook.com/groups/Dharmo.Rakshati.Rakshitah/ Page - 131 of 196

Designs, a WIPO-administered treaty, offers a procedure for international


registration of designs.
Applicants can file a single international application either with WIPO or the
national or regional office of a country party to the treaty.
The design will then be protected in as many member countries of the treaty as the
applicant designates.

This is Module-3. Menu ---> CONTENTS | Module-1 | Module-2 | Module-3 | Module-4

Explain under the Design Act : Owner of Design (Oct-2013)


ANSWER :
Refer :

This is Module-3. Menu ---> CONTENTS | Module-1 | Module-2 | Module-3 | Module-4

Explain the provisions relating to registration under the Designs Act of 2000. (Nov-
2011)
Write Short note : Procedure for registration of designs (Nov-2014)
Discuss under the Design Act, 2000 : Prohibition on registration of Designs.
(Nov-2012)
Discuss under the Design Act, 2000 : About cancellation of registration of Design.
(Nov-2012)
ANSWER :
Refer :
http://www.nishithdesai.com/fileadmin/user_upload/pdfs/Research
%20Papers/Intellectual_Property_Law_in_India.pdf
https://www.icsi.edu/docs/webmodules/Publications/9.4%20Intellectual
%20Property%20Rights.pdf
http://hanumant.com/articles/AvinashB-Inellectual%20Property%20Rights%20IPR
%20Notes.html#_Toc317423149
Outline :
Who can Apply for Registration of a Design?
Designs prohibited from registration : [Sec-4]
Process of Registration :
Type of Applications

CONTENTS | Module-1 | Module-2 | Module-3 | Module-4 http://duralex.bhatt.net.in/


https://www.facebook.com/groups/Dharmo.Rakshati.Rakshitah/ Page - 132 of 196

Representation Sheet :
Classification of Designs : [Section 5(3), Rule 10, 11]
Processing of the Application and Formality Check : [Section 5(1)]
Substantive Examination : [Section 2 (d), 2(g), 5(1), 46 ]
Registration : [Section 5(1), Rule 18]
Rejection : Denial to register a design :
Register of designs :
Period of Protection : [Section 11]
Publication of registered design :
Rights Conferred by Registration : [Sec-11]
Cancellation of Registration of Design : [Section 19, Rule 29]
Who can Apply for Registration of a Design?
Any person (Indian or from a Convention Country),
claiming to be the proprietor of any new or original design
not previously published in any country
and is not contrary to public order or morality
can apply for the registration of the design.
The expressions public order or morality are not defined in the Designs Act.
Sec-2(g) : The term original, with respect to design, means
a design originating from the author of such a design
and includes the cases that, although old in themselves, are new in their
application.
Absolute novelty is now the criterion for registration. [Section 4]
A design shall be considered new if it is significantly distinguishable from known
designs or combination of known designs.
Illustration :
The figure of Taj Mahal is centuries old.
But if a person conceives for the first time, the idea of making a flower vase or
an ash tray in the form of figure of Taj Mahal,
that may be an original design and shall be registrable.
Designs prohibited from registration : [Sec-4]
The following designs are prohibited from registration :
1. A design which is not new or original.
2. A design which has been disclosed to the public anywhere in the world prior to
the filing date or the priority date of application.
3. A design which is not significantly distinguishable from known design or

CONTENTS | Module-1 | Module-2 | Module-3 | Module-4 http://duralex.bhatt.net.in/


https://www.facebook.com/groups/Dharmo.Rakshati.Rakshitah/ Page - 133 of 196

combination of known design.


4. A design which comprises or contains scandalous or obscene matter.
5. A design which is contrary to public order or morality.
Process of Registration :
The important purpose of design Registration is to see that the artisan, creator,
originator of a design having aesthetic look is not deprived of his bonafide reward
by others applying it to their goods.
File an application for registration of design with the prescribed fee
with the Controller of Patents and Designs.
Type of Applications : [Section 5, 44]
(a) Ordinary application.
An ordinary application does not claim priority.
(b) Reciprocity application.
A reciprocity application claims priority of an application filed previously in a
convention country.
Such an application shall be filed in India within six month from the date of
filing in Convention Country.
Priority Document : [Section 44, Rule 2(d), 15 ]
Reciprocity application shall be accompanied by a duly certified copy of the
design application filed in the Convention Country.
Representation Sheet :
(a) The representation sheet of an article needs to be prepared diligently and
shall be filed along with the application, in duplicate.
(b) Representation means the exact representation of the article for which
registration is sought.
Classification of Designs : [Section 5(3), Rule 10, 11]
(a) For the purpose of registration of designs,
articles are classified into thirty-one classes and a miscellaneous class 99, as
described in the Third Schedule of The Design Rules, 2001.
(b) The appropriate class shall be clearly identified with reference to The Third
Schedule and shall be mentioned in Form 1.
In case of any ambiguity, the Controller may decide the same, if necessary, in
consultation with the applicant.
Processing of the Application and Formality Check : [Section 5(1)]
On receipt of an application, the Office accords a date and serial number to the
application.
This serial number, upon registration, becomes the registration number of the

CONTENTS | Module-1 | Module-2 | Module-3 | Module-4 http://duralex.bhatt.net.in/


https://www.facebook.com/groups/Dharmo.Rakshati.Rakshitah/ Page - 134 of 196

design.
The application for registration of a design is refereed by the Controller of
Designs to an Examiner of Designs for conducting examination as to :
(a) whether the application and the documents satisfy the formal
requirements, and
(b) whether such design as applied to an article is registrable, under the
provisions of the Designs Act, 2000 and Designs Rules, 2001.
Substantive Examination : [Section 2 (d), 2(g), 5(1), 46 ]
Substantive examination is carried out to determine
whether the design under consideration is :
(a) a design under the Act?
(b) new or original?
(c) prejudicial to public order or morality?
(d) prejudicial to the security of India?
Registration : [Section 5(1), Rule 18]
The Controller shall consider the report of Examiner on registrability of a design
as applied to an article and if it is registrable, the same shall be registered
forthwith.
Sec-5 provides that the Controller may,
on the application of any person claiming to be the proprietor of any new or
original design not previously published in any country and which is not
contrary to public order or morality
register the design under the Act.
Sec-9 provides that, the Controller grants a certificate of registration to the
proprietor of the design when it is registered.
When registered, a design is deemed to have been registered as of the date of
the application for registration.
Rejection : Denial to register a design :
If upon consideration of the report of the Examiner, the Controller is of the
opinion that there are observations adverse to the applicant or the application
requires some amendment(s),
a statement of objections shall be communicated to the applicant or to his
agent.
The defects shall be corrected and the application resubmitted for acceptance
within six months.
If the Controller rejects the application, the aggrieved person can appeal to the
High Court.
Note :

CONTENTS | Module-1 | Module-2 | Module-3 | Module-4 http://duralex.bhatt.net.in/


https://www.facebook.com/groups/Dharmo.Rakshati.Rakshitah/ Page - 135 of 196

Unlike, Trade Mark, in Design Registration,


there is no procedure to publish design (prior to registration) and invite
public opposition to registration.
Register of designs :
Sec-10(1) :
There shall be kept at the patent office a book called the register of designs,
wherein shall be entered the names and addresses of proprietors of registered
designs, notifications of assignments and of transmissions of registered
designs, and such other matter as may be prescribed
and such register may be maintained wholly or partly on computer, floppies or
diskettes, subject to such safeguards as may be prescribed.
Sec-10(4) :
The register of designs shall be prima facie evidence of any matter by this Act
directed or authorized to be entered therein.
Period of Protection : [Section 11]
The duration of a design registration is initially ten years from the date of
registration,
but in cases where claim to priority has been allowed, the duration is ten years
from the priority date.
This initial period of ten years may be extended by a further period of 5 years, if
the registered proprietor applies for extension in prescribed manner.
Publication of registered design :
Sec-7 provides that, the Controller shall, as soon as may be after the registration
of a design,
cause publication of the prescribed particulars of the design to be published in
the prescribed manner and the design be open to public inspection.
Rights Conferred by Registration : [Sec-11]
Registration of a design confers upon the registered proprietor a copyright with
respect to the design.
Under the Designs Act, the word copyright refers to
the exclusive right to apply the design to any article in any class in which the
design has been registered.
The first term of registration is ten years after which it can be renewed for an
additional five-year period.
Cancellation of Registration of Design : [Section 19, Rule 29]
The registration of a design may be cancelled at any time after the registration of
the design,
on a petition for cancellation in Form 8, along with the prescribed fee.

CONTENTS | Module-1 | Module-2 | Module-3 | Module-4 http://duralex.bhatt.net.in/


https://www.facebook.com/groups/Dharmo.Rakshati.Rakshitah/ Page - 136 of 196

Any person interested may present a petition for a cancellation of the design
registration at any time after the registration, on the following grounds :
that the design has been previously registered in India;
that it has been published in India or in any other country prior to the
registration date;
that the design is not a new or original design;
that the design is not registrable under the Designs Act
or that it is not a design as defined under Section 2(d) of the Designs Act.
An appeal from any order by the Controller can be filed with the High Court.
Note : [Section 15 of The Copyright Act, 1957]
Copyright shall not subsist in any design which is registered under the Designs
Act.
Copyright in any design, which is capable of being registered under the Designs
Act, but is not registered,
will lose its copyright as soon as the design has been reproduced 50 times by
an industrial process by either the owner of the copyright or his licensee.

This is Module-3. Menu ---> CONTENTS | Module-1 | Module-2 | Module-3 | Module-4

Discuss : Powers of the Central Government under Design Act, 2000.


ANSWER :
Refer :
Sec-44 to Sec-47 of the Design Act 2000
<work on this>

This is Module-3. Menu ---> CONTENTS | Module-1 | Module-2 | Module-3 | Module-4

Discuss authorities under Designs Act 2000 and their powers and duties : (i)
Controller, (ii) Registrar.
Discuss Design Controller under the Designs Act, 2000. (Nov-2011, Nov-2012, Nov-
2014, Dec-2015)
Explain the controls over Registration of Designs. (Dec-2015)
ANSWER :
Refer :
Establishment of Design Controller and Examiners :
Sec-3 : Controller and other officers :

CONTENTS | Module-1 | Module-2 | Module-3 | Module-4 http://duralex.bhatt.net.in/


https://www.facebook.com/groups/Dharmo.Rakshati.Rakshitah/ Page - 137 of 196

(1) The Controller-General of Patents, Designs and Trade Marks


appointed under Section 4(1) of the Trade and Merchandise Marks Act, 1958
shall be the Controller of Designs for the purposes of this Act.
(2) For the purposes of this Act, the Central Government may appoint as many
examiners and other officers with such designations as it thinks fit.
Powers and duties of Examiners :
Sec-3 :
(3) The officers appointed under sub-section (2)
shall discharge under the superintendence and directions of the Controller
such functions of the Controller under this Act
as he may, from time to time, by general or special order in writing,
authorise them to discharge.
(4) The Controller may, by order in writing and for reasons to be recorded
therein,
withdraw any matter pending before an officer appointed under sub-section
(2)
and deal with such matter himself either de novo or from the stage it was so
withdrawn
or transfer the same to another officer appointed under sub-section (2) who
may, subject to special directions in the order of transfer, proceed with the
matter either de novo or from the stage it was so transferred.
Duties of examiners : Read following from discussion on Registration of Design :
Processing of the Application and Formality Check : [Section 5(1)]
Substantive Examination : [Section 2 (d), 2(g), 5(1), 46 ]
Functions of the Design Controller :
As per Sec-3(1),
Controller-General of Patents, Designs and Trade Marks
is the Controller of Designs for the purposes of this Act.
He is the top authority for compliance and enforcement of the Design Act 2000,
and is accordingly empowered to discharge all functions as
provided under the Design Act 2000 and the Design Rules 2001.
Sec-5 : Power to decide application for registration of designs
Sec-7 : Publish particulars of registered designs.
Sec-8 : Power of Controller to make orders regarding substitution of application.
Sec-9 : Certificate of registration
Sec-10, Sec-30, Sec-31, Sec-46 : Maintain Register of Design and

CONTENTS | Module-1 | Module-2 | Module-3 | Module-4 http://duralex.bhatt.net.in/


https://www.facebook.com/groups/Dharmo.Rakshati.Rakshitah/ Page - 138 of 196

ensure that proper entries of


registration status of design,
assignment & transmissions,
notice of interest in case of licenses & mortgages,
rectification of error, if any, (in response to application from any person)
are recorded in the Design Register.
Sec-12, Sec-13, Sec-14 : Power to restore lapsed designs.
Sec-15 : Ensure that owner of design right furnishes exact representation or
specimen of design as applied to articles, prior to delivery on sale of such articles.
Sec-17, Sec-46 : Let public inspect Design Register and provide certified copies of
registered designs, on completion of due formalities including payment of fees.
Sec-18 : Respond to any persons request to ascertain
whether registration still exists in respect of a design,
and, if so, in respect of what classes of articles,
and shall state the date of registration,
and the name and address of the registered proprietor
Sec-19 : Decide petitions for cancellation of registration.
Powers and Duties of the Design Controller : [Sec-32 to Sec-36)
Sec-32 : Powers of Controller in proceedings under Act :
the Controller in any proceedings before him under this Act
shall have the powers of a civil court
for the purpose of receiving evidence, administering oaths, enforcing the
attendance of witnesses, compelling the discovery and production of
documents, issuing commissions for the examining of witnesses and awarding
costs
and such award shall be executable in any court having jurisdiction as if it were
a decree of that court.
Sec-33 : Exercise of discretionary power by Controller : Doing natural justice :
Where any discretionary power is by or under this Act given to Controller,
he shall not exercise that power adversely to the applicant
without giving the applicant an opportunity of being heard.
Sec-34 : Power of Controller to take directions of the Central Government :
The Controller may, in any case of doubt or difficulty arising in the
administration of any of the provisions of this Act,
apply to the Central Government for directions in the matter.
Sec-35 : Refusal to register a design in certain cases :

CONTENTS | Module-1 | Module-2 | Module-3 | Module-4 http://duralex.bhatt.net.in/


https://www.facebook.com/groups/Dharmo.Rakshati.Rakshitah/ Page - 139 of 196

(1) The Controller may refuse to register a design of which the use would, in his
opinion, be contrary to public order or morality.
Sec-45 : Report of the Controller to be placed before Parliament :
Design Controller has duty to prepare, once a year, a report in respect to the
execution of the Design Act 2000, and
the Central Government shall cause, such report, to be placed before both
Houses of Parliament.

This is Module-3. Menu ---> CONTENTS | Module-1 | Module-2 | Module-3 | Module-4

Explain under the Design Act : Rectification of register (Oct-2013)


ANSWER :
Refer :

This is Module-3. Menu ---> CONTENTS | Module-1 | Module-2 | Module-3 | Module-4

Discuss : Assignment, transmission and licensing of designs.


ANSWER :
Refer :
https://www.icsi.edu/docs/webmodules/Publications/9.4%20Intellectual
%20Property%20Rights.pdf
http://www.nishithdesai.com/fileadmin/user_upload/pdfs/Research
%20Papers/Intellectual_Property_Law_in_India.pdf
Assignment/ transmission and licensing of Design :
There is no concept of common law license under design law.
The Copyright in the design is only protected if the same is statutorily recognized
under the provisions of the Designs Act, 2000.
Similarly, the rights acquired by third parties by way of assignments or licenses are
only made effective if the same is duly registered in accordance with the provisions
of the Act and the Rules framed there-under.
Section 30 of the Design Act, 2000 read with Rules 32, 33, 34 and 35 of the Design
Rules, 2001,
recognizes the contracts relating to assignment of designs and provides
procedure for the same.
Sec-30 :

CONTENTS | Module-1 | Module-2 | Module-3 | Module-4 http://duralex.bhatt.net.in/


https://www.facebook.com/groups/Dharmo.Rakshati.Rakshitah/ Page - 140 of 196

(1) Where a person becomes entitled by assignments, transmission or other


operation of law to the copyright in a registered design,
he may make application in the prescribed form to the Controller to register
his title,
and the Controller shall, on receipt of such application and on proof of title to
his satisfaction,
register him as the proprietor of such design,
and shall cause an entry (of such assignment, transmission or other
instrument affecting the title) to be made in the register of designs.
(2) Where any person becomes entitled as mortgagee, licensee or otherwise to
any interest in a registered design,
he may make an application in the prescribed form to the Controller to register
his title,
and the Controller shall, on receipt of such application and on proof of title to
his satisfaction,
cause notice of the interest to be entered in the register of designs,
with particulars of the instrument, if any, creating such interest.
Section 30(3) of the Design Act, 2000 makes it clear that,
for an assignment to be valid,
it must be in writing
and the agreement between the parties concerned has to be reduced to the
form of an instrument embodying all the terms and conditions governing their
rights and obligation,
and the application for registration of title under such instrument is filed in the
prescribed manner with the Controller within the stipulated time- that being
within six months from the execution of the instrument.
Section 30(4) of the Design Act, 2000 states that
the absolute right to assign the design rights lie with the person registered as
proprietor of the design.

This is Module-3. Menu ---> CONTENTS | Module-1 | Module-2 | Module-3 | Module-4

Discuss : Nature of Copyright in contrast to Design Right.


Discuss : Dart Industries Inc & Anr. vs Techno Plast & Ors.
Discuss : Samsonite Corp vs Vijay Sales.
ANSWER :
Refer :
http://lawtimesjournal.in/copyright-vs-design/

CONTENTS | Module-1 | Module-2 | Module-3 | Module-4 http://duralex.bhatt.net.in/


https://www.facebook.com/groups/Dharmo.Rakshati.Rakshitah/ Page - 141 of 196

Indian Copyright Act, 1957.


Copyright Rules, 1958.
Designs Act, 2000.
Designs Rules, 2001.
Copyright on registration :
Sec-11 :
(1) When a design is registered,
the registered proprietor of the design shall, subject to the provisions of this
Act
have copyright in the design during ten years from the date of registration.
When does an IP protected under copyright come under the realm of Design Act?
An artwork is protected under the Copyright Act 1957.
The moment
(i) the artwork is applied to any article to give the article an aesthetic appeal,
and
(ii) the article is sold in more than 50 number of units, under Section 15(2) of
the Copyright Act,
the artwork comes under the realms of the Design Act from the Copyright Act.
Display of design prior to application for registration of design :
Under Designs Act, 2000 protection is available to a person once he applies and
thereafter granted a registration for his design.
The Design Act bars displaying the article in question to general public before the
application.
Many designers, inadvertently, display their creations prematurely. eg in fashion
weeks, in their boutiques, showrooms etc.
This constitutes displaying to general public and such creations are not eligible
for design protections.
When some other persons copies the design, the designer, based on wrong legal
advice or her own understanding, files an action for copyright infringement against
the person.
Case of Samsonite Corp vs Vijay Sales at Delhi High Court :
<Move this discussion to 214K Legal Principles through Case Study>
Issues involved :
Nature of Copyright in contrast to Design Right.
Premature display of design, (hence no novelty anymore)
Design right infringement, (no design right because no novelty)
Copyright infringement, (no copyright after sale of 50 pieces of articles)

CONTENTS | Module-1 | Module-2 | Module-3 | Module-4 http://duralex.bhatt.net.in/


https://www.facebook.com/groups/Dharmo.Rakshati.Rakshitah/ Page - 142 of 196

Passing off, (defendant was not passing-off, infact they claimed to offer better
deal)
Copying designs by reverse engineering,
Unfair competition and unfair trade practice.
Facts of the case :
The Plaintiffs (Dart & Tupperware Indian Pvt. Ltd.) claimed
ownership of the know-how, intellectual property rights and trade secrets in its
designs used for the manufacture of Tupperware products.
they were a world-famous company having immense goodwill and reputation
and that the Defendants (Techno Plast), the proprietor of the mark
Signoware had copied their product designs which amounted to,
design infringement, copyright infringement, passing off, copying designs by
reverse engineering, unfair competition and unfair trade practice.
On the other hand, the Defendants claimed that
the products were distinct in terms of trade name, appearance, color scheme,
and were a product of their own skill and knowledge.
and that the designs were in public domain.
Court proceedings :
Single judge :
The Court held that once an object has been sold in over 50 units, the only
protection is under the Designs Act.
The Court looked into the meaning of publication and noted that it meant the
availability of knowledge or awareness about a particular design.
Registration is only prima facie evidence of novelty and originality and the
designs lacked originality
as the designs had been published in the 1989 book, The Tupperware
Cook.
and as they were substantially similar to those published earlier.
Therefore, the Court directed that the Plaintiff would be dis-entitled to
injunction on this ground.
The Court also rejected the claim of the Plaintiff viz. copyright infringement,
because the claims clearly fell within the ambit of the Designs Act, and
accordingly, copyright claims in industrial drawings and designs, to be used
for manufacturing processes were not maintainable.
The copyright ceases to exist once the design has been applied and
reproduced over 50 times.
However, since there were serious evidential doubts in proving that the
copyright survived design registration, the Court did not injunct the

CONTENTS | Module-1 | Module-2 | Module-3 | Module-4 http://duralex.bhatt.net.in/


https://www.facebook.com/groups/Dharmo.Rakshati.Rakshitah/ Page - 143 of 196

Defendants.
Issue of passing off :
Single judge held that the situation was actually contrary.
As per a report, the Defendants salesmen made representations to the
customers not that their products were Tupperware products, but that
they were as good in quality and appearance, yet half the price of
Tupperware.
ie the Defendants were not passing off their goods; rather they were
merely puffing up their products by claiming superior quality.
Division bench :
Contentions of the defendants :
That the 13 designs asserted by the Plaintiff relate to articles such as bowls,
casseroles and lunchboxes etc. which are commonplace and lack novelty
and originality. These designs were pre-published in nature and in the
public domain.
That no copyright can subsist in a registered design.
That it cannot be asserted that copyright can subsist in drawings of purely
functional day to day articles, unless they have inherent creativity.
That drawings of glasses, bowls etc. per se are incapable of copyright
protection as they are not artistic works.
Issue of passing off :
A visual comparison of the parties products proved that the Defendants
were copying the plaintiffs products.
Shapes and colors may acquire distinctiveness and are protectable by law.
A passing off action is not confined by trademark ONLY, it also covers trade
dress.
and that the similarity or confusion was not to be judged by the claim of
the parties but rather by what the products were saying and representing
about themselves.
action for passing off is a common law right, independent of the Designs
Act.
However, for a passing off action,
it must be proved that the general public associates the shape, trade
dress etc. with the Plaintiff alone
which was neither pleaded nor established.
Issue of designs being artistic work :
held that the Single Judge had erred in concluding that
IF the original works such as `product drawings intended for

CONTENTS | Module-1 | Module-2 | Module-3 | Module-4 http://duralex.bhatt.net.in/


https://www.facebook.com/groups/Dharmo.Rakshati.Rakshitah/ Page - 144 of 196

manufacture were used for manufacture of the designed article


they would not qualify as `artistic works under Section 2(c)(i) of the
Copyright Act.
Issue of novelty and originality of plaintiffs designs :
was pleased to agree with the analysis of the Single Judge that
the designs were prima facie in the public domain and lacked novelty,
and hence, could not be protected by the Designs Act despite
registration.
Issue of copyright infringement :
The Court held that the question of asserting a copyright infringement claim
independently cannot arise at all
when design protection subsists, but infringement has not been prima
facie established.
However, the Court went on to discuss the issue as an arguendo. It
observed that,
the language of Section 15 of the Copyright Act makes it clear that
copyright does not subsist in a registered design as this design
acquires a commercial element on mass reproduction which is not the
case with an artistic work.
The court noted that this is a complex issue, as works of art which have
been registered as designs lose even copyright protection,
thus undermining the efficacy of copyright itself.
The English case of Elzie Chrisler Segar (Deceased) v O & M Kleeman Ltd.
was discussed which pertained to the character,
Popeye the Sailor used in comics and later reproduced in merchandize.
In this case, the House of Lords distinguished between copyright and
design copyright and held that
design rights in this character did not result in the extinction of
copyright.
Considering this case, the Division Bench finally concluded that
copyright would subsist even after design registration in an original work
of art such as a painting.
The Division bench finally concurred with the single judge that
the drawings of commonplace household items do not satisfy the standards
of minimum creativity and cannot be legitimately monopolized.
Final order :
Holding that there was no infirmity with the conclusions of the single judge, the
Court dismissed the appeal by the original complainant.

CONTENTS | Module-1 | Module-2 | Module-3 | Module-4 http://duralex.bhatt.net.in/


https://www.facebook.com/groups/Dharmo.Rakshati.Rakshitah/ Page - 145 of 196

This is Module-3. Menu ---> CONTENTS | Module-1 | Module-2 | Module-3 | Module-4

Discuss : Patents to designs.


ANSWER :
Refer :

This is Module-3. Menu ---> CONTENTS | Module-1 | Module-2 | Module-3 | Module-4

Discuss : (i) Piracy of registered designs, (ii) Remedial aspects, (iii) Appeal
provisions.
Discuss under the Design Act, 2000 : Piracy of Registered Designs. (Nov-2012, Oct-
2013, Nov-2014)
ANSWER :
Refer :
http://www.nishithdesai.com/fileadmin/user_upload/pdfs/Research
%20Papers/Intellectual_Property_Law_in_India.pdf
(i) Piracy of registered designs :
The proprietor of a registered design gets exclusive right to apply the design to the
article in a class in which the design is registered.
During the existence of copyright over any design,
other persons are prohibited from using the design except or with the permission
of the proprietor, his licensee or assignee.
Essence of Sec-22 : Piracy of registered design : Infringement :
(1) During the existence of copyright in any design it shall not be lawful for any
person
(i) to apply for the purpose of sale
the design or any fraudulent imitation of it
to any article in any class of articles in which the design is registered;
(ii) to import for sale
any article to which the design or fraudulent or obvious imitation of it,
has been applied;
(iii) to publish or to expose such article for sale
knowing that the design or any fraudulent or obvious imitation of it has

CONTENTS | Module-1 | Module-2 | Module-3 | Module-4 http://duralex.bhatt.net.in/


https://www.facebook.com/groups/Dharmo.Rakshati.Rakshitah/ Page - 146 of 196

been applied to such article.


Note :
Any grounds on which the design can be cancelled can also be used as a defense
in an infringement action.
(ii) Remedial aspects : Penalties for infringement :
A registered proprietor can institute a suit for injunction as well as recovery of
damages against any person engaged in piracy of the registered design.
Such legal proceedings can be instituted from the date of registration and till the
expiry of design copyright.
In case of reciprocity application, the registered proprietor can claim damages only
from the actual date on which the design is registered in India.
If any person commits piracy of a registered design, as defined in Section 22,
he shall be liable to pay for a payment of a sum not exceeding 25,000/-
recoverable as contract debt.
However, the total sum recoverable in respect of any one design shall not
exceed ` 50,000/-.
The suit for injunction/ damages shall not be instituted in any Court below the
Court of District Judge. [Section 2(c),11,22.]
In a case between Ampro Food Products v. Ashok Biscuit Works, AIR 1973 AP 17,
The appellant manufactured biscuits with AP embossed on them.
The respondent also manufactured biscuits with identical design except that
letters AB were embossed on them, in place of AP.
The suit claimed injunction bringing a charge of piracy of design.
Issuing a temporary injunction, the Court held that in such cases,
the defence cannot argue that the appellants registered design was not new
or original, unless steps had been taken earlier seeking cancellation of the
registration of the design.
In a case between Hindustan Lever Ltd. V. Nirma Pvt. Ltd., AIR 1992 Bom 195,
The plaintiff alleged, (i) infringement of its registered trade mark, (ii) passing
off, and (iii) infringement of the copyrights in original artistic work.
and sought permanent injunction to restrain the defendant from using the
impinged carton in relation to soaps or detergent powder.
The defence took the plea that the said label was in fact a design that could be
registered under the Designs Act, and the fact that it was not so registered
makes copyrights if any, under the copyright Act non-existent when the article
to which the design has been applied was reproduced fifty times by industrial
process.
The Court ruled that a label to be put on a carton for the goods is not a design.

CONTENTS | Module-1 | Module-2 | Module-3 | Module-4 http://duralex.bhatt.net.in/


https://www.facebook.com/groups/Dharmo.Rakshati.Rakshitah/ Page - 147 of 196

(iii) Appeal provisions :


This is Module-3. Menu ---> CONTENTS | Module-1 | Module-2 | Module-3 | Module-4

CONTENTS | Module-1 | Module-2 | Module-3 | Module-4 http://duralex.bhatt.net.in/


https://www.facebook.com/groups/Dharmo.Rakshati.Rakshitah/ Page - 148 of 196

Module-4 :
4) The Law of Intellectual Property: Patents :
4.1) Concept of patent, Essentials, Historical view of the patents law in India
4.2) Patentable inventions with special reference to biotechnology products
entailing creation of new forms of life including Amendments of the year
2005
4.3) Patent protection for computer programme and computer software
4.4) Process of obtaining a patent :
4.4.1) Application and Examination : Procedure, Essentials for obtaining
patent : Elements of Novclty, Non-obviousness
4.4.2) Opposition and sealing of patent : general introduction, Grounds for
opposition
4.4.3) The problem of limited locus standi to oppose, especially in relation
inventions having potential of ecological and mass disasters
4.5) Patent Co-operation treaty : Object, reasons and important provisions
4.6) Wrong fully obtaining the invention : Prior Publication or anticipation,
Obviousness and the lack of inventive step, Insufficient description
4.7) Rights and obligations of a Patentee:
4.7.1) Patents as chose in action
4.7.2) Duration of patents law and policy considerations, Use and exercise
rights, Right to secrecy
4.7.3) The notion of "abuse" of patent rights, Compulsory licenses, Special
Categories, Employee Invention Law and Policy Consideration
4.8) International Patents, Transfer of Technology, Know-How and problems
of self reliant development.
4.9) Infringement, Criteria of infringement, Onus of Proof in India, Modes of
infringement, Doctrine of Colorable Variation
4.10) Remedies in case of Infringement : Injunctions and related remedies,
Defense in suits of infringement
4.11) Penal Provisions

This is Module-4. Menu ---> CONTENTS | Module-1 | Module-2 | Module-3 | Module-4

MODULE-4 QUESTIONS :

Explain in detail : Concept and essentials of patent (Apr-2016)

CONTENTS | Module-1 | Module-2 | Module-3 | Module-4 http://duralex.bhatt.net.in/


https://www.facebook.com/groups/Dharmo.Rakshati.Rakshitah/ Page - 149 of 196

Discuss : Historical view of the patents law in India.


Discuss under the Patent Act, 1970 : Power of Central Government to make
rules. (Nov-2012)
Explain under the Patent Act : Concept of Patent (Oct-2013)
Explain patentable invention and rights of patentee under Patent Act. (Dec-2015)
Write note : Inventions which not-patentable. (Nov-2011)
Explain under the Patent Act : Invention (Oct-2013)
Discuss under the Patent Act, 1970 : Patentable invention. (Nov-2012)
Who can apply for patents and who can not apply for patent ? (Nov-2014, Dec-
2015)
Discuss the meaning, types of specification under the Patents Act, 1970. (Nov-
2011)
Discuss under the Patent Act, 1970 : Qualification of a patent agent. (Nov-2012)
Discuss in detail : Patent Agent.
Explain under the Patent Act : Powers of Controller (Oct-2013)
State the powers of controller under Patent Act, (Nov-2014)
Discuss : Patentable biotechnology inventions entailing creation of new forms of
life including amendments of the year 2005
Explain in detail : Patent in computer programme and computer software (Apr-
2016)
Discuss : Process of obtaining a patent, including, (i) Application and Examination,
(ii) Essentials for obtaining patent, (iii) Elements of Novelty, Non-obviousness, (iv)
Opposition to the patent, (v) Problem of limited locus standi to oppose, especially in
relation to inventions having potential of ecological and mass disasters, (vi) Grant of
patents,
Discuss : Wrongfully obtaining the invention : Prior Publication or anticipation,
Obviousness and the lack of inventive step, Insufficient description.
Explain under the Patent Act : Revocation of Patents (Oct-2013)
Explain in detail : Compulsory licenses.
Write note : Lapsed Patent. (Nov-2011)
Discuss : Duration of patents.
Explain in detail : Rights and obligations of a patentee (Apr-2016)
Discuss : (i) Patents as chose in action, (ii) Duration of patents law and policy
considerations, (iii) Use and exercise rights, (iv) Right to secrecy , (v) "abuse"
of patent rights, (vi) Assignment & Licences, (vii) Special Categories, (viii)
Employee Invention Law and Policy Consideration
Discuss : Transfer of Technology, Know-How and problems of self reliant
development.

CONTENTS | Module-1 | Module-2 | Module-3 | Module-4 http://duralex.bhatt.net.in/


https://www.facebook.com/groups/Dharmo.Rakshati.Rakshitah/ Page - 150 of 196

Discuss under the Patent Act, 1970 : Rights of Patentee. (Nov-2012)


Discuss : Compulsory licence for benefit of disabled, (Nov-2014)
Discuss : Patent Co-operation treaty : Object, reasons and important provisions.
Discuss : (i) International Patents, (ii) Transfer of Technology, Know-How and
problems of self reliant development.
Discuss : Exceptions & Limitations To Patent Rights In India.
Discuss suits for infringement of Patent under the Patents Act, 1970. (Nov-2011)
Discuss : (i) Criteria of infringement, (ii) Modes of infringement, (iii)
Jurisdiction, (iv) Onus of Proof in India, (v) Doctrine of equivalent, (vi)
Doctrine of Colorable Variation.
Explain in detail : Remedies in case of infringement of the patent. (Apr-2016)
Discuss : (i) Injunctions and related remedies, (ii) Defense in suits of
infringement, (iii) Penal Provisions.

This is Module-4. Menu ---> CONTENTS | Module-1 | Module-2 | Module-3 | Module-4

MODULE-4 ANSWERS :

Explain in detail : Concept and essentials of patent (Apr-2016)


Discuss : Historical view of the patents law in India.
Discuss under the Patent Act, 1970 : Power of Central Government to make
rules. (Nov-2012)
Explain under the Patent Act : Concept of Patent (Oct-2013)
ANSWER :
Refer :
http://ipindia.gov.in/writereaddata/Portal/IPOGuidelinesManuals/1_28_1_manual-
of-patent-office-practice_and-procedure.pdf <--- Key document for Patents.
http://www.wipo.int/edocs/pubdocs/en/intproperty/450/wipo_pub_450.pdf
http://www.simplynotes.in/e-notes/mbabba/international-business-
management/ipr-intellectual-property-rights/
Patents : The Concept :
A patent is an exclusive right granted for an invention a product or a process,
that provides a new way of doing something,
or that offers a new technical solution to a problem.
Definition : Sec-2(1)(m) of the Patents Act, 1970 :

CONTENTS | Module-1 | Module-2 | Module-3 | Module-4 http://duralex.bhatt.net.in/


https://www.facebook.com/groups/Dharmo.Rakshati.Rakshitah/ Page - 151 of 196

patent means a patent for any invention granted under this Act
A patent provides patent owners with protection for their inventions.
A patent in the law is a property right and hence can be gifted, inherited, assigned,
sold or licensed.
As the right is conferred by the State, it can be revoked by the State under very
special circumstances even if the patent has been sold or licensed or manufactured
or marketed in the meantime.
Protection is granted for a limited period, generally 20 years.
Why are patents necessary?
Patents provide incentives to individuals by recognizing their creativity and
offering the possibility of material reward for their marketable inventions.
These incentives encourage innovation, which in turn enhances the quality of
human life.
What kind of protection do patents offer?
Patent protection means an invention cannot be commercially made, used,
distributed or sold without the patent owners consent.
Patent rights are usually enforced in courts that, in most systems, hold the
authority to stop patent infringement.
Likewise, a court can also declare a patent invalid upon a successful challenge
by a third party.
What rights do patent owners have?
A patent owner has the right to decide who may or may not use the
patented invention for the period during which it is protected.
Patent owners may give permission to, or license other parties to use their
inventions on mutually agreed terms.
Owners may also sell their invention rights to someone else, who then becomes
the new owner of the patent.
Once a patent expires, protection ends and the invention enters the public
domain.
This is also known as becoming off-patent. And such inventions becomes
available for commercial exploitation by others without having to obtain any
consent from anybody.
What role do patents play in everyday life?
Patented inventions have pervaded every aspect of human life,
from electric lighting (patents held by Edison and Swan), and sewing machines
(patents held by Howe and Singer),
to magnetic resonance imaging (MRI) (patents held by Damadian), and the
iPhone (patents held by Apple).

CONTENTS | Module-1 | Module-2 | Module-3 | Module-4 http://duralex.bhatt.net.in/


https://www.facebook.com/groups/Dharmo.Rakshati.Rakshitah/ Page - 152 of 196

In return for patent protection, all patent owners are obliged to publicly disclose
information on their inventions in order to enrich the total body of technical
knowledge in the world.
This ever-increasing body of public knowledge promotes further creativity and
innovation.
THUS, patents provide not only protection for their owners but also valuable
information and inspiration for future generations of researchers and inventors.
How is a patent granted?
The first step in securing a patent is to file a patent application.
The patent right is territorial in nature.
Inventors and their assignees will have to file separate patent applications in
countries of their interest, along with necessary fees for obtaining patents in
those countries.
The application generally contains the title of the invention, as well as an
indication of its technical field.
It must include the background and a description of the invention, in clear
language and enough detail that an individual with an average understanding of
the field could use or reproduce the invention.
Such descriptions are usually accompanied by visual materials drawings, plans
or diagrams that describe the invention in greater detail.
The application also contains various claims, that is, the extent of protection
that the inventor expects for the patent.
What kinds of inventions can be protected?
An invention must, in general, fulfill the following conditions to be protected by a
patent.
a) It must be of practical use;
b) it must show an element of novelty, meaning some new characteristic
that is not part of the body of existing knowledge (called prior art) in its
particular technical field.
c) The invention must show an inventive step that could not be deduced by a
person with average knowledge of the technical field.
d) Its subject matter must be accepted as patentable under law.
In many countries following are not generally patentable,
scientific theories, mathematical methods, plant or animal varieties,
discoveries of natural substances, commercial methods or methods of medical
treatment (as opposed to medical products)
Types of patents
i) Utility patents

CONTENTS | Module-1 | Module-2 | Module-3 | Module-4 http://duralex.bhatt.net.in/


https://www.facebook.com/groups/Dharmo.Rakshati.Rakshitah/ Page - 153 of 196

ii) Design patents


iii) Plant patents
i) Utility patents :
A utility patent may be available for inventions which are novel, useful, and non-
obvious.
Utility patents can be obtained for the utilitarian or functional aspects of an
invention.
Utility patents have a term of twenty years from the date of filling the patent
application with the patent and trademark office, although term extensions are
available in certain specific situations.
During the term of a utility patent, maintenance fees must be paid in order to
sustain the patent.
ii) Design patents :
It can be granted to any one who invents a new, original ornamental design for
an article of manufacture.
A design patent protects the ornamental design (i.e. appearance) of the article.
A design patent has duration of 14 years from the date of filing.
Design patents are not subject to maintenance fee payments.
iii) Plant patents :
Plant patent can be granted to any one who invents or discovers and reproduces
a new variety of plant.
A plant patent may be issued for the invention or discovery of a distinct and new
variety of plants, which may be asexually reproduced.
To qualify for this type of patent, the discovery or invention must be novel,
distinct, and non-obvious.
A plant patent has a term of 20 years from the date of filing.
Who grants patents?
Patents are granted by,
(i) national patent offices or
(ii) regional offices that carry out examination work for a group of countries.
for example, the European Patent Office (EPO) and the African Intellectual
Property Organization (OAPI).
Under such regional systems, an applicant requests protection for an invention in
one or more countries, and each country decides whether to offer patent
protection within its borders.
The WIPO-administered Patent Cooperation Treaty (PCT) provides for filing of,
a single international patent application that has the same effect as national
applications filed in all such nations which are party to PCT.

CONTENTS | Module-1 | Module-2 | Module-3 | Module-4 http://duralex.bhatt.net.in/


https://www.facebook.com/groups/Dharmo.Rakshati.Rakshitah/ Page - 154 of 196

An applicant seeking protection may file one application and request protection
in as many signatory states as needed.
Patents law in India :
The Patents Act, 1970 is a landmark in the industrial development of India.
The basic philosophy of the Act is that
patents are granted to encourage inventions and to secure that these inventions
are worked on a commercial scale without undue delay;
and patents are granted not merely to enable patentee to enjoy a monopoly for
the importation of the patented article into the country,
but also for larger socio-economic development.
The said philosophy is being implemented through
compulsory licensing,
registration of only process patents for food, medicine or drug, pesticides
and substances produced by chemical processes which, apart from chemical
substances also include items such as alloys, optical glass, semi-conductors,
inter metallic compounds etc.
It may, however, be noted that products vital for our economy such as agriculture
& horticulture products, atomic energy inventions and all living things are not
patentable.
Thus, the Patents Act 1970 was expected to provide a reasonable balance
between adequate and effective protection of patents on the one hand
and the technology development, public interest and specific needs of the
country on the other hand.
After Uruguay round of GATT and WTO, India was put under the contractual
obligation to amend its Patents Act in compliance with the provisions of TRIPS.
India had to meet the first set of requirements on 1-1-1995.
Accordingly an Ordinance effecting certain changes in the Act was issued on 31st
December 1994, which ceased to operate after six months.
Subsequently, another Ordinance was issued in 1999. This Ordinance was
replaced by the Patents (Amendment) Act, 1999 that was brought into force
retrospectively from 1st January, 1995.
India amended its Patents Act again in 2002 increasing the term of patent to 20
years for all technology, Reversal of burden of proof, compulsory licences etc.
This Act came into force on 20th May 2003 with the introduction of the new Patent
Rules, 2003.
Power of Central Government to make rules :
Under the provisions of Section 159 of the Patents Act, 1970 the Central
Government is empowered to make rules for implementing the Act and regulating

CONTENTS | Module-1 | Module-2 | Module-3 | Module-4 http://duralex.bhatt.net.in/


https://www.facebook.com/groups/Dharmo.Rakshati.Rakshitah/ Page - 155 of 196

patent administration.
Accordingly, the Patents Rules, 2003 were notified and brought into force w.e.f.
20.4.1972.
These Rules were amended from time to time till 20th May 2003 when new Patents
Rules, 2003 were brought into force by replacing the 1972 rules.
These rules were further amended by the Patents (Amendment) Rules, 2005 and
the Patents (Amendment) Rules, 2006.
The last amendments are made effective from 5th May 2006.
There are four Schedules to the Patents (Amendment) Rules;
the First Schedule prescribes the fees to be paid;
the Second Schedule specifies the list of forms and the texts of various forms
required in connection with various activities under the Patents Act. These forms
are to be used wherever required and if needed, they can be modified with the
consent of the Controller.
the Third Schedule prescribes form of Patent to be issued on Grant of the Patent.
the Fourth Schedule prescribes costs to be awarded in various proceedings
before the Controller under the Act.

This is Module-4. Menu ---> CONTENTS | Module-1 | Module-2 | Module-3 | Module-4


GO TO MODULE-4 QUESTIONS.
GO TO CONTENTS.

Explain patentable invention and rights of patentee under Patent Act. (Dec-2015)
Write note : Inventions which not-patentable. (Nov-2011)
Explain under the Patent Act : Invention (Oct-2013)
Discuss under the Patent Act, 1970 : Patentable invention. (Nov-2012)
ANSWER :
Refer :
https://www.icsi.edu/docs/webmodules/Publications/9.4%20Intellectual
%20Property%20Rights.pdf
Patents Act, 1970.
Patentable subject matters :
Intro :
A patent is granted for an invention which may be related to any process or
product.
An invention is different from a discovery.
Discovery is something that already existed but had not been found.

CONTENTS | Module-1 | Module-2 | Module-3 | Module-4 http://duralex.bhatt.net.in/


https://www.facebook.com/groups/Dharmo.Rakshati.Rakshitah/ Page - 156 of 196

Not all inventions are patentable. An invention must fulfill certain requirements
known as conditions of patentability.
Section 2(1)(j) : Meaning of invention :
Invention means a new product or process involving an inventive step and
capable of industrial application.
Here, capable of industrial application, means that the invention is capable of
being made or used in an industry [Section 2(1)(ac)].
Elements of Patentability :
The patent must be in respect of an invention and not a discovery.
The fundamental principle of Patent Law is that a patent is granted only for an
invention which must be new and useful.
That is to say, it must have novelty and utility.
It is essential for the validity of a patent that it must be the inventors own work
as opposed to mere verification of what was already known before the date of
the patent.
It is important to bear in mind that in order to be patentable an improvement
should be something more than a mere workshop improvement;
and must independently satisfy the test of invention or an inventive step.
To be patentable the improvement or the combination must produce a new
result, or a new article or a better or cheaper article than before.
Sec-2(1)(l) : New invention is defined as any invention or technology which
has not been anticipated by publication in any document or used in the country
or elsewhere in the world before the date of filing of patent application with
complete specification,
i.e., the subject matter has not fallen in public domain or that it does not form
part of the state of the art;
In Raj Prakash v. Mangat Ram Choudhary AIR 1978 Del, it was held that
invention, as is well known, is to find out some thing or discover some thing
not found or discovered by anyone before.
It is not necessary that the invention should be any thing complicated.
The essential thing is that the inventor was first to adopt it.
The principal therefore, is that every simple invention that is claimed, so long as
it is something which is novel or new, it would be an invention and the claims
and specifications have to be read in that light.
Therefore, the essential elements of patentability are :
1. Novelty
2. Inventive step (non-obviousness) and
3. Industrial applicability (utility)

CONTENTS | Module-1 | Module-2 | Module-3 | Module-4 http://duralex.bhatt.net.in/


https://www.facebook.com/groups/Dharmo.Rakshati.Rakshitah/ Page - 157 of 196

1. Novelty :
A novel invention is one, which has not been disclosed, in the prior art
Here, prior art means everything that has been published, presented or
otherwise disclosed to the public on the date of patent
The prior art includes documents in foreign languages disclosed in any
format in any country of the world.
ie For an invention to be judged as novel, the disclosed information should not
be available in the 'prior art'.
This means that there should not be any prior disclosure of any information
contained in the application for patent (anywhere in the public domain, either
written or in any other form, or in any language) before the date on which the
application is first filed i.e. the 'priority date'.
The term prior art has not been defined under the Indian Patents Act,
it shall be determined by the provisions of Section 13 read with the provisions
of Sections 29 to 34.
ie An invention shall NOT be considered to be novel IF
(a) it has been anticipated by publication before the date of the filing of the
application
in any of the specification filed in pursuance of application for patent in
India on or after the 1st day of January 1912.
or in any of the documents in any country.
(c) if it has been claimed in any claim of any other complete specification filed
in India which is filed before the application but published after said
application.
(d) if it has been anticipated having regard to the knowledge, oral or
otherwise, available within any local or indigenous community in India or
elsewhere.
Case-law :
In Ganendro Nath Banerji v. Dhanpal Das Gupta, AIR1945 Oudh 6, it was held
that
no general rule can be laid down as to what does or does not constitute an
invention.
Novelty need only be established in the process of manufacturing, not in the
article produced.
Novel combination of two known ideas may be sufficient to establish novelty
of subject matter in this respect.
In Ram Narain Kher v. M/s Ambassador Industries, AIR 1976 Del 87, the Delhi
High Court has held that

CONTENTS | Module-1 | Module-2 | Module-3 | Module-4 http://duralex.bhatt.net.in/


https://www.facebook.com/groups/Dharmo.Rakshati.Rakshitah/ Page - 158 of 196

at the time the patent is granted to a party it is essential that the party
claiming patent should
specify what particular features of his device distinguish it from those
which had gone before
and show the nature of the improvement which is said to constitute the
invention.
A person claiming a patent has not only to allege the improvement in art in
the form but also that the improvement effected a new and very useful
addition to the existing state of knowledge.
2. Inventive Step (Non-obviousness) :
Inventive step is a feature of an invention that involves technical advance as
compared to existing knowledge or having economic significance or both, making
the invention non obvious to a person skilled in art.
Here definition of inventive step has been enlarged to include economic
significance of the invention apart from already existing criteria for determining
inventive step.
An invention shall not be considered as involving an inventive step, if, having
regard to the state of the art, it is obvious to a person skilled in the art.
The term "obvious" means that which does not go beyond the normal progress
of technology but merely follows plainly or logically from the prior art,
i.e. something which does not involve the exercise of any skill or ability beyond
that to be expected of the person skilled in the Art.
For this purpose a person skilled in the art should be presumed to be an
ordinary practitioner aware of what was general common knowledge in the
relevant art at the relevant date.
In some cases the person skilled in the art may be thought of as a group or
team of persons rather than as a single person.
3. Industrial Applicability :
An invention is capable of industrial application if it satisfies three conditions,
cumulatively:
can be made;
can be used in at least one field of activity;
can be reproduced with the same characteristics as many times as necessary.
What kind of utility/ usefulness is required for patenr?
An invention to be patentable must be useful. If the subject matter is devoid of
utility it does not satisfy the requirement of invention.
For the purpose of utility, the element of commercial or pecuniary success has
no relation to the question of utility in patent law.

CONTENTS | Module-1 | Module-2 | Module-3 | Module-4 http://duralex.bhatt.net.in/


https://www.facebook.com/groups/Dharmo.Rakshati.Rakshitah/ Page - 159 of 196

The usefulness of an alleged invention depends on, whether by following the


directions in the complete specification the effects that the application/ patentee
professed to produce, could be obtained.
The usefulness of the invention is to be judged, by the reference to the state of
things at the date of filing of the patent application,
the fact that subsequent improvement have replaced the patented invention
render it obsolete and commercially of no value, does not invalidate the
patent.
Speculation or imaginary industrial uses are not considered to satisfy the
industrial application requirement.
Non patentable subject matters :
An invention may satisfy the condition of novelty, inventiveness and usefulness,
but it may not qualify for a patent.
Sec-3 : Patents Act 1970 : The following are not inventions within the meaning of
Section 3 of the Patents Act, 1970 :
(a) an invention which is frivolous or which claims anything obviously contrary to
well established natural laws;
(b) an invention the primary or intended use or commercial exploitation of which
could be contrary to public order or morality or which causes serious prejudice to
human, animal or plant life or health or to the environment;
(c) the mere discovery of a scientific principle or the formulation of an abstract
theory or discovery of any living thing or non-living substances occurring in
nature;
(d) the mere discovery of a new form of a known substance which does not
result in the enhancement of the known efficacy of that substance or the mere
discovery of any property or mere new use for a known substance or of the mere
use of a known process, machine or apparatus unless such known process
results in a new product or employs at least one new reactant;
(d) salts, esters, polymorphs, metabolites, pure form, particle size, isomers,
mixtures of isomers, complexes, combinations and other derivatives of known
substance shall be considered to be the same substance, unless they differ
significantly in properties with regard to efficacy.
(e) a substance obtained by a mere admixture resulting only in the aggregation
of the properties of the components thereof or a process for producing such
substance;
(f) the mere arrangement or re-arrangement or duplication of known devices
each functioning independently of one another in a known way;
(g) omitted by Patents (Amendment) Act, 2002.
(h) a method of agriculture or horticulture;

CONTENTS | Module-1 | Module-2 | Module-3 | Module-4 http://duralex.bhatt.net.in/


https://www.facebook.com/groups/Dharmo.Rakshati.Rakshitah/ Page - 160 of 196

(i) any process for the medicinal, surgical, curative, prophylactic diagnostic,
therapeutic or other treatment of human beings or any process for a similar
treatment of animals to render them free of disease or to increase their
economic value or that of their products;
(j) plants and animals in whole or any part thereof other than micro-organisms
but including seeds, varieties and species and essentially biological processes for
production or propagation of plants and animals;
(k) a computer programme per se other than its technical application to industry
or a combination with hardware;
(l) a literary, dramatic, musical or artistic work or any other aesthetic creation
whatsoever including cinematographic works and television productions;
(m) a mere scheme or rule or method of performing mental act or method of
playing game;
(n) a presentation of information;
(o) topography of integrated circuits;
(p) an invention which in effect, is traditional knowledge or which is an
aggregation or duplication of known properties of traditionally known component
or components.
Sec-4 of Patents Act 1970 prohibits the grant of patent in respect of an invention
relating to atomic energy falling within Sec-20(1) of the Atomic Energy Act, 1962.
Sec-20 of the Atomic Energy Act, 1962 contains special provision relating to
inventions.
Under Sec-20 (1) of the Atomic Energy Act, 1962
no patents shall be granted for inventions which in the opinion of the Central
Government are useful for or relate to the production, control, use or disposal of
atomic energy or the prospecting, mining, extraction, production, physical and
chemical treatment, fabrication, enrichment, canning or use of any prescribed
substance or radioactive substance or the ensuring of safety in atomic energy
operations.

This is Module-4. Menu ---> CONTENTS | Module-1 | Module-2 | Module-3 | Module-4

Who can apply for patents and who can not apply for patent ? (Nov-2014, Dec-
2015)
ANSWER :
Refer :

CONTENTS | Module-1 | Module-2 | Module-3 | Module-4 http://duralex.bhatt.net.in/


https://www.facebook.com/groups/Dharmo.Rakshati.Rakshitah/ Page - 161 of 196

This is Module-4. Menu ---> CONTENTS | Module-1 | Module-2 | Module-3 | Module-4

Discuss the meaning, types of specification under the Patents Act, 1970. (Nov-
2011)
ANSWER :
Refer :
https://www.icsi.edu/docs/webmodules/Publications/9.4%20Intellectual
%20Property%20Rights.pdf
Specification :
In order to obtain a patent, an applicant must fully and particularly describe the
invention therein claimed in a complete specification.
The disclosure of the invention in a complete specification must be such that a
person skilled in the art may be able to perform the invention.
This is possible only when an applicant discloses the invention fully and particularly
including the best method of performing the invention.
The Specification is a techno-legal document containing full scientific details of the
invention and claims to the patent rights.
The Specification, thus, forms a crucial part of the Patent Application.
It is mandatory on the part of an applicant to disclose fully and particularly various
features constituting the invention.
Types of specifications :
The Specification may be filed either,
(i) as a provisional specification, or
(ii) as a complete specification.
The Specification (provisional or complete) is to be submitted in Form-2
along with the Application in Form-1 and other documents, in duplicate,
along with the prescribed fee as given in the First Schedule.
The first page of the Form 2 shall contain :
(a) Title of the invention;
(b) Name, address and nationality of each of the applicants for the Patent; and
(c) Preamble to the description.
The title of the invention shall sufficiently indicate the specific features of the
invention.
Every Specification whether provisional or complete shall describe the invention.
The applicant shall submit drawings, wherever required.
The Controller may also require the applicant to submit drawings, if necessary at

CONTENTS | Module-1 | Module-2 | Module-3 | Module-4 http://duralex.bhatt.net.in/


https://www.facebook.com/groups/Dharmo.Rakshati.Rakshitah/ Page - 162 of 196

the examination stage [Section 9, 10. Rule 13. Form-1, 2].


Such drawings shall form a part of the Specification and suitable references thereto
shall be made in the Specification.
The Controller may require the applicant to submit, anytime before the grant,
models or samples related to the invention for better illustration of the invention.
However, such models or samples shall not form part of the Specification.
1. Provisional Specification :
When the applicant finds that his invention has reached a stage wherein it can be
disclosed on paper, but has not attained the final stage,
he may prepare a disclosure of the invention in the form of a written description
and submit it to Patent Office as a provisional specification which describes the
invention.
Need : A provisional specification secures a priority date for the application
over any other application which is likely to be filed in respect of the same
invention being developed concurrently.
Immediately on receiving the Provisional Specification the Patent Office accords a
filing date and application number to the Application.
Sec-9 stipulates that an application for a patent accompanying a provisional
specification,
a complete specification shall be filed within twelve months from the date of
filing of the application, and if the complete specification is not so filed, the
application shall be deemed to be abandoned.
If two provisional specifications filed by an applicant are cognate or if one is a
modification of the other, the applicant may file one complete specification covering
both the provisional applications.
Such a complete specification shall have to be filed within twelve months from
the date of filing of the first provisional application.
In such cases, date of filing of application is the date of filing of the earliest
provisional specification and shall bear the number of that application.
An applicant may, within twelve months from the filing of a complete specification
(not being a convention application or a PCT National Phase Application), convert
the same into a provisional specification.
Consequently, the applicant has to file a complete specification within twelve
months from the date of first filing.
A provisional specification (i.e. the one filed directly or the one converted from a
complete specification) may be postdated to the date of filing of the complete
specification.
2. Complete Specification :
The complete specification is a techno-legal document which

CONTENTS | Module-1 | Module-2 | Module-3 | Module-4 http://duralex.bhatt.net.in/


https://www.facebook.com/groups/Dharmo.Rakshati.Rakshitah/ Page - 163 of 196

fully and particularly describes the invention


and discloses the best method of performing the invention.
As the complete specification is an extremely important document in the patent
proceedings it is advised that it should be drafted with utmost care without any
ambiguity.
Important Elements of the Complete Specification :
Sec-10 : Every complete specification is required to -
(a) fully and particularly describe the invention and its operation or use and
the method by which it is to be performed;
(b) disclose the best method of performing the invention which is known to the
applicant and for which he is entitled to claim protection; and
(c) end with a claim or claims defining the scope of the invention for which
protection is claimed; and
(d) be accompanied by an abstract to provide technical information on the
invention.
A complete specification customarily begins after the title,
with a general preamble stating the subject to which the invention relates,
followed by a detailed description of one or more embodiments of the inventions.
In an infringement action, the function of the Court is
to construe the claims which are alleged to have been infringed,
without reference to the body of the specification,
and to refer to the body of the specification only if there is any ambiguity or
difficulty in the construction of the claims in question [Farbwerke Hoechst v.
Unichem Laboratories, AIR1969 Bom 255]
The requirements of the specifications have been discussed under Rule 13, 14, &
15.
In case of an international application designating India,
the title, description, drawings, abstracts and claims filed with the application
shall be taken as the complete specification for the purposes of the Act.
The claim or claims of a complete specification
shall relate to a single invention, or to a group of inventions linked
so as to form a single inventive concept,
shall be clear and succinct and shall be fairly based on the matter disclosed in
the specification. [Section 10(4)(a)]

This is Module-4. Menu ---> CONTENTS | Module-1 | Module-2 | Module-3 | Module-4

CONTENTS | Module-1 | Module-2 | Module-3 | Module-4 http://duralex.bhatt.net.in/


https://www.facebook.com/groups/Dharmo.Rakshati.Rakshitah/ Page - 164 of 196

Discuss under the Patent Act, 1970 : Qualification of a patent agent. (Nov-2012)
Discuss in detail : Patent Agent.
ANSWER :
Refer :
https://www.icsi.edu/docs/webmodules/Publications/9.4%20Intellectual
%20Property%20Rights.pdf
Intro :
The work relating to
drafting of specifications,
making of application for a patent,
subsequent correspondence with the Patent office on the objections raised,
representing the applicants case at the hearings,
filing opposition and defending application against opposition
is entrusted to a qualified Patent Agent.
Sections 125-132 of the Patents Act, 1970 and Rules 108-120 of the Patents Rules,
2003 deal with the Patent Agents.
The Controller maintains a register to be called the register of patent agents in
which there will be entered the names, address and other relevant particular as
may be prescribed of all persons qualified to have their names so entered according
to section 126.
According to rule 4 such register has to be maintained at the Head Office of the
Patent Office, Calcutta.
The register will contain the name, nationality address of the principal place of
business, branch office address and other relevant particular as may be prescribed,
if any, the qualification and the date of registration of entry as Patent agent.
Qualifications for Registration as Patent Agents :
According to Section 126 a person will be qualified to have his name entered in the
register of patent agents if he fulfils the following conditions, namely :
(a) He/she is a citizen of India;
(b) He/she has completed the age of 21 years;
(c) He/she has obtained a degree in Science, Engineering or Technology from
any University established under the law for the time being in force in the
territory of India or possesses such other equivalent qualifications as the Central
Government may specify in this behalf, and, in addition, -
(i) has passed the qualifying examination prescribed for the purpose; or
(ii) has, for a total period of not less than ten years, functioned either as an
examiner or discharged the functions of the Controller under section 73, or
both, but ceased to hold any such capacity at the time of making the

CONTENTS | Module-1 | Module-2 | Module-3 | Module-4 http://duralex.bhatt.net.in/


https://www.facebook.com/groups/Dharmo.Rakshati.Rakshitah/ Page - 165 of 196

application for registration;


(d) He has paid such fee as may be prescribed.
However, a person who has been registered as a patent agent before the
commencement of Patent (Amendment) Act, 2005 will be entitled to continue to
be, or when required to be re-registered as a patent agent, on payment of the fee.
Qualifying Examinations for Patent Agents :
In order to maintain high professional efficiency, the Act has provided for certain
prescribed qualification for the patent agents, as mentioned above in the section
126.
As per clause (c)(ii) of sub section 1 of section 126 there is a provision for
conducting the qualifying examination for Patent agents, which will consist of a
written test and a viva voce examination.
The written Test shall be of two papers i.e. l) Patents Act and Rules and 2) Drafting
and interpretation of Patent specification and other documents of 100 marks each
(Rule 110(2)).
The qualifying marks for written paper & for the viva voice examination is fifty per
cent each, and a candidate shall be declared to have passed the examination only if
he obtains an aggregate of sixty percent of the qualifying marks (Rule 110 (3))
Registration of Patent Agents :
As per Rule 111, after a candidate passes the qualifying examination specified in
rule 110 and after obtaining any further information, which the Controller considers
necessary, and on receipt of the fee will enter the candidates name in the register
of patent agents and issue to him a certificate of registration as a patent agent.

This is Module-4. Menu ---> CONTENTS | Module-1 | Module-2 | Module-3 | Module-4

Explain under the Patent Act : Powers of Controller (Oct-2013)


State the powers of controller under Patent Act, (Nov-2014)
ANSWER :
Refer :

This is Module-4. Menu ---> CONTENTS | Module-1 | Module-2 | Module-3 | Module-4

Discuss : Patentable biotechnology inventions entailing creation of new forms of


life including amendments of the year 2005
ANSWER :

CONTENTS | Module-1 | Module-2 | Module-3 | Module-4 http://duralex.bhatt.net.in/


https://www.facebook.com/groups/Dharmo.Rakshati.Rakshitah/ Page - 166 of 196

Refer :

This is Module-4. Menu ---> CONTENTS | Module-1 | Module-2 | Module-3 | Module-4

Explain in detail : Patent in computer programme and computer software (Apr-


2016)
ANSWER :
Refer :

This is Module-4. Menu ---> CONTENTS | Module-1 | Module-2 | Module-3 | Module-4

Discuss : Process of obtaining a patent, including, (i) Application and Examination,


(ii) Essentials for obtaining patent, (iii) Elements of Novelty, Non-obviousness, (iv)
Opposition to the of patent, (v) Problem of limited locus standi to oppose, especially in
relation to inventions having potential of ecological and mass disasters, (vi) Grant of
patents, (vii) Term of patents.
ANSWER :
Refer :
http://www.simplynotes.in/e-notes/mbabba/international-business-
management/ipr-intellectual-property-rights/
https://www.icsi.edu/docs/webmodules/Publications/9.4%20Intellectual
%20Property%20Rights.pdf
(i) Application and Examination :
Sec-6 of the Act provides that an application for a patent for an invention may be
made by any of the following persons either alone or jointly with another :
(a) by any person claiming to be the true and first inventor of the invention;
(b) by any person being the assignee of the person claiming to be the true and
first inventor in respect of the right to make such an application;
(c) by the legal representative of any deceased person who immediately before
his death was entitled to make such an application.
Patent Cooperation Treaty (PCT) :
Several countries in 1970 decided to simplify the process for protecting patents
around the world by creating the Patent Cooperation Treaty (PCT).

CONTENTS | Module-1 | Module-2 | Module-3 | Module-4 http://duralex.bhatt.net.in/


https://www.facebook.com/groups/Dharmo.Rakshati.Rakshitah/ Page - 167 of 196

Under the Patent Cooperation Treaty, inventors can submit just one international
application which is valid in any or all of the more than 120 countries that are
members of this Treaty.
Inventors may apply for a patent either in all the member countries of PCT or in
selected group of countries.
Only inventors who are citizens or residents of the member countries of the PCT
can use this easier system to file international patent applications.
Where to apply?
Application for the patent has to be filled in the respective patent office.
The territorial jurisdiction is decided based on whether any of the following
occurrences falls within the territory.
Place of residence,domicile or business of the applicant(first mentioned
applicant in the case of joint applicants)
Place from where the invention actually originated.
Address for services in India given by the applicant when he has no place of
business or domicile in India.
A foreign applicant should give an address for service in India and the
jurisdiction will be decided upon that. An applicant (Indian or foreigner) also can
his Patent Agents address as address for serving documents,if he/she so wishes.
How to apply?
Steps involved in obtaining a Patent are as follows :
Section 7 dealing with form of application requires every application for a
patent to be made for one invention only.
A patent may be in respect of a substance or in respect of a process.
But it is not possible to bifurcate a patent and state that it relates to the
substance and the other to the process.
Every international application under the Patent Cooperation Treaty (PCT) for a
patent, as may be filed designating India shall be deemed to be an application
under the Act, if a corresponding application has also been filed before
Controller in India.

In order to have a complete patent, the specifications and the claims must be
clearly and distinctly mentioned.
To file an application for patent accompanied with either a provisional
specification or a complete specification
To file complete specification,if provisional application was submitted earlier.
Examination of the application by patent office.
Acceptance of applications and publication in the gazette of accepted

CONTENTS | Module-1 | Module-2 | Module-3 | Module-4 http://duralex.bhatt.net.in/


https://www.facebook.com/groups/Dharmo.Rakshati.Rakshitah/ Page - 168 of 196

applications.
Responding and satisfactory overcoming the opposition/objections,if any to the
grant of patent.
Sealing of patent
Application for patent (Form 1) in duplicate should be accompanied by required
documents along with requisite fees.
Patent documents to be submitted :
There are two types of patent documents usually known as patent
specification,namely
A. Provisional specification
B. Complete Specificaion
A. Provisional Specification :
A provisional specification is usually filled to establish priority of the invention
in case the disclosed invention is at a conceptual stage and a delay is expected
in submitting full and specific description of the invention.
Although,a patent application accompanied with provisional specification does
not confer any legal patent rights to the applicants, it is however a very
important document to establish the earliest ownership of an invention.
Complete specification must be submitted within 12 months of filing the
provisional specification. This period can be extended by 3 months.
<detailed note elsewhere in this doc>
B. Complete Specification :
Submission of complete specification is necessary to obtain a patent.
The contents of a complete specification would include the following :
Title of the invention
Field to which invention belongs.
Background of the invention including prior art giving drawbacks of the
known inventions practices.
Complete description of the invention along with experimental results.
Drawings etc essential for understanding the invention.
Claims, which are statements, related to the invention on which legal
proprietorship is being sought. Therefore the claims have to be drafted very
carefully.
<detailed note elsewhere in this doc>
(ii) Essentials for obtaining patent :

(iii) Elements of Novelty, Non-obviousness :

CONTENTS | Module-1 | Module-2 | Module-3 | Module-4 http://duralex.bhatt.net.in/


https://www.facebook.com/groups/Dharmo.Rakshati.Rakshitah/ Page - 169 of 196


(iv) Opposition to the patent :
Rule 55 dealing with opposition by representation against the grant of patent
requires the representation for opposition under section 25 to be filed at the
appropriate office
within a period not exceeding three months from the date of publication of the
application under section 11A of the Act, or before the grant of patent,
whichever is later
Such representation shall also include a statement and evidence, if any, in support
of the representation and a request for hearing if so desired.
The Controller has been empowered to consider such representation only when a
request for examination of the application has been filed.
On consideration of the representation if the Controller is of the opinion that
application for patent shall be refused or the complete specification requires
amendment, he shall give a notice to the applicant to that effect.
On receiving the notice the applicant shall, if he so desires, file his statement and
evidence, if any in support of his application within one month from the date of the
notice.
On consideration of the statement and evidence filed by the applicant, the
Controller may either refuse to grant a patent on the application or require the
complete specification to be amended to his satisfaction before the patent is
granted.
After considering the representation and submission made during the hearing if so
requested, the Controller shall proceed further simultaneously
either rejecting the representation and granting the patent
or accepting the representation and refusing the grant of patent on that
application,
ordinarily within one month from the completion of above proceedings.
<for details of grounds read Sec-25(1) and Sec-25(2) from bare act>
Under Rule 57 the opponent is required to send a written statement in duplicate
setting out the nature of the opponents interest, the facts upon which he bases his
case and relief which he seeks and evidence, if any, along with notice of opposition
and to deliver to the patentee a copy of the statement and the evidence, if any.
Constitution of Opposition Board and its Proceeding :
Section 25(3) provides that where any such notice of opposition is duly given,
the Controller shall notify the patentee and constitute a Board by order in writing
to be known as the Opposition Board consisting of such officers as he may
determine and refer such notice of opposition along with the documents to that
Board for examination and submission of its recommendation.

CONTENTS | Module-1 | Module-2 | Module-3 | Module-4 http://duralex.bhatt.net.in/


https://www.facebook.com/groups/Dharmo.Rakshati.Rakshitah/ Page - 170 of 196

The Opposition Board shall conduct the examination of the notice of opposition
along with documents filed under rules 57 to 60 referred to under section 25(4),
and submit a report with reasons on each ground taken in the notice of
opposition with its joint recommendation within three months from the date on
which the documents were forwarded to them.

(v) Problem of limited locus standi to oppose, especially in relation to
inventions having potential of ecological and mass disasters :
(vi) Grant of patents :
Section 43 dealing with grant of patents provides that where an application for a
patent has been found to be in order for grant of the patent
and either the application has not been refused by the Controller by virtue of any
power vested in him by the Act,
or the application has not been found to be in contravention of any of the
provisions of the Act,
the patent shall be granted as expeditiously as possible to the applicant or, in
the case of a joint application, to the applicants jointly, with the seal of the
patent office
and the date on which the patent is granted shall be entered in the register.
The Controller has been put under obligation to publish the fact that the patent has
been granted and thereupon the application, specification and other documents
related thereto shall be open for public inspection.
Grant of patents subject to conditions :
As per Section 47 the grant of a patent shall be subject to the conditions that :
(1) any machine, apparatus or other article in respect of which the patent is
granted or any article made by using a process in respect of which the patent
is granted, may be imported or made by or on behalf of the Government for
the purpose merely of its own use;
(2) any process in respect of which the patent is granted may be used by or on
behalf of the Government for the purpose merely of its own use;
(3) any machine, apparatus or other article in respect of which the patent is
granted or any article made by the use of the process in respect of which the
patent is granted, may be made or used, and any process in respect of which
the patent is granted may be used by any person, for the purpose merely of
experiment or research including the imparting of instructions to pupils; and
(4) in the case of a patent in respect of any medicine or drug, the medicine or
drug may be imported by the Government for the purpose merely of its own
use or for distribution in any dispensary, hospital or other medical institution
maintained by or on behalf of the government or any other dispensary,

CONTENTS | Module-1 | Module-2 | Module-3 | Module-4 http://duralex.bhatt.net.in/


https://www.facebook.com/groups/Dharmo.Rakshati.Rakshitah/ Page - 171 of 196

hospital or other medical institution which the Central Government may,


having regard to the public service that such dispensary, hospital or medical
institution renders, specify in this behalf by notification in the official gazette.

This is Module-4. Menu ---> CONTENTS | Module-1 | Module-2 | Module-3 | Module-4

Discuss : Wrongfully obtaining the invention : Prior Publication or anticipation,


Obviousness and the lack of inventive step, Insufficient description.
ANSWER :
Refer :

This is Module-4. Menu ---> CONTENTS | Module-1 | Module-2 | Module-3 | Module-4

Explain under the Patent Act : Revocation of Patents (Oct-2013)


Discuss : Compulsory Licensing.
ANSWER :
Refer :
https://www.icsi.edu/docs/webmodules/Publications/9.4%20Intellectual
%20Property%20Rights.pdf
http://www.mondaq.com/india/x/607436/Licensing+Syndication/Compulsory+Lice
nse+Of+Copyright+Work
Intro :
Sec-85 deals with revocation of patents by Controller for not working and provides
that
where, in respect of a patent, a compulsory licence has been granted,
the Central Government or any person interested may, after the expiration of
two years from the date of the order granting the first compulsory licence,
apply to the Controller for an order revoking the patent on the ground that
the patented invention has not been worked in the territory of India or
reasonable requirements of the public with respect to the patented invention
has not been satisfied or
the patented invention is not available to the public at a reasonably affordable
price.
Every application for revocation should contain prescribed particulars, the facts
upon which the application is based,

CONTENTS | Module-1 | Module-2 | Module-3 | Module-4 http://duralex.bhatt.net.in/


https://www.facebook.com/groups/Dharmo.Rakshati.Rakshitah/ Page - 172 of 196

and, in the case of an application other than by the Central Government, should
also set out the nature of the applicants interest.
The Controller, if satisfied that
the reasonable requirements of the public with respect to the patented
invention has not been satisfied or
patented invention has not been worked in the territory of India or
is not available to the public at a reasonably affordable price,
may make an order revoking the patent.
The controller has however been put under obligation to ordinarily decide such
application within one year of its presentation.
Need for compulsory licensing :
If everybody is free to access new knowledge, inventors have little incentive to
commit resources to producing it.
However, IPR holder also has a duty to be sensitive to meet public needs, at
affordable price.
And in case he fails in this duty, then the legal regime shall provides for
compulsory licensing, or even revocation of patent (or other IPR), in extreme
cases.
Hence, in todays world, the rationale for protecting intellectual property is
essentially utilitarian (need to be useful or practical).
Accordingly, under international legal regime for protection of IPR there are a
provisions for compulsory licensing to assure public access to intellectual
properties
if the national interest requires it,
or if the IPR holder is unable to meet the demand.
During TRIP negotiations developed countries, including the US, had raised
concerns over developing countries like India liberally issuing the CL.
The 2002 Doha Declaration affirmed that the TRIPS agreement should not prevent
members from taking measures necessary to protect public health.
However, despite this recognition, less-developed countries have argued that
TRIPS's flexible provisions, such as compulsory licensing, are near-on impossible
to exercise.
In particular, less developed countries have cited their infant domestic
manufacturing and technology industries as evidence of the policy's bluntness.
Patents : Duties of patentee :
Section 83 dealing with general principles applicable to working of patented
invention provides that in exercising the powers conferred for working of patents
and compulsory licences, regard shall be had to the following general

CONTENTS | Module-1 | Module-2 | Module-3 | Module-4 http://duralex.bhatt.net.in/


https://www.facebook.com/groups/Dharmo.Rakshati.Rakshitah/ Page - 173 of 196

considerations, namely :
(a) that patents are granted to encourage inventions and to secure that the
inventions are worked in India on a commercial scale and to the fullest extent
that is reasonably practicable without undue delay;
(b) that they are not granted merely to enable patentees to enjoy a monopoly
for the importation of the patented article;
(c) that the protection and enforcement of patent rights contribute to the
promotion of technological innovation and to the transfer and dissemination of
technology, to the mutual advantage of producers and users of technological
knowledge and in a manner conducive to social and economic welfare, and to a
balance of rights and obligations;
(d) that patents granted do not impede protection of public health and nutrition
and should act as instrument to promote public interest specially in sectors of
vital importance for socio-economic and technological development of India;
(e) that patents granted do not in any way prohibit Central Government in
taking measures to protect public health;
(f) that the patent right is not abused by the patentee, and the patentee does
not resort to practices which unreasonably restrain trade or adversely affect the
international transfer of technology; and
(g) that patents are granted to make the benefit of the patented invention
available at reasonably affordable prices to the public.
Procedure for Dealing with Applications for revocation and compulsory License :
Sec-87 provides that where the Controller is satisfied, upon consideration of an
application for compulsory licence or revocation of patent, that a prima facie case
has been made out for the making of an order,
he shall direct the applicant to serve copies of the application upon the patentee
and any other person appearing from the register to be interested in the patent
in respect of which the application is made, and shall publish the application in
the Official Journal.
The patentee or any other person desiring to oppose the application may, within
prescribed time or within such further time as the Controller may on application
allow, give to the Controller notice of opposition.
Any such notice of opposition should contain a statement setting out the grounds
on which the application is opposed.
Where any such notice of opposition is duly given, the Controller shall notify the
applicant, and shall give to the applicant and the opponent an opportunity to be
heard before deciding the case.
Patents : Compulsory Licence :
As per the WTO norms, a Compulsory Licence can be invoked by a government
allowing a company to produce a patented product without the consent of the

CONTENTS | Module-1 | Module-2 | Module-3 | Module-4 http://duralex.bhatt.net.in/


https://www.facebook.com/groups/Dharmo.Rakshati.Rakshitah/ Page - 174 of 196

patent owner in public interest.


The provisions for compulsory licenses are made
to prevent the abuse of patent as a monopoly
and to make the way for commercial exploitation of the invention by an
interested person.
According to Section 84 of Indian Patents Act any person interested can make an
application for grant of compulsory license for a patent after three years from
the date of grant of that patent on any of the following grounds
(a) that the reasonable requirements of the public with respect to the patented
invention have not been satisfied, or
(b) that the patented invention is not available to the public at a reasonably
affordable price, or
(c) that the patented invention is not worked in the territory of India.
Under the Indian Patents Act 1970, a CL can be issued for a drug if the medicine
is deemed unaffordable, among other conditions, and the government grants
permission to qualified generic drug makers to manufacture it.
Illustration :
India has issued only CL for a cancer drug.
In the case of Natco Pharma Limited v. Bayer Corporation, the Indian Patents
Office granted its first compulsory license, for the manufacture and sale of
Bayers patented drug Nexavar.
In this case Bayers inability to make Nexavar available to nearly 98 per
cent of the Indian public was held by the Controller to amount to a failure to
satisfy the reasonable requirements of the public.
This case set the precedent for making expensive patented drugs available
for compulsory licensing under the Patents Act.
Termination of Compulsory Licence :
Section 94 provides that on an application made by the patentee or any other
person deriving title or interest in the patent,
a compulsory licence may be terminated by the controller,
provided the circumstances that give rise to the grant thereof no longer
exist and such circumstances are unlikely to recur.
In this regard the holder of the compulsory licence has been entitled to object
to such termination.
Copyrights :
Compulsory Licences has an application in copyrights as well.
What happens when the owner of a work after its publication has refused to re-
publish the work or allowed re-publication of the work.

CONTENTS | Module-1 | Module-2 | Module-3 | Module-4 http://duralex.bhatt.net.in/


https://www.facebook.com/groups/Dharmo.Rakshati.Rakshitah/ Page - 175 of 196

Section 31 of the Copyright Act 1957 enables a complaint to be filed before the
Copyright Board, which jurisdiction is now vested with the Intellectual
Property Appellate Board, when such refusal has resulted in the work being
withheld from the public.
Similarly, when the owner of the work, has refused to allow communication of
the work by broadcast or that of a sound recording on terms which the
complainant considers not to be reasonable, a complaint can be filed before the
Board.
The Board will hear the owner of the work and on determination of a cause
direct Registrar of Copyright to grant license to qualified persons subject to
payment of compensation to the owner of the work.
Under Section 31 (A) of the Copyright Act 1957, when a work is unpublished, or
published but the author is dead or unknown or not traceable,
any person may apply to the Board for exploitation of the work. The Board
after due notice and hearing, if satisfied, direct the Registrar to grant license.
Likewise, Section 31B (1) provides that any person working for the benefit of
persons with disability on a profit basis or for business may apply to the
Copyright Board in prescribed manner for a compulsory licence to publish any
work in which copyright subsists for the benefit of such persons.
Conclusion :
The principle of conditional grants to proprietary rights in any intellectual property
is to promote public interest. This is universally recognized and incorporated in
intellectual property system.
Protection and enforcement of intellectual property rights must :
be conducive to social and economic welfare;
safeguard an individuals fundamental rights; and
Promote commerce, competition and innovation.

This is Module-4. Menu ---> CONTENTS | Module-1 | Module-2 | Module-3 | Module-4

Write note : Lapsed Patent. (Nov-2011)


Discuss : Duration of patents.
ANSWER :
Refer :
https://www.icsi.edu/docs/webmodules/Publications/9.4%20Intellectual
%20Property%20Rights.pdf
Patents Act, 1970
Patents Rules, 1972

CONTENTS | Module-1 | Module-2 | Module-3 | Module-4 http://duralex.bhatt.net.in/


https://www.facebook.com/groups/Dharmo.Rakshati.Rakshitah/ Page - 176 of 196

Duration/ term of patents :


Section 53 provides that the term of every patent granted after the
commencement of the Patents (Amendment) Act, 2002 and the term of every
patent which has not expired and has not ceased to have effect, on the date of
such commencement,
shall be twenty years from the date of filing of application for the patent.
Explanation to Section 53(1) clarifies that the term of patent in case of
international applications filed under the PCT designating India, shall be twenty
years from the international filing date accorded under the Patent Cooperation
Treaty.
A patent shall cease to have effect on the expiration of the period prescribed for
the payment of any renewal fee, if that fee is not paid within the prescribed period
or within such extended period as may be prescribed.
Further on cessation of the patent right due to non-payment of renewal fee or on
expiry of the term of patent, the subject matter covered by the said patent shall
not be entitled to any protection.
Rule 80 requires that to keep a patent in force, the renewal fees specified in the
First Schedule should be paid at the expiration of the second year from the date of
the patent or of any succeeding year and the same should be remitted to the
patent office before the expiration of the second or the succeeding year.
Sub-rule (1A) inserted by Patents (Amendment) Rules, 2005 provides that the
period for payment of renewal fees may be extended to such period not being more
than six months if the request for such extension of time is made in Form 4 with
the fee specified in the First Schedule.
While paying the renewal fee, the number and date of the patent concerned and
the year in respect of which the fee is paid is required to be quoted.
The annual renewal fees payable in respect of two or more years may be paid in
advance.
Restoration of lapsed patents :
Section 60 provides that where a patent has ceased to have effect by reason of
failure to pay any renewal fee within the period prescribed under section 53 or
within period as may be allowed under section 142(4),
the patentee or his legal representative and where the patent was held by two or
more persons jointly, then with the leave of the Controller one or more of them
without joining the others, may within eighteen months from the date on which
the patent ceased to have effect, make an application for the restoration of the
patent.
Rights of Patentee of Lapsed Patents which have been Restored :
Section 62 provides that where a patent is restored, the rights of the patentee shall
be subject to such conditions as may be prescribed and to such other conditions as

CONTENTS | Module-1 | Module-2 | Module-3 | Module-4 http://duralex.bhatt.net.in/


https://www.facebook.com/groups/Dharmo.Rakshati.Rakshitah/ Page - 177 of 196

the Controller thinks fit to impose for the protection or compensation of


persons who may have begun to avail themselves of, or have taken definite
steps by contract or otherwise to avail themselves of, the patented invention
between the date when the patent ceased to have effect and the date of the
publication of the application for restoration of the patent.
No suit or other proceeding shall be commenced or prosecuted in respect of an
infringement of a patent committed between the date on which the patent ceased
to have effect and the date of the publication of the application for restoration of
the patent.

This is Module-4. Menu ---> CONTENTS | Module-1 | Module-2 | Module-3 | Module-4

Explain in detail : Rights and obligations of a patentee (Apr-2016)


Discuss : (i) Patents as chose in action, (ii) Duration of patents law and policy
considerations, (iii) Use and exercise rights, (iv) Right to secrecy , (v) "abuse"
of patent rights, (vi) Patent Assignment/License (vii) Special Categories, (viii)
Employee Invention Law and Policy Consideration
Discuss : Transfer of Technology, Know-How and problems of self reliant
development.
Discuss under the Patent Act, 1970 : Rights of Patentee. (Nov-2012)
Discuss : Compulsory licence for benefit of disabled, (Nov-2014)
ANSWER :
Refer :
http://epgp.inflibnet.ac.in/epgpdata/uploads/epgp_content/law/08._intellectual_pr
operty_law/27._intellectual_property_assignment_and_licensing_/et/5795_et_27_
et.pdf
https://www.icsi.edu/docs/webmodules/Publications/9.4%20Intellectual
%20Property%20Rights.pdf
(i) Patents as chose in action :

(ii) Duration of patents law and policy considerations :

(iii) Use and exercise rights :

(iv) Right to secrecy :

(v) "abuse" of patent rights :

CONTENTS | Module-1 | Module-2 | Module-3 | Module-4 http://duralex.bhatt.net.in/


https://www.facebook.com/groups/Dharmo.Rakshati.Rakshitah/ Page - 178 of 196


(vi) Patent Assignment/License : Transfer of technology and know-how :
http://epgp.inflibnet.ac.in/epgpdata/uploads/epgp_content/law/08._intellectual_pr
operty_law/27._intellectual_property_assignment_and_licensing_/et/5795_et_27_
et.pdf
Licensing of a patent could obviously be done upon the issuance of the patent but it
could also be attempted during the pendency of the patent application.
A contract for licensing or assignment has to be in writing and has to be reduced to
the form of an executed document which includes all the terms and conditions
governing the rights and obligations of the parties.
Failure to comply with any of these conditions will render the license or assignment
invalid.
Further, the document must be registered by filing an application in the prescribed
manner with the Controller of Patents.
In order to ensure that the employer has the rights to exploit the patent rights in
the invention, most employers, in actual practice, require employees to sign
contracts to assign or license the future inventions to the employers.
Assignment :
Assignment refers to the act of the patentee by which the patent rights are
wholly or partially transferred to the assignee who acquires the right
to prevent others from making, using or exercising or vending the invention.
Section 70 of the Patents Act, 1970 gives the person/persons, persons
registered as grantee or proprietor of a patent,
power to assign, grant licences under, or otherwise deal with, the patent
and to give effectual receipts for any consideration for any such assignment,
licence or dealing.
The assignment can either be exclusive or non exclusive.
The exclusivity can be further limited, for example exclusivity to a territory or
market or line of products.
Following are three main types of assignments in patents :
Legal Assignments :
An assignment of an existing deed is a legal assignment.
A patent which is created by deed can only be assigned by a deed.
A legal assignee is entitled to be registered as the proprietor of the patent
and acquires all the rights thereof.
Equitable Assignments :
A document agreeing to transfer a patent or a share of a patent with
immediate effect is an equitable assignment.

CONTENTS | Module-1 | Module-2 | Module-3 | Module-4 http://duralex.bhatt.net.in/


https://www.facebook.com/groups/Dharmo.Rakshati.Rakshitah/ Page - 179 of 196

This affects proprietorship, but does not directly change it.


The man to whom it is equitably assigned gets the right in equity to have
the ownership of the patent altered in law.
Mortgages :
A mortgage is a document through which patent rights are transferred to
the assignee in return for a sum of money.
Once the assignor repays the sum, the patent rights are restored to him.
The term assignee as per Section 2(1) of the Patents Act, 1970 includes in its
meaning the legal representative of a deceased assignee.
Section 70 of the Patents Act, 1970 confers inter alia the right on a grantee or
proprietor of the patent
to fully or partially assign his patent to another or others.
As per Section 68 of the Patents Act, 1970 an assignment to be valid
shall be in writing,
to be contained in a document that embodies all terms and conditions
governing their rights and obligations
and the application for registration of such document is filed in the prescribed
manner with the Controller within six months from the commencement of the
Act or the execution of the document, whichever is later.
Section 69 says once the person becomes entitled by assignment to a patent he
shall apply in writing to the Controller for the registration of his title in the
prescribed manner.
Licensing :
An important aspect of patent licensing is a cross license wherein both parties
of a prospective license contract have patent rights which the other party wishes
to acquire.
It, therefore, means mutual sharing of patents between patent owners.
Depending on the patent strategy of particular companies, entire patent
portfolios containing thousands of patents may be cross licensed.
Such practice is common in computer hardware and software industries where
on account of the presence of patent thickets companies can require thousands
of patents to produce a single product.
Cross licensing must be distinguished from a patent pool which is another
vehicle for providing access to patented material.
Patent pools are created when multiple patents from many patentees are
packaged and then licensed to a third party.
The purpose of cross licenses is usually unblocking technology of each party so
that each can use the same without the threat of litigation.

CONTENTS | Module-1 | Module-2 | Module-3 | Module-4 http://duralex.bhatt.net.in/


https://www.facebook.com/groups/Dharmo.Rakshati.Rakshitah/ Page - 180 of 196

Cross licensing is necessitated because of two way blocking relationship of


patents where it becomes impossible to work a patent without infringing each
others patent.
Cross licensing is also used to enable enterprises to settle IP disputes.
There are no inherent difficulties in cross licensing, yet a potential problem could
arise if the effect of the cross license affects competition in a market.
Under a compulsory license, an individual or company seeking to use another's
patent can do so without seeking the owners consent, and pays the rights
holder a set fee for the license which is determined by the state.
The following general purposes guide the Controller of Patents in granting
compulsory licenses :
(a) that patented inventions are worked on a commercial scale in the territory
of India without undue delay and to the fullest extent that is reasonably
practicable;
(b) that the interests of any person for the time being working or developing
an invention in the territory of India under the protection of a patent are not
unfairly prejudiced.
In March 2012, India granted a compulsory license to a generic drug
manufacturer Natco Pharma Ltd. to manufacture Sorafenib Tosylate, a cancer
drug patented by Bayer.
(vii) Special Categories :

(viii) Employee Invention Law and Policy Consideration :

This is Module-4. Menu ---> CONTENTS | Module-1 | Module-2 | Module-3 | Module-4

Discuss : Patent Co-operation treaty : Object, reasons and important provisions.


ANSWER :
Refer :

This is Module-4. Menu ---> CONTENTS | Module-1 | Module-2 | Module-3 | Module-4

Discuss : International Patents.


ANSWER :

CONTENTS | Module-1 | Module-2 | Module-3 | Module-4 http://duralex.bhatt.net.in/


https://www.facebook.com/groups/Dharmo.Rakshati.Rakshitah/ Page - 181 of 196

Refer :

This is Module-4. Menu ---> CONTENTS | Module-1 | Module-2 | Module-3 | Module-4

Discuss : Exceptions & Limitations To Patent Rights In India.


ANSWER :
Refer :
http://www.mondaq.com/india/x/325620/Patent/EXCEPTIONS+LIMITATIONS+TO+
PATENT+RIGHTS+IN+INDIA
Intro :
In this knowledge based economy the understanding of Intellectual Property Rights
(IPRs) is crucial to promote innovation and growth of the country.
The various forms of IPRs are frequently talked about in debates and general
conversations on such diverse topics as
industrial policy, traditional knowledge, agriculture, biotechnology, medical
devices, education, food security, entertainment and mass media.
The omnipresence of IPRs in the areas of human development has made it critical
to access the benefits and encumbrances it carries.
For this purpose countries adopted exceptions to patent rights and other related
rights to use IPRs as a tool for their sustainable development.
However, after signing of the World Trade Organization's Agreement on Trade-
Related Aspects of Intellectual Property Rights (TRIPS), the exceptions and
limitations to patent rights in India underwent something of an evolution.
The 'Exceptions to Rights Conferred' has been incorporated as Article 30 of TRIPS.
It provides that 'Member Countries may provide limited exceptions to the
exclusive rights conferred by a patent, provided that
such exceptions do not unreasonably conflict with a normal exploitation of the
patent
and do not unreasonably prejudice the legitimate interests of the patent
owner, taking account of the legitimate interests of third parties.
Types of Exceptions & Limitations :
Article 30 of TRIPS allows for limited exceptions to the exclusive rights conferred by
a patent. The exceptions must not unfairly prejudice the legitimate interests of the
patent owner.
1. Private and Non-commercial Use Exception
2. Experimental / Scientific Use Exception

CONTENTS | Module-1 | Module-2 | Module-3 | Module-4 http://duralex.bhatt.net.in/


https://www.facebook.com/groups/Dharmo.Rakshati.Rakshitah/ Page - 182 of 196

3. Regulatory-use/ Prior-use Exemption


4. Foreign Vessels, Aircraft or Land Vehicles Exception
5. Exhaustion of Patent Rights
1. Private and Non-commercial Use Exception :
The exclusive rights conferred by a patent do not allow private use or monopoly
over commercial activity.
If a patentee is neither using nor vending the invention for profit, the Government
has the power to grant a license, known as Compulsory License (CL), to a third
party to use the patented invention.
The concept of CL comes into play when
the patented invention is not commercialized in India
or the invention is not available to the public at reasonable prices
or the invention is not manufactured in requisite amount.
The provisions of private and non-commercial use exception have been provided
under following section of Patents Act, 1970 :
Section 84 (Compulsory licenses),
Section 85 (Revocation of patents by the Controller for non-working)
and Section 92 (Special provision for compulsory licences on notifications by
Central Government).
2. Experimental / Scientific Use Exception :
The experimental use exemption is incorporated under Section 47(3) of the Act.
Accordingly the grant of a patent is subject to the condition that
any product or process, in respect of which the patent is granted, may be made
or used by any person for the purpose merely of experiment or research
including the imparting of instructions to pupils.
This is one of the most widely known exceptions to patent rights and it grew up out
of the concern that patent rights should not hamper the "bona fide" experiments
and scientific processes.
This form of experimental use exception permits third parties to carry out
experimental or scientific activities relating to the subject matter of the patent
without infringing the patent holder's rights.
3. Regulatory-use/ Prior-use Exemption :
The patent rights, on the one hand, provide economic incentives to innovate, but
on the other hand, the exclusive rights they confer result into monopoly and
unaffordable pharmaceutical products.
Consequently, the Indian Patents (Amendment) Act, 2005 incorporated the
regulatory-use or prior-use exemption under section 107A to offer a trade-off
between incentives to the innovators, and limited access and costs to consumers.

CONTENTS | Module-1 | Module-2 | Module-3 | Module-4 http://duralex.bhatt.net.in/


https://www.facebook.com/groups/Dharmo.Rakshati.Rakshitah/ Page - 183 of 196

This provision allows the generic producers to market and manufacture their goods
as soon as the patent term expires.
However, this does not allow for the use of such patented IP to obtain the
license to manufacture and distribute the generic drug before the expiry of the
term of patent.
This exemption is also referred as Bolar Provision and is a statutorily created
exemption to patent rights that allows the manufacturers of generic drugs to
undertake steps reasonably related to the development and submission of
information required for obtaining marketing approval anywhere in the world in
respect of a patented product without the consent of the patentee.
4. Foreign Vessels, Aircraft or Land Vehicles Exception :
As per Article 5 of the Paris Convention,
rights conferred by a patent shall not extend to the use of the patented invention
on board of vessels when such vessels temporarily or accidentally enter the
waters,
provided that the invention is used exclusively for the needs of the vessel .
This exception is beneficial in facilitating uninterrupted international travel and
reducing tensions between countries over the treatment of vessels flying their flag.
As this exception is not optional for countries party to the Paris Convention, Indian
Patents Act, in order to comply, incorporated the said exception under Section 49.
According to the said section, the patent rights are not infringed when the patented
invention is used exclusively for the needs of foreign vessels, aircraft, or land
vehicles and other accessories thereof, when such foreign vessels, aircraft, or land
vehicles temporarily or accidentally comes into India.
The term "Temporarily" not only includes the accidental and unintentional entry but
also the intentional and regular going into a port, provided that the vessels,
aircraft, or land vehicles does not remain permanently in the territorial waters or
the territory of the country.
5. Exhaustion of Patent Rights :
The Doctrine of Exhaustion or First Sale Doctrine refers to
the exhaustion of the exclusive rights of the patent holder once the patented
invention is sold without any restriction.
As per this Doctrine, the first UNRESTRICTED sale of a patented item exhausts the
patentee's further control over that particular item.
The rationale behind the patent holder exhausting their rights once they have sold
the patented product is that,
by first sale of the patent invention the Patent holder
has already used the exclusive rights to prevent others from making, using,
selling, offering for sale in the territory of patent grant or importing an

CONTENTS | Module-1 | Module-2 | Module-3 | Module-4 http://duralex.bhatt.net.in/


https://www.facebook.com/groups/Dharmo.Rakshati.Rakshitah/ Page - 184 of 196

invention into the territory of patent grant


and therefore has already reaped the benefits conferred by a patent.
Conclusion :
A Patent on an invention confers right on the patentee to exclude others
from making, using, selling, offering for sale
in the territory of patent grant or importing an invention into the territory of
patent grant
for a limited time, in return for the public disclosure of the invention.
This might have led to monopoly and uncompetitive practices.
Therefore, the TRIPS agreement introduced substantive provisions on exceptions
under Article 30 for the sustainable growth and development of the developing
countries.
Article 30 of TRIPS, provides that :
"Members may provide limited exceptions to the exclusive rights conferred by a
patent,
provided that such exceptions do not unreasonably conflict with a normal
exploitation of the patent
and do not unreasonably prejudice the legitimate interests of the patent
owner,
taking account of the legitimate interests of third parties."
However, if a policy maker of any of the member countries wishes to include a new
exception, in order for it to be valid, it must meet the criterion laid under Article 30
of TRIPS.

This is Module-4. Menu ---> CONTENTS | Module-1 | Module-2 | Module-3 | Module-4

Discuss suits for infringement of Patent under the Patents Act, 1970. (Nov-2011)
Discuss : (i) Criteria of infringement, (ii) Modes of infringement, (iii)
Jurisdiction, (iv) Onus of Proof in India, (v) Doctrine of equivalent, (vi)
Doctrine of Colorable Variation.
ANSWER :
Refer :
http://corporatelawreporter.com/2016/12/14/patent-infringement/
Intro :
Patent infringement typically is caused by using or selling a patented invention
without permission from the patent holder.
The scope of the patented invention or the extent of protection is defined in the

CONTENTS | Module-1 | Module-2 | Module-3 | Module-4 http://duralex.bhatt.net.in/


https://www.facebook.com/groups/Dharmo.Rakshati.Rakshitah/ Page - 185 of 196

claims of the granted patent.


Patent right is available only for a limited period of time. However, the use or
exploitation of a patent may be affected by other laws of the country which has
awarded the patent.
In general, patent infringement cases are handled under civil law (e.g., in the
United States) but several jurisdictions incorporate infringement in criminal law
also (for example, Argentina, China, France, Japan, Russia, South Korea).

(i) Criteria of infringement :
It is the unauthorized making, using, offering for sale or selling any patented
invention within India or importing into India of any patented invention during the
term of the patent.
If any person exercises the exclusive rights of the patent holder without the patent
owners authorization then that person is liable for patent infringement.
Sections 104 to 114 of the Patents Act, 1970 provide guidelines relating to patent
infringement.
Unlike other intellectual property laws, the patents law does not specify as to what
constitutes infringement of a patented product or process.
However, the following acts when committed without the consent of the patentee
shall amount to infringement.
1. making, using, offering for sale, selling, importing the patented product;
2. using the patented process, or using, offering for sale, selling or importing the
product directly obtained by that process.
(ii) Modes of infringement :
There are two types of patent infringement i.e. Direct and Indirect Patent
Infringement.
1. Direct Patent Infringement :
The most common form of infringement is the direct infringement. Where the
claims of the patent literally describe the infringing invention or the invention
performs substantially the same function.
In other words, when a product that is substantially close to a patented product
or invention is marketed, sold, or used commercially without a permission form
the owner of the patented product or invention.
2. Indirect Patent Infringement :
The indirect infringement is further divided into two types :
(i) Infringement by Inducement : Infringement by inducement is any activity
by a third party that causes another person to directly infringe the patent. This
can include selling parts that can only be realistically used for a patented
invention, selling an invention with instructions on using in a certain method

CONTENTS | Module-1 | Module-2 | Module-3 | Module-4 http://duralex.bhatt.net.in/


https://www.facebook.com/groups/Dharmo.Rakshati.Rakshitah/ Page - 186 of 196

that infringes on a method patent or licensing an invention that is covered by


anothers patent.
The inducer must knowingly aid the infringement, but intent to infringe on
the patent is not required.
(ii) Contributory Infringement : Contributory infringement is the selling of
material components that have been made for use in a patented invention and
have no other commercial use. There is a significant overlap with inducement,
but contributory infringement requires a higher level of guilt.
To be contributory infringement, the seller must intend for the direct
infringement to occur.
In order for there to be liability for indirect infringement, there must also be
direct infringement resulting from the indirect act.
Exclusion from Infringement :
The law enumerates certain exception to the infringement.
Experimental and Research :
Any patented article or process can be used for the following purposes :
Experiment
Research
Education
It is also permitted to make, construct, use, sell or import a patented invention
solely for the users reasonably related to the development and submission of
information required under any law for the time being in force, in India, or in a
country other than India, that regulates the manufacture, construction, use,
sale or import of any product.
All such acts, if within the bounds as created above, cannot be challenged as
infringing the rights of the patentee.
Parallel Importation under certain conditions :
Patented article or article made by using the patented process can be imported
by government for its own use.
Also a patented process can be used by the government solely for its own use.
Moreover the government can import any patented medicine or drug for the
purpose of its own use or for the distribution in any dispensary, hospital or
other medical institution maintained by the government or any other
dispensary, hospital or medical institution notified by the government.
(iii) Jurisdiction :
The legal provisions with regard to jurisdiction are provided in Section 104 of the
Patents Act, 1970.
Types of Patent related cases : The courts in India receive

CONTENTS | Module-1 | Module-2 | Module-3 | Module-4 http://duralex.bhatt.net.in/


https://www.facebook.com/groups/Dharmo.Rakshati.Rakshitah/ Page - 187 of 196

(a) Patent Administrative Cases, where the Indian Patent Office is the defendant.
These types of cases include dispute on grant of a patent, patent invalidation
and upholding and compulsory licensing.
(b) Patent Infringement Cases, where the patentee or patent assignee pursue
damages against willful infringement conduct by the alleged infringer.
These include infringement of patent, disputes relating to ownership of patent,
disputes regarding patent rights or right for application, patent contractual
disputes, and contractual disputes of assignment right, patent licensing and
dispute relating to revocation of patents.
Section 104 of the Patents Act, 1970 says that
the infringement suit shall not be instituted in a court lower than the District
Court in India.
Further, if the defendant files a counter-claim against revocation of the patent,
then the suit along with the counter-claim shall be transferred to the High Court
for decision.
Like any other civil suit the jurisdiction shall be determined in accordance with rules
of the Code of Civil Procedure. Accordingly the appropriate forum would be :
1. Principal place where the plaintiff carries on his business; or
2. Principal place where the defendant carries on his business; or
3. Place where the infringing articles are manufactured or sold or infringing
process is being applied or where the articles manufactured by the infringing
process are sold.
Period of Limitation :
The period of limitation for instituting a suit for patent infringement is three
years from the date of infringement.
(iv) Onus of Proof in India :
The traditional rule of burden of proof is adhered to with respect to patented
product.
Accordingly in case of alleged infringement of a patented product the onus of
proof rests on the plaintiff.
However, TRIPS prompted amendment inserted by way of Sec-104(A) has
reversed burden of proof in case of infringement of patented process.
Under the current law, the Court can at its discretion shift the burden of proof on
the defendant, in respect of process patent if either of the following two conditions
is met :
1. The subject matter of the patent is a process for obtaining a new product; or
2. There is substantial likelihood that an identical product is made by the process
and the plaintiff/ patentee has made reasonable efforts to determine the process
actually used but has failed

CONTENTS | Module-1 | Module-2 | Module-3 | Module-4 http://duralex.bhatt.net.in/


https://www.facebook.com/groups/Dharmo.Rakshati.Rakshitah/ Page - 188 of 196

In considering whether a party has discharged the burden imposed upon him under
Section 104(A), the Court shall not require him to disclose any manufacturing or
commercial secrets, if it appears to the Court that it would be unreasonable to do
so.
(v) Doctrine of equivalent :
Patent infringement generally falls into two categories :
A. Literal infringement :
The term literal infringement means that each and every element recited in
claim has identical correspondence in the allegedly infringing device or
process.
B. Infringement under the doctrine of equivalent :
Even if there is no literal infringement, a claim may be infringed under the
doctrine of equivalents if some other element of the accused device or process
performs substantially the same function, in substantially the same way, to
achieve substantially the same result.
The doctrine of equivalents is a legal rule in most of the worlds patent systems
that allows a Court to hold a party liable for patent infringement even though the
infringing device or process does not fall within the literal scope of patent claim,
but nevertheless is equivalent to the claimed invention.
The expansion of the claim coverage permitted by the doctrine of equivalents,
however, is not unbounded.
The scope of coverage which is afforded to the patent owner is limited by (i) the
doctrine of prosecution history estoppel and (ii) the prior art.
An infringement analysis determines whether a claim in a patent
literally reads on an accused infringers device or process,
or covers the allegedly infringing device under the doctrine of equivalents.
The steps in the analysis are :
Construe the scope of literal language of the claims.
Compare the claims, as properly construed with the accused device or process to
determine whether there is literal infringement.
IF there is no literal infringement,
THEN, construe the scope of the claims under the doctrine of equivalents.
The doctrine of equivalents is an equitable doctrine which effectively expands the
scope of the claims beyond their literal language to the true scope of the inventors
contribution to the art.
However, there are limits on the scope of the equivalents to which the patent
owner is entitled.
(vi) Doctrine of Colorable Variation/ Alteration :

CONTENTS | Module-1 | Module-2 | Module-3 | Module-4 http://duralex.bhatt.net.in/


https://www.facebook.com/groups/Dharmo.Rakshati.Rakshitah/ Page - 189 of 196

A colorable variation or immaterial variation amounting to infringement is where


an infringer makes slight modification in the process or product but in fact takes
in substance the essential features of the patentees invention.
It refers to the small modification or a change made to an invention and such
alteration is made only to distinguish an invention or work form an existing patent.
In Lekhophone Corporation v. Rola Company, 282 U.S. 168 (1930),
a patent holders patent were of sound-reproducing instruments for
phonographs.
According to the patent application, size an dimensions of the invention were the
essence of the patent.
That patent holder claimed that a radio loud speaker manufactured by the
defendant manufacturer infringed the patents.
The manufacturers device also had a central paper cone, but the cone was
smaller than that of the patented device and that constituted colorable
alteration.
The court held that because colorable alterations of the manufacturer's devise, it
would not accomplish the object specified in the patent claims and hence did not
infringe upon the patent holder's claims.

This is Module-4. Menu ---> CONTENTS | Module-1 | Module-2 | Module-3 | Module-4

Explain in detail : Remedies in case of infringement of the patent. (Apr-2016)


Discuss : (i) Injunctions and related remedies, (ii) Defense in suits of
infringement, (iii) Penal Provisions.
ANSWER :
Refer :
http://epgp.inflibnet.ac.in/epgpdata/uploads/epgp_content/law/08._intellectual_pr
operty_law/39._patent_law_-_remedies/et/8098_et_et.pdf
Intro :
Patent protection is generally sought in order to secure and preserve the
competitive and commercial advantage that may result from an invention, as well
as to recoup the cost incurred in developing an invention.
Patent rights are of limited value unless they are enforced to deter potential
infringers and to provide a remedy for a person whose rights have been infringed.
Nonetheless, a patent holder typically makes strategic decisions about the best use
of its resources in enforcing its rights.
PATENT RIGHTS :
The rights of a patent holder (patentee or a patent owner) are enshrined in

CONTENTS | Module-1 | Module-2 | Module-3 | Module-4 http://duralex.bhatt.net.in/


https://www.facebook.com/groups/Dharmo.Rakshati.Rakshitah/ Page - 190 of 196

Section 48 of the Patents Act 1970, which are subject to certain other provisions
of the Act including Section 47, Section 49, Section 100 and Section 107A.
SUMMARY :
The Patents Act provides remedies to the patent owner, the patent applicant and
also to a third party who may be threatened with infringement action.
The remedies to patent owners are based on certain rights granted under Section
48 of the Patents Act subject to certain exceptions.
The Act also offers remedies on the infringement of these rights.
The remedies include temporary injunction, permanent injunction, damages or
account of profits.
The suit may be filed in a civil court not inferior to the district court, except where
a counterclaim of revocation is filed by the defendant to the suit, in which case the
entire suit shall be transferred to the High Court along with the counterclaim of
revocation.
However injunction is not a statutory right but merely an equitable remedy.
Interim relief is available to prevent infringing goods from entering the channels of
trade, pending the resolution of litigation.
The patent applicant and third party also have certain remedies under the act
especially when threatened with a suit of patent infringement.
WHO can avail of remedies?
The remedies available to a party under the Act are directly dependent on the issue
at hand.
The parties who may avail of remedies under the Patent Act are any persons who
are aggrieved by the patent law provisions.
These persons may be broadly classified into three categories namely,
the patent owner,
the patent applicant
and the third party aggrieved by the effect of patent or likely to be aggrieved by
the possible effect of patent application when granted.
WHEN can remedies be granted?
Following are the situations in which remedy may be sought by a person aggrieved
under the provisions of the Act :
1. Infringement of the patent right of the patent holder
2. Invalidity of patent claims including pre-grant opposition, post grant
opposition, revocation and counter-claim for revocation
3. Declaration of non-infringement
4. Groundless threats of infringement
There are also situations in which a patent applicant or a third person may seek

CONTENTS | Module-1 | Module-2 | Module-3 | Module-4 http://duralex.bhatt.net.in/


https://www.facebook.com/groups/Dharmo.Rakshati.Rakshitah/ Page - 191 of 196

remedy to correct a situation or to counter an allegation or a assumption. These


situations include
5. Failure to obtain foreign filing licence under Section 39
6. Falsification of entries in the register
7. Unauthorized claim to patent rights
8. Practice by a non-registered patent agent
The remedies provided in the law are many and attempt to address each of the
situation above.
INFRINGEMENT :
The term infringement is not defined in the Act.
Any action that infringes the right of the patent holder and is not within the
exceptions shall be deemed to be an infringement of the rights of the patent holder
over his invention.
(i) REMEDIES :
The remedies in a suit for infringement are enshrined in Section 108.
Section 108(1) grants the reliefs of injunction and at the option of the plaintiff
either damages or accounts of profits.
Section 108(2) states that the Court may also order the infringing goods to be
seized and / or destroyed without payment of any compensation to the defendant.
Definition Sec-108 :
(1) The reliefs which a court may grant in any suit for infringement include an
injunction (subject to such terms, if any, as the court thinks fit)
and, at the option of the plaintiff, either damages or an account of profits
(2) The court may also order that the goods which are found to be infringing and
materials and implements,
the predominant use of which is in the creation of infringing goods shall be
seized, forfeited or destroyed, as the court deems fit under the circumstances
of the case without payment of any compensation
The court may grant preliminary / temporary injunction during the hearing of the
infringement suit to restrain the alleged infringer / defendant from infringing the
patent during the pendency of the suit.
The defendant may file an application for the vacation of the injunction.
However, if infringement is established after the trial the court may order a
permanent injunction.
Injunction is not a statutory right but an equitable remedy
and granted by the Courts in situations where the plaintiff / patent owner can
establish a prima facie case of success on the merits of the case, that there
would be irreparable injury to the patent owner and that the balance of

CONTENTS | Module-1 | Module-2 | Module-3 | Module-4 http://duralex.bhatt.net.in/


https://www.facebook.com/groups/Dharmo.Rakshati.Rakshitah/ Page - 192 of 196

convenience is in favour of the plaintiff / patent owner


i.e. the threatened harm to the plaintiff if the injunction is not granted far
outweighs the harm the injunction may inflict on the defendant.
The court may also order that the goods which are found to be infringing and
materials and implements, the predominant use of which is in the creation of
infringing goods shall be seized, forfeited or destroyed, as the court deems fit
under the circumstances of the case without payment of any compensation.
INVALIDATION/ REVOCATION OF PATENT CLAIMS :
Both a patent application as well as a granted patented can be subject to scrutiny
by a third party.
The Act provides for Pre-grant Opposition under S.25(1)
The Act provides for three situations to revoke the patent, each in different
forum :
a. The post-grant opposition under S. 25(2) by an interested party at the Patent
Office to be initiated within 12 months from the date of grant of patent
b. The revocation petition under S.64 by an interested party at the IPAB anytime
during the life of a patent
c. The counter-claim for revocation using any of the grounds under S.64 by the
Defendant in a suit for infringement in the court usually filed with the Written
Statement
After publication of the patent application on the expiry of 18 months from the date
of filing,
any third party may file a pre-grant opposition by way of a written
representation under S.25(1).
The 9 grounds for the pre-grant opposition are similar to those of the post-grant
opposition and include grounds that
the invention was anticipated and lacks novelty,
the invention was publicly known,
it is obvious to a person skilled in the art,
it is not a patentable subject matter under the Act,
the complete specification does not completely and clearly describe the
invention,
The grounds for pre-grant and post grant opposition are almost identical except
that
any person can file an pre-grant opposition by way of a written representation
before the patent is granted
while in case of a post grant opposition, any person interested may file the
opposition within one year from the grant of a patent.

CONTENTS | Module-1 | Module-2 | Module-3 | Module-4 http://duralex.bhatt.net.in/


https://www.facebook.com/groups/Dharmo.Rakshati.Rakshitah/ Page - 193 of 196

While the grounds of opposition are same,


the claims in a patent application and claims the granted patent following patent
prosecution may differ in the extent and scope of the invention claimed as a
patent.
In some cases the claims have been considerably revised and / or narrowed during
patent prosecution thereby opening fresh grounds for post-grant opposition.
Declaration as to Non-Infringement :
This remedy is available to persons who are not patent owners, and may either be
sued for infringement.
Under Section 105 of the Act, any person after the grant of publication of patent
may institute a suit for a declaration as to non-infringement.
For this the plaintiff must show that
he applied in writing to the patentee or his exclusive licensee for a written
acknowledgement to the effect that the process used or the article produced by
him does not infringe the patent
and, patentee or the licensee refused or neglected to give such an
acknowledgment.
It is not necessary that the plaintiff must anticipate the infringement suit.
Power of the Court to grant relief on groundless threats of Infringement Proceedings :
This remedy is available to persons who are not patent owners, and may be
threatened to be sued for infringement.
Under Section 106 of the Act,
if any person whether or not entitled or interested in a patent or application for
patent threatens any other person by circulars or advertisements or by
communications oral or in writing addressed to that or any other person
with proceedings for infringement of a patent,
any person aggrieved thereby may bring a suit against him praying for the
following reliefs as mentioned below unless the defendant proves that the acts in
respect of which the proceedings were initiated constitute or if done would
constitute an infringement of a patent.
1. A declaration to the effect that such threats are unjustifiable
2. An injunction against the continuance of the threats
3. Damages if any sustained thereby
Doctrine of res judicata :
It is ill-advised to assume that if a pre-grant opposition has been filed and
adjudicated, then the doctrine of res judicata operates preventing the same person
from filing a post-grant opposition or a revocation.
Due to changes in the claims during prosecution the grounds of post-grant

CONTENTS | Module-1 | Module-2 | Module-3 | Module-4 http://duralex.bhatt.net.in/


https://www.facebook.com/groups/Dharmo.Rakshati.Rakshitah/ Page - 194 of 196

opposition may be different.


It has to be decided on a case-to-case basis.
However, the situation may be treated differently where action for revocation is
being contemplated after a patent has been granted.
Case-law :
In the case of Alloys Wobben and anr v Yogesh Mehra and ors, 2014 (Supreme
Court Civil Appeal no. 6718 of 2013) the Honble Supreme Court held that
if a party has filed a post-grant-opposition, the remedies or revocation and/ or
counter-claim for revocation shall not lie.
The Judgement also clarifies that where a party has filed a Petition for revocation
of Patent, he shall not be eligible to file a counter-claim for revocation in a Suit
for infringement of that patent.
This is to avoid multiplicity of actions on a patent;
however there is no bar on institution of multiple challenges to a granted patent
initiated by different parties.
(ii) DEFENCES IN A SUIT FOR INFRINGEMENT :
There are altogether 17 grounds for the revocation of a patent. Some of these
grounds are merely formal while the others strike at the root of technical criteria of
patentability.
A party in a suit for infringement can invoke the defences enshrined in Section 107
which states that
every ground on which a patent may be revoked under Section 64 shall be
available as a ground for defence.
Definition : Section 107 :
(1) In any suit for infringement of a patent every ground on which it may be
revoked under section 64 shall be available as a ground for defence.
(2) In any suit for infringement of a patent by the making, using or importation
of any machine, apparatus of other article or by the using of any process or by
the importation, use or distribution or any medicine or drug,
it shall be a ground for defence that such making, using, importation or
distribution is in accordance with any one or more of the conditions specified in
section 47
In his defence the defendant may also invoke any one of following grounds :
the invention is not useful (S.64(1)(g)),
the claim(s) are not patentable subject matter (S.64(1)(k)),
the geographical origin of the biological material is wrongfully mentioned
(S.64(1)(p)),
or that the claims are anticipated by traditional knowledge (S.64(1)(q)).

CONTENTS | Module-1 | Module-2 | Module-3 | Module-4 http://duralex.bhatt.net.in/


https://www.facebook.com/groups/Dharmo.Rakshati.Rakshitah/ Page - 195 of 196

Further the defendant may invoke the exceptions under Section 47 such as use
for research or for governments own use, as a ground for defence.
One of the main defences in a suit for infringement is the invalidation of patent
claims which is usually by way of filing a counter-claim for revocation under
Section 64, usually along with the written statement.
(iii) Penal Provisions :

FAILURE TO OBTAIN FOREIGN FILING LICENCE UNDER SECTION 39 :
The S.39 requires a patent applicant to first file in India or to obtain a foreign
filing licence (FFL) to first file abroad.
Besides being a ground for the revocation of patent, the failure to obtain FFL
under Section 39 is also an offence.
The Section 118 states that failure to comply with S. 39 (or with the Secrecy
Orders under S.35) shall be punishable with imprisonment for a term which may
be extended to 2 years or with fine or both.
There is no remedy available to a patent applicant for non-compliance of S. 39
requirement to obtain a foreign filing licence to file abroad without first filing in
India,

This is Module-4. Menu ---> CONTENTS | Module-1 | Module-2 | Module-3 | Module-4

*** End-of-Compilation ***


Source : Public domain print/ internet contents.
URLs of some such resources are listed herein above.
Credits/ copyrights duly acknowledged.

Suggested Readings :
Cornish W.R., Intectual Property, Patents, Trade Marks, Copy Rights and Allied Rights,
Asia Law House, Hyderabad.
Vikas Vashishth, Law and Practice of Intellectual Property, Bharat Law House, Delhi.
P. Narayanan, Intellectual Property Law, Eastern Law House, Calcutta.
Dr. B. L. Wadehra, Law relating to Intellectual Property, Universal Law Publishing Co.
Chakravarty's Intellectual Property Law, Ashoka Law House, New Delhi
Bibeck Debroy (ed), Intellectual Property Rights Rajiv Gandhi Foundation, Delhi.
E.I.F. Anderfelt, International Patent Legislation and Developing Countries
W.R. Cornish. Intellectual Property, Sweet and Maxwell.

CONTENTS | Module-1 | Module-2 | Module-3 | Module-4 http://duralex.bhatt.net.in/


https://www.facebook.com/groups/Dharmo.Rakshati.Rakshitah/ Page - 196 of 196

Mata Din, Law of Passing off and Infringement Action of Trade Marks
P.S. Sengal and Kishore Singh, Indian Patent System and Paris Convention : Legal
Perspectives
K. Thairani, Copyright The Indian Experience
W.R. Cornish, Para and Materials on Intellectual Property, Sweet & Maxwell.
N. K. Acharya, Textbook on Intellectual Property Rights, Asia Law House
Dr. S. R. Myneni, Law of Intellectual Property, Asia Law House
Justice P. S. Narayanas Intellectual Property in India, Gogia Law Agency
Manish Arora, Guide to Trademarks Law, Universal Law Book Co.
Iyengar's The Trademarks Act, Universal Law Book Co.
Pr. Ashwani Kr. Bansal, The Designs Law, Universal Book Co.
Dr. B. L. Wadehra, Law Relating to Patent, Trademarks, Copyright, Designs &
Geographical Indication, Universal Law Publishing Co.
Patent Co-operation Treaty

CONTENTS | Module-1 | Module-2 | Module-3 | Module-4 http://duralex.bhatt.net.in/

Vous aimerez peut-être aussi